CBSE Sample Papers for Class 10 Hindi A Set 7 with Solutions

CBSE Sample Papers for Class 10 Hindi A Set 7 with Solutions

Students must start practicing the questions from CBSE Sample Papers for Class 10 Hindi with Solutions Set 7 are designed as per the revised syllabus.

CBSE Sample Papers for Class 10 Hindi A Set 7 with Solutions

निर्धारित समय : 3 घंटे
अधिकतम अंक : 80

सामान्य निर्देश :

  • इस प्रश्नपत्र में दो खंड हैं-खंड ‘क’ और ‘ख’। खंड-क में वस्तुपरक/बहुविकल्पीय और खंड-ख में वस्तुनिष्ठ/ वर्णनात्मक प्रश्न दिए गए हैं।
  • प्रश्नपत्र के दोनों खंडों में प्रश्नों की संख्या 17 है और सभी प्रश्न अनिवार्य हैं।
  • यथासंभव सभी प्रश्नों के उत्तर क्रमानुसार लिखिए।
  • खंड ‘क’ में कुल 10 प्रश्न हैं, जिनमें उपप्रश्नों की संख्या 49 है। दिए गए निर्देशों का पालन करते हुए 40 उपप्रश्नों के उत्तर देना अनिवार्य है।
  • खंड ‘ख’ में कुल 7 प्रश्न हैं, सभी प्रश्नों के साथ उनके विकल्प भी दिए गए हैं। निर्देशानुसार विकल्प का ध्यान रखते हुए सभी प्रश्नों के उत्तर दीजिए।

खंड ‘क’
वस्तुपरक/बहुविकल्पीय प्रश्न (अंक : 40)

प्रश्न 1.
नीचे दिए गए गद्यांश को ध्यानपूर्वक पढ़िए और उस पर आधारित प्रश्नों के उत्तर दीजिए।(1 × 5 = 5)
पुरुष और स्त्री एक-दूसरे के पूरक हैं। नर के बिना नारी का जीवन अधूरा है तथा नारी के बिना नर के अस्तित्व की कल्पना नहीं की जा सकती। इसमें संदेह नहीं है कि सृष्टि के अस्तित्व के लिए दोनों का ही होना अनिवार्य है।

प्राचीन भारत के इतिहास में नारियों की गौरवमयी गाथाएँ भरी पड़ी हैं। हमारे प्राचीन ग्रंथों का कथन है ‘यत्र नार्यस्तु पूज्यंते रमते तत्र देवता।’ यहाँ पूजा से तात्पर्य नारी की मान-मर्यादा और अधिकारों की रक्षा करने से है। प्राचीन काल में नारी को लक्ष्मी और गृह-देवी के नाम से संबोधित किया जाता था। पुरुष के समान ही उन्हें भी शिक्षा मिलती थी। जीवन के बड़े-से-बड़े धार्मिक अनुष्ठान उनके बिना पूरे नहीं होते थे। देवासुर संग्राम में कैकेयी के अद्वितीय कौशल को देखकर दशरथ भी चकित रह गए थे। मैत्रेयी, शकुंतला, अनुसूया, दमयंती, सावित्री आदि स्त्रियाँ भी अपने आप में स्वयंसिद्ध उदाहरण हैं। समय परिवर्तनशील है। आज के समय में भारतीय समाज लड़की की अपेक्षा लड़के को अधिक महत्त्व देता है। ईश्वर ने पुरुष को नारी की अपेक्षा अधिक शक्ति प्रदान की है। वह स्वयं को नारी का संरक्षक मानता है। पुरुष प्रायः धन का अर्जन करता है तथा नारी घर का दायित्व सँभालती है। वर्तमान युग में भी पुरुष को नारी की अपेक्षा अधिक स्वतंत्रता प्राप्त है। भारत में ही नहीं अपितु विश्व के विकसित देशों में भी नारी की स्थिति दयनीय है। उन्हें उचित सम्मान नहीं मिलता। जो देश जितना अधिक विकसित है वहाँ नारी की स्थिति उतनी ही दयनीय है। इसके पीछे कारण यह है कि उन देशों में नारी धन कमाने के लिए घर से बाहर निकलती है। बाहर निकलने का परिणाम यह होता है कि उसको पुरुषों के अहम् से पग-पग पर जूझना पड़ता है। हमारा समाज पुरुष प्रधान है। यहाँ जो भी नियम बनाए जाते हैं वे पुरुषों के द्वारा बनाए जाते हैं। अतः यह कैसे संभव होगा कि वे अपने विरुद्ध नियम बनाएँ। नारी उत्थान और पतन के पीछे भी पुरुष का ही हाथ होता है। नारी से संबंधित जो भी प्रश्न उठाए जाते हैं वे प्रायः पुरुष वर्ग से ही उठते हैं। अतः नारी की हमारे समाज में क्या स्थिति है यह स्वयं ही परिभाषित है। भारत के गांवों में आज भी स्त्रियों को पिता, पति अथवा पुत्र के अधीन रहना पड़ता है। ग्रामीण स्त्रियाँ प्रायः तंगी का शिकार होती हैं। उनके स्वास्थ्य और शिक्षा के विषयों पर भी अधिक ध्यान नहीं दिया जाता।
(क) सृष्टि के अस्तित्व के लिए क्या आवश्यक है?
(i) वनस्पति
(ii) पेड़-पौधे
(iii) पुरुष और स्त्री
(iv) प्रकृति
उत्तर:
(iii) पुरुष और स्त्री।

व्याख्यात्मक हल:
सृष्टि का अस्तित्व पुरुष और स्त्री पर ही निर्भर करता है क्योंकि इनसे ही संतान जन्म लेकर सृष्टि को आगे बढ़ाती है।

(ख) नारी की पूजा का क्या अर्थ है?
(i) नारी को पूजना
(ii) नारी के अधिकारों की रक्षा करना
(iii) नारी को भगवान मानना
(iv) ईश्वर को पूजना
उत्तर:
(ii) नारी के अधिकारों की रक्षा करना

(ग) हमारा समाज पुरुष प्रधान क्यों है?
(i) क्योंकि यहाँ सभी नियम पुरुषों द्वारा बनाए जाते हैं।
(ii) क्योंकि यहाँ पुरुष ही पुरुष निवास करते हैं।
(iii) क्योंकि यहाँ स्त्रियों को कोई अधिकार प्राप्त नहीं हैं।
(iv) क्योंकि स्त्री को नियम बनाने नहीं आते।
उत्तर:
(i) क्योंकि यहाँ सभी नियम पुरुषों द्वारा बनाए जाते हैं।

(घ) देवासुर संग्राम में दशरथ किसके साहस के सामने चकित रह गए थे?
(i) कैकेयी के
(ii) शकुंतला के
(ii) अनुसूया के
(iv) दमयंती के
उत्तर:
(i) कैकेयी के।

(ङ) कथन (A) और कारण (R) को पढ़कर उपयुक्त विकल्प चुनिएकथन (A) : पुरुष स्वयं को नारी का संरक्षक मानता है।
कारण (R):समाज के पुरुष प्रधान होने के कारण जहाँ जो भी नियम बनाए जाते हैं, वे पुरुषों द्वारा बनाए जाते
(i) कथन (A) गलत है किन्तु कारण (R) सही
(ii) कथन (A) और कारण (R) दोनों ही गलत
(iii) कथन (A) सही है और कारण (R) कथन (A) की सही व्याख्या है
(iv) कथन (A) सही है किन्तु कारण (R) कथन (A) की सही व्याख्या नहीं हैं
उत्तर:
(iii) कथन (A) सही है और कारण (R) कथन (A) की सही व्याख्या है

व्याख्यात्मक हल:
पुरुष प्रधान समाज होने के कारण वह स्वयं को नारी के भाग्य का विधाता मानता है इसलिए वह स्वयं को नारी का संरक्षक मानता है।

CBSE Sample Papers for Class 10 Hindi A Set 7 with Solutions

प्रश्न 2.
नीचे 2 पद्यांश दिए गए हैं। किसी 1 पद्यांश को ध्यानपूर्वक पढ़िए और उस पर आधारित प्रश्नों के अतर दीजिए। (1 × 5 = 5)
यदि आप इस पद्यांश का चयन करते हैं तो कृपया उत्तर-पुस्तिका में लिखें कि आप प्रश्न संख्या 2 में दिए गए पद्यांश-1 पर आधारित प्रश्नों के उत्तर लिख रहे हैं। किस भाँति जीना चाहिए किस भाँति मरना चाहिए, सो सब हमें निज पूर्वजों से याद करना चाहिए। पूर्व-चिह्न उनके यत्नपूर्वक खोज लेना चाहिए, निज पूर्व-गौरव-दीप को बुझने न देना चाहिए, आओ मिलें सब देश-बांधव हार बनकर देश के, साधक बनें सब प्रेम से सुख-शांतिमय उद्देश्य के। क्या सांप्रदायिक भेद से है, ऐक्य मिट सकता अहो, बनती नहीं क्या एक माला विविध सुमनों की कहो॥ प्राचीन हों कि नवीन, छोड़ो रूढ़ियाँ जो हों बुरी, बनकर विवेकी तुम दिखाओ हंस जैसी चातुरी, प्राचीन बातें ही भली हैं, यह विचार अलीक है जैसी अवस्था हो जहाँ, वैसी व्यवस्था ठीक है, मुख से न होकर चित्त से देशानुरागी हो सदा। देकर उन्हें साहाय्य भरसक सब विपत्ति व्यथा हरो, निज दुःख से ही दूसरों के दुःख का अनुभव करो।।
(क) पद्यांश में कौन-से गौरव दीपक को नहीं बुझने देने की बात हो रही है?
(i) निज पूर्वजों के
(ii) स्वजनों के
(iii) दुखीजनों के
(iv) रोशनी को
उत्तर:
(i) निज पूर्वजों के।

(ख) पद्यांश में कवि ने क्या त्यागने की बात कही है?
(i) पुरानी बातों को?
(ii) अच्छाइयों को
(iii) बुराइयों को
(iv) गुणों को
उत्तर:
(iii) बुराइयों को।

(ग) ‘हृदय में स्वदेश के प्रति अनुराग होना’ ये अर्थ काव्यांश की किस पंक्ति से प्रकट हो रहा है?
(i) आओ मिलें सब देश-बांधव
(ii) साधक बने सब प्रेम से
(iii) बनती नहीं क्या एक माला
(iv) चित्त से देशानुरागी हो सदा
उत्तर:
(iv) चित्त से देशानुरागी हो सदा।

(घ) विभिन्न सम्प्रदायों को काव्यांश में किसे समान बताया गया है?
(i) विभिन्न पुष्पों के
(ii) विभिन्न धर्मों के
(iii) विभिन्न जातियों के
(iv) विभिन्न मतों के
उत्तर:
(i) विभिन्न पुष्पों के।

(ङ) ‘किस भांति जीना चाहिए, किस भांति मरना चाहिए सो सब हमें निज पूर्वजों से याद करना चाहिए’ – पंक्ति का भाव है
(I) जीवन जीने का सही अर्थ अपने पूर्वजों से सीखना चाहिए।
(II) अपने पूर्वजों के पद चिह्न पर चलना चाहिए।
(III) अपने पूर्वजों द्वारा किए गए कार्यों का अनुसरण करना चाहिए।
(IV) अपनी विरासत पर गर्व करना चाहिए।
(i) कथन (II) सही है
(ii) कथन (I), (II) व (III) सही हैं
(iii) कथन (I), (II) व (IV) सही हैं
(iv) कथन (I), (II), (III) व (IV) सही हैं
उत्तर:
(ii) कथन (I), (II) व (III) सही हैं।

अथवा
यदि आप इस पद्यांश का चयन करते हैं तो कृपया उत्तर-पुस्तिका में लिखें कि आप प्रश्न संख्या 2 में दिए गए पद्यांश-II पर आधारित प्रश्नों के उत्तर लिख रहे हैं।
नीड़ का निर्माण फिर-फिर,
नेह का आह्वान फिर-फिर!
वह उठी आंधी कि नभ में
छा गया सहसा अँधेरा,
धूलि धूसर बादलों ने
भूमि को इस भांति घेरा,
रात-सा दिन हो गया, फिर
रात आई और काली,
लग रहा था अब न होगा
इस निशा का फिर सवेरा
रात के उत्पात भय से
भीत जन-जन, भीत कण-कण
किन्तु प्राची से उषा की
मोहिनी मुस्कान फिर-फिर,
नेह का आह्वान फिर-फिर!

(क) ‘छा गया सहसा अँधेरा’ पंक्ति का भाव है
(i) सहसा बादलों का छा जाना
(ii) सहसा धुल भरी आंधी चलना
(iii) सहसा जीवन में कष्टों का आगमन
(iv) सहसा बिजली का चले जाना
उत्तर:
(iii) सहसा जीवन में कष्टों का आगमन।

(ख) रात में आने पर कवि को किस बात का डर लगा
(i) रास्ता भटकने का डर
ii) मुसीबतें समाप्त न होने का डर
(iii) रात में अकेले होने का डर
(iv) रात न समाप्त होने का डर
उत्तर:
(ii) मुसीबतें समाप्त न होने का डर।

(ग) उषा की मोहिनी मुस्कान क्या संदेश देती है?
(i) सवेरा होने पर अँधेरा दूर होने का
(ii) सवेरे सबका मन मोह लेने का
(iii) सवेरे सभी का काम में लग जाना
(iv) दुःख के बाद सुख के आगमन का
उत्तर:
(iv) दु:ख के बाद सुख का आगमन होता है।

(घ) कवि नीड़ का पुनर्निर्माण क्यों करना चाहता है?
(i) क्योंकि वह घोंसला बनाना जानता है
(ii) क्योंकि वह पक्षियों को आश्रय देना चाहता
(iii) क्योंकि वह आशावादी है
(iv) क्योंकि वह दुखी रहता है
उत्तर:
(iii) क्योंकि वह आशावादी है।

(ङ) कथन (A) और कारण (R) को पढ़कर उपयुक्त विकल्प चुनिएकथन (A) : मनुष्य को कष्टों से घबराना नहीं चाहिए।
कारण (R) : जीवन में आशावादी मनुष्य ही आगे बढ़ सकता है।
(i) कथन (A) गलत है किन्तु कारण (R) सही
(ii) कथन (A) और कारण (R) दोनों ही गलत
(iii) कथन (A) सही है और कारण (R) कथन (A) की सही व्याख्या है
(iv) कथन (A) सही है किन्तु कारण (R) कथन (A) की सही व्याख्या नहीं है
उत्तर:
(iii) कथन (A) सही है और कारण (R) कथन (A) की सही व्याख्या है।

CBSE Sample Papers for Class 10 Hindi A Set 7 with Solutions

प्रश्न 3.
निम्नलिखित में से किन्हीं चार प्रश्नों के लिए उचित विकल्प चुनिए। (1 × 4 = 4)
(क) “यद्यपि वह बीमार था तथापि मेहनत करता रहा”- वाक्य का सरल वाक्य में रूप होगा
(i) बीमार होने पर भी वह मेहनत करता रहा।
(ii) वह बीमार था लेकिन फिर भी मेहनत करता रहा।
(iii) वह बीमार था इसलिए मेहनत करता रहा।
(iv) मेहनत करने के कारण वह बीमार हो गया।
उत्तर:
(i) बीमार होने पर भी वह मेहनत करता रहा।

व्याख्यात्मक हल:
सरल वाक्य में एक उद्देश्य और एक क्रिया होते हैं। अतः यही सही उत्तर है।

(ख) हमारे घर पहुँचकर फुटबॉल खेलने लगा। रचना के आधार पर वाक्य भेद बताइए।
(i) सरल वाक्य
(ii) संयुक्त वाक्य
(iii) मिश्र वाक्य
(iv) प्रधान उपवाक्य
उत्तर:
(i) सरल वाक्य।

व्याख्यात्मक हल:
सरल वाक्य में एक उद्देश्य और एक क्रिया होते हैं। अतः यही सही उत्तर है।

(ग) जो लोग ईमानदार होते हैं, उनका सर्वत्र आदर होता है। रचना के आधार पर वाक्य भेद बताइए।
(i) सरल वाक्य
(ii) संयुक्त वाक्य
(iii) मिश्र वाक्य
(iv) आश्रित उपवाक्य
उत्तर:
(iii) मिश्र वाक्य।

व्याख्यात्मक हल:
मिश्र वाक्य में एक वाक्य प्रधान और दूसरा आश्रित उपवाक्य होता है। इस वाक्य में ‘जो लोग ईमानदार होते हैं’ उपवाक्य है।

(घ) उसे घर जाना था इसलिए वह चला गया रचना के आधार पर वाक्य भेद बताइए
(i) सरल वाक्य
(ii) संयुक्त वाक्य
(iii) मिश्र वाक्य
(iv) संज्ञा उपवाक्य
उत्तर:
(ii) संयुक्त वाक्य।

व्याख्यात्मक हल:
संयुक्त वाक्य में दो सरल वाक्य होते हैं जो अपनेआप में पूर्ण अर्थ को व्यक्त करते हैं। यहाँ पहला वाक्य ‘उसे घर जाना था’ और दूसरा वाक्य ‘वह चला गया’ स्वतंत्र वाक्य हैं जो आपस में ‘इसलिए’ योजक द्वारा जुड़े हुए हैं।

(ङ) कॉलम 1 को कॉलम 2 के साथ सुमेलित कीजिए और सही विकल्प चुनकर लिखिए

कॉलम 1 कॉलम 2
1. यह वही छात्र है जिसने (I) संज्ञा आश्रित प्रथम स्थान प्राप्त किया उपवाक्य था।
2. श्यामा ने कहा था कि वह आज जल्दी जाएगी। (II) क्रियाविशेषण उपवाक्य
3. जब तुम जाओगे तभी मैं उपवाक्य (III) विशेषण जाऊँगा।

(i) 1-(III), 2-(I), 3-(II)
(ii) 1-(II), 2-(III), 3-(I)
(iii) 1-(I), 2-(II), 3-(III)
(iv) 1-(III), 2-(II), 3-(I)
उत्तर:
(i) 1-(III), 2-(I), 3-(II)

प्रश्न 4.
निम्नलिखित में से किन्हीं चार प्रश्नों के लिए उचित विकल्प चुनिए। (1 × 4 = 4)
(क) सरला बहुत मधुर गीत गाती है। वाक्य का कर्मवाच्य में रूप होगा
(i) सरला के द्वारा मधुर गीत गाया जाता है।
(ii) सरला ने मधुर गीत गाया।
(iii) मधुर गीत गाया जाता है सरला के द्वारा।
(iv) गीत गाया मधुर सरला ने
उत्तर:
(i) सरला के द्वारा मधुर गीत गाया जाता है।

व्याख्यात्मक हल:
कर्मवाच्य में परिवर्तन करते समय कर्ता के साथ ‘से’ या ‘द्वारा’ लगाया जाता है और क्रिया को कर्म के अनुसार परिवर्तित किया जाता है।

(ख) मीरा द्वारा आज बहुत लम्बी कविता सुनाई गई। वाक्य का कर्तृवाच्य में रूप होगा
(i) मीरा ने आज बहुत लम्बी कविता सुनाई।
(ii) मीरा से बहुत लम्बी कविता सुनाई गई।
(iii) लम्बी कविता मीरा के द्वारा सुनाई गई।
(iv) लम्बी कविता मीरा ने सुनाई।
उत्तर:
(i) मीरा ने आज बहुत लम्बी कविता सुनाई।

व्याख्यात्मक हल:
कृतवाच्य में वाक्य की क्रिया कर्ता के अनुसार परिवर्तित होती है और यहाँ कर्ता के साथ लगे हुए ‘द्वारा’ को हटा दिया जाता है।

(ग) सीमा समझाने पर भी नहीं समझती। वाक्य का भाववाच्य में रूप होगा
(i) सीमा से समझाने पर भी नहीं समझा गया।
(ii) समझाने पर भी सीमा से नहीं समझा गया।
(iii) नहीं समझा जाता सीमा से।
(iv) नहीं समझती सीमा
उत्तर:
(ii) समझाने पर भी सीमा से नहीं समझा गया।

व्याख्यात्मक हल:
भाववाच्य में परिवर्तित करते समय क्रिया को एकवचन, पुल्लिग में परिवर्तित कर दिया जाता है और भाव की प्रधानता रखी जाती है।

(घ) मैं कल आपके लिए उपहार लाऊँगा। वाक्य का कर्मवाच्य रूप होगा
(i) कल उपहार लाया जाएगा मेरे द्वारा।
(iiमैं उपहार लाऊँगा कल आपके लिए।
(iii) मेरे द्वारा कल आपके लिए उपहार लाया जाएगा।
(iv) उपहार लाया कल मैं
उत्तर:
(iii) मेरे द्वारा कल आपके लिए उपहार लाया जाएगा।

(ङ) कॉलम 1 को कॉलम 2 के साथ सुमेलित कीजिए और सही विकल्प चुनकर लिखिए

कॉलम 1  कॉलम 2
1. यह लेख मेरी माँ के द्वारा लिखा गया है। (I) कर्तृवाच्य
2. उसने भगत को दुनियादारी से निवृत्त कर दिया था। (II) भाववाच्य
3. अब मुझसे सहा नहीं जाता। (III) कर्मवाच्य

(i) 1-(III), 2-(I), 3-(II)
(ii) 1-(II), 2-(III), 3-(I)
(iii) 1-(I), 2-(II), 3-(III)
(iv) 1-(III), 2-(II), 3-(I)
उत्तर:
(i) 1-(III), 2-(I), 3-(II)

CBSE Sample Papers for Class 10 Hindi A Set 7 with Solutions

प्रश्न 5.
निम्नलिखित में से किन्हीं चार प्रश्नों के लिए उचित विकल्प चुनिए। (1 × 4 = 4)
(क) हम राहुल के घर गए परन्तु वह वहाँ नहीं मिला। वाक्य में रेखांकित वाक्य के लिए उचित पद परिचय होगा
(i) समुच्चयबोधक अव्यय।
(ii) क्रियाविशेषण।
(iii) संबंधबोधक अव्यय।
(iv) विस्मयादिबोधक अव्यय।
उत्तर:
(i) समुच्चयबोधक अव्यय।

(ख) जो अपना सामान नहीं सँभालता, वह पछताता है। वाक्य में रेखांकित वाक्य के लिए उचित पद परिचय होगा
(i) निश्चयवाचक सर्वनाम, पुल्लिग, एकवचन।
(ii) अनिश्चयवाचक सर्वनाम, पुल्लिग, एकवचन।
(iii) निश्चयवाचक सर्वनाम, पुल्लिग, बहुवचन।
(iv) निश्चयवाचक सर्वनाम, स्त्रीलिंग, एकवचन।
उत्तर:
(ii) अनिश्चयवाचक सर्वनाम, पुल्लिंग, एकवचन।

(ग) मैं वहाँ आठवीं मंजिल पर रहता हूँ। वाक्य में रेखांकित वाक्य के लिए उचित पद परिचय होगा
(i) परिमाणवाचक विशेषण, स्त्रीलिंग, एकवचन, विशेष्य ‘कक्षा’।
(ii) गुणवाचक विशेषण, स्त्रीलिंग, एकवचन, विशेष्य ‘कक्षा’।
(iii) संख्यावाचक विशेषण, स्त्रीलिंग, एकवचन, विशेष्य ‘कक्षा’।
(iv) निजवाचक विशेषण, स्त्रीलिंग, एकवचन, विशेष्य ‘कक्षा।
उत्तर:
(iii) संख्यावाचक विशेषण, स्त्रीलिंग, एकवचन, विशेष्य ‘मंजिल’।

(घ) वह प्रातः तेज चाल से चलता हुआ विद्यालय जाता है। वाक्य में रेखांकित वाक्य के लिए उचित पद परिचय होगा
(i) परिमाणवाचक क्रियाविशेषण।
(ii) स्थानवाचक क्रियाविशेषण।
(iii) गुणवाचक क्रियाविशेषण।
(iv) रीतिवाचक क्रियाविशेषण।
उत्तर:
(iv) रीतिवाचक क्रियाविशेषण।

(ङ) हम उस कस्बे से गुज़रे थे। वाक्य में रेखांकित वाक्य के लिए उचित पद परिचय होगा
(i) सर्वनाम, उत्तमपुरुष, बहुवचन, पुल्लिग।
(ii) सर्वनाम, मध्यमपुरुष, बहुवचन, पुल्लिग।
(iii) सर्वनाम, अन्यपुरुष, बहुवचन, पुल्लिग।
(iv) सर्वनाम , प्रश्नवाचक , बहुवचन, पुल्लिग।
उत्तर:
(i) सर्वनाम , उत्तमपुरुष, बहुवचन, पुल्लिंग।

प्रश्न 6.
निम्नलिखित में से किन्हीं चार प्रश्नों के लिए उचित विकल्प चुनिए।. (1 × 4 = 4)
(क) ‘विधि के समान हैं, विमानीकृत राजहंस’ – पंक्ति में प्रयुक्त अलंकार है
(i) श्लेष
(ii) उत्प्रेक्षा
(iii) मानवीकरण
(iv) अतिशयोक्ति
उत्तर:
(क) (i) श्लेष

व्याख्यात्मक हल:
यहाँ ‘विमानीकृत राजहंस’ के दो अर्थ है- पहला हंसों के राजा को अपना विमान बनाना और दूसरा बड़े-बड़े राजाओं को विमानीकृत करना अर्थात् उनका मान भंग करना।

(ख) ‘सिर फट गया उसका वहीं, मानों अरुण रंग का घड़ा’ – पंक्ति में प्रयुक्त अलंकार है
(i) श्लेष
(ii) उत्प्रेक्षा
(iii) मानवीकरण
(iv) अतिशयोक्ति
उत्तर:
(i) उत्प्रेक्षा

व्याख्यात्मक हल:
यहाँ फटा हुआ सिर उपमेय पर लाल रंग के घड़े की संभावना की गई है अतः उत्प्रेक्षा अलंकार है।

(ग) ‘प्राण छूट प्रथमै रिपु के रघुनायक सायक छूट न पाए’ – पंक्ति में प्रयुक्त अलंकार है
(i) श्लेष
(ii) उत्प्रेक्षा
(iii) मानवीकरण
(iv) अतिशयोक्ति
उत्तर:
(iv) अतिशयोक्ति

व्याख्यात्मक हल:
राम के बाण छूट भी नहीं पाते थे, उससे पहले ही शत्रु के प्राण छूट जाते थे। यह संभव नहीं हो सकता, अतः यहाँ अतिशयोक्ति अलंकार है।

(घ) ‘यह हरा ठिगना चना, बांधे मुरैठा शीश पर’ पंक्ति में प्रयुक्त अलंकार है
(i) श्लेष
(ii) उत्प्रेक्षा
(iii) मानवीकरण
(iv) अतिशयोक्ति
उत्तर:
(iii) मानवीकरण

व्याख्यात्मक हल:
यहाँ चने को मानव के समान व्यवहार करते हुए बताया गया है अतः यहाँ मानवीकरण अलंकार है।

(ङ) ‘मैं रति की प्रतिकृति लज्जा हूँ, शालीनता सिखाती हूँ’ – पंक्ति में प्रयुक्त अलंकार है
(i) श्लेष
(ii) उत्प्रेक्षा
(iii) मानवीकरण
(iv) अतिशयोक्ति
उत्तर:
(ii) मानवीकरण

व्याख्यात्मक हल:
यहाँ लज्जा का मानवीकरण हुआ है कि वह युवतियों को शालीनता की शिक्षा देती है।

CBSE Sample Papers for Class 10 Hindi A Set 7 with Solutions

प्रश्न 7.
निम्नलिखित गद्यांश को पढ़कर पूछे गए प्रश्नों के लिए उचित विकल्प चुनिए। (1 × 5 = 5)
गर्मियों में उनकी ‘संझा’ कितनी उमस भरी शाम को शीतल करती अपने घर के आँगन में आसन जमा बैठते। गाँव के उनके कुछ प्रेमी भी जुट जाते। खंजड़ियों और करतालों की भरमार हो जाती। एक पद बालगोबिन भगत कह जाते, उनकी प्रेमी मंडली उसे दुहराती तिहराती।
धीरे-धीरे स्वर ऊँचा होने लगता एक निश्चित ताल, एक निश्चित गति से। उस ताल स्वर के चढ़ाव के साथ श्रोताओं के मन भी ऊपर उठने लगते। धीरे-धीरे मन तन पर हावी हो जाता। होते-होते, एक क्षण ऐसा आता कि बीच में खंजड़ी लिए बालगोबिन भगत नाच रहे हैं और उनके साथ ही सबके तन और मन नृत्यशील हो उठे हैं। सारा आँगन नृत्य और संगीत से ओत-प्रोत है। (क) भगत के गायन का श्रोताओं पर क्या प्रभाव पड़ता था?
(i) श्रोताओं के मन झूमने लगते।
(ii) लोग आँगन में जमा हो जाते।
(iii) लोग गाने लगते।
(iv) वे सो जाते थे।
उत्तर:
(i) श्रोताओं के मन झूमने लगते।

(ख) भगत और गाँववालों के द्वारा कौन-कौन से वाद्ययंत्र बजाए जाते थे?
(i) खंजड़ी और वीणा
(ii) करताल और ढोलक
(iii) खंजड़ी और करताल
(iv) ढोलक और खंजड़ी
उत्तर:
(iii) खंजड़ी और करताल।

(ग) सारा आँगन नृत्य और संगीत से कब ओत-प्रोत हो जाता
(i) जब गर्मी होती थी
(ii) जब भगत नाचने लगते
(iii) जब श्रोता गीत गाते
(iv) जब वर्षा होती थी
उत्तर:
(ii) जब भगत नाचने लगते।

(घ) गर्मियों में कौन उमस भरी हुई संझा को शीतल करता?
(i) भगत के गीत
(ii) गाँववालों का नाचना
(iii) दोनों सही हैं
(iv) लेखक
उत्तर:
(i) भगत के गीत।

(ङ) गद्यांश किस पाठ में से उद्धृत है?
(i) नेताजी का चश्मा
(ii) लखनवी अंदाज़
(iii) मानवीय करुणा की दिव्य चमक
(iv) बालगोबिन भगत
उत्तर:
(iv) बालगोबिन भगत।

प्रश्न 8.
निम्नलिखित प्रश्नों के लिए उचित विकल्प चुनिए। (1 x 2 = 2)
(क) हालदार साहब के अनुसार देशभक्ति आजकल क्या होती जा रही है ‘नेताजी का चश्मा’ पाठ के आधार पर बताइए।
(i) प्रेम की भावना
(ii) लगाव की भावना
(iii) मजाक की भावना
(iv) त्याग की भावना
उत्तर:
(iii) मज़ाक की भावना।

(ख) लेखक ने सेकण्ड क्लास का टिकट क्यों खरीदा? लखनवी अंदाज़’ पाठ के आधार पर बताइए।
(i) क्योंकि वे एकांत में नयी कहानी सोचना चाहते थे।
(ii) क्योंकि वे बहुत पैसे वाले थे।
(iii) क्योंकि वे नवाब साहब के साथ सफर करना चाहते थे।
(iv) क्योंकि वे इतने ही पैसे लाए थे।
उत्तर:
(i) क्योंकि वे एकांत में नई कहानी सोचना चाहते थे।

व्याख्यात्मक हल:
लेखक को लगा कि सेकण्ड क्लास में भीड़ तो होगी नहीं इसलिए प्राकृतिक दृश्य को देखते हुए वे आसानी से नयी कहानी के बारे में सोच लेंगे।

प्रश्न 9.
निम्नलिखित पद्यांश को पढ़कर पूछे गए प्रश्नों के लिए उचित विकल्प चुनिए। (1×5=5)
मन की मन ही माँझ रही। कहिए जाइ कौन पै ऊधौ, नाहीं परत कही। अवधि अधार आस आवन की, तन मन बिथा सही। अब इन जोग सँदेसनि सुनि-सुनि, बिरहिनि बिरह दही। चाहति हुतौ गुहारि जितहिं तैं, उत तें धार बही। ‘सूरदास’ अब धीर धरहिं क्यौं, मरजादा न लही।।
(क) किसके मन की बात मन में ही रह गई?
(i) उद्धव के
(ii) गोपियों के
(iii) सूरदास के
(iv) श्रीकृष्ण के
उत्तर:
(ii) गोपियों के।

व्याख्यात्मक हल:
कृष्ण के वापस न आने पर गोपियाँ उनसे रुष्ट हो गईं और अपने मन की बात लज्जावश वे उद्धव से नहीं कह पाईं।

(ख) गोपियाँ कौन-सी व्यथा सहन कर रही थीं?
(i) कृष्ण वियोग की
(ii) उद्धव के आने की
(iii) प्रेम में डूबने की
(iv) योग की
उत्तर:
(i) कृष्ण वियोग की।

(ग) किसे सुनकर गोपियों की विरहाग्नि बढ़ गई?
(i) भ्रमर के गुंजन को सुनकर
(ii) सूरदास के गीतों को सुनकर
(iii) कृष्ण के योग संदेश को सुनकर
(iv) स्वयं की आवाज सुनकर
उत्तर:
(iii) कृष्ण के योग संदेश को सुनकर।

(घ) गोपियाँ किस आशा में तन-मन की व्यथा को सहन कर रही थीं?
(i) उद्धव के संदेश को सुनने की
(ii) कृष्ण के आगमन की
(iii) दोनों सही हैं
(iv) उद्धव के आने की
उत्तर:
(ii) कृष्ण के आगमन की।

(ङ) काव्यांश में गोपियाँ कृष्ण को क्या उलाहना दे रही हैं?
(i) योग संदेश भेजने का
(ii) स्वयं न आने का
(iii) प्रेम की मर्यादा का पालन न करने का
(iv) अपने प्रेम का
उत्तर:
(iii) प्रेम की मर्यादा का पालन न करने का।

CBSE Sample Papers for Class 10 Hindi A Set 7 with Solutions

प्रश्न 10.
निम्नलिखित प्रश्नों के लिए उचित विकल्प चुनिए। (1 × 2 = 2)
(क) परशुराम के क्रोधित होने का क्या कारण था?
(i) राम का जनक सभा में आना
(ii) शिवधनुष का टूटना
(iii) लक्ष्मण का परशुराम से बात करना
(iv) राम का चुप रहना
उत्तर:
(ii) शिवधनुष का टूटना।

(ख) आत्मकथ्य कविता में कवि ने अपने जीवन को छोटा क्यों कहा है?
(i) क्योंकि उसके जीवन की कोई विशिष्ट उपलब्धि नहीं है
(ii) क्योंकि उसका जीवन छोटा-सा है
(iii) क्योंकि वह अभी छोटा है
(iv) विकल्प (i) व (ii) सही हैं
उत्तर:
(i) क्योंकि उसके जीवन की कोई विशिष्ट उपलब्धि नहीं हैं।

खंड ‘ब’
वर्णनात्मक प्रश्न (अंक : 40)

प्रश्न 11.
निम्नलिखित प्रश्नों में से किन्हीं तीन प्रश्नों के उत्तर लगभग 25-30 शब्दों में लिखिए। (2 × 3 = 6)
(क) नवाब साहब के किन हाव-भावों से महसूस हुआ कि वे बातचीत करने को तनिक भी उत्सुक नहीं हैं?
उत्तर:
लेखक ने जब रेल-डिब्बे में प्रवेश किया तो नवाब साहब की आँखों में असन्तोष का भाव देखा। नवाब साहब लेखक की ओर देखे बिना खिड़की से बाहर की ओर देखते रहे। उन्होंने संगति के लिए उत्साह नहीं दिखाया इसलिए वे बातचीत के लिए कतई उत्सुक नहीं लग रहे थे।

(ख) बालगोबिन भगत किन विशेषताओं के कारण साधु कहलाते थे?
उत्तर:
बालगोबिन भगत कबीर की विचारधारा में विश्वास करते थे। वे गृहस्थ होते हुए भी साधु जैसा सीधा-सादा जीवन जीते थे। उन्हें सांसारिक वस्तुओं के प्रति मोह न था। अपने नियमों का वे बड़ी कठोरता से पालन करते थे। अपनी इन विशेषताओं के कारण वे साधु कहलाए।

(ग) भाइयों की गतिविधियों का दायरा घर से बाहर और लेखिका का घर तक ही सीमित क्यों था ? ‘एक कहानी यह भी’ के आधार पर बताइए।
उत्तर:
लेखिका मन्नू भंडारी के पिताजी लड़कियों को घर के बाहर निकलने की आजादी देने के पक्षधर नहीं थे। इसी कारण भाइयों की गतिविधियों का दायरा घर से बाहर तक था, क्योंकि वे लड़के थे और लेखिका का दायरा घर तक ही सीमित होने का कारण यह था कि वह एक लड़की थी। लड़का-लड़की में इसी अंतर के कारण वे मोहल्ले आदि से बाहर जाकर खेल सकते थे, परन्तु लेखिका नहीं।

(घ) मूर्ति के पास से गुजरते हुए अंत में हालदार साहब ‘ भावुक क्यों हो उठे थे?
उत्तर:
हालदार साहब ने अंत में जब मूर्ति पर सरकंडे का चश्मा लगा हुआ देखा तो वे ये सोच कर भावुक हो गए कि देश की युवा पीढी अभी भी देशभक्तों का आदर और सम्मान करती है। उनके मन में देशभक्ति की भावना है।

प्रश्न 12.
निम्नलिखित प्रश्नों में से किन्हीं तीन प्रश्नों के उत्तर लगभग 25-30 शब्दों में लिखिए। (2 × 3 = 6)
(क) गोपियों ने यह क्यों कहा कि हरि अब राजनीति पढ़ आए हैं? क्या आपको गोपियों के इस कथन का विस्तार समकालीन राजनीति में नजर आता है? स्पष्ट कीजिए।
उत्तर:
गोपियों के अनुसार जब कृष्ण ब्रज में रहते थे तब सभी ग्वाल-बालों और गोपियों से मिलते-जुलते थे, उनके सखा थे। अब वे मथुरा जाकर राजकाज सँभालकर राजनीति के कुशल खिलाड़ी हो गए हैं। अब उनकी हर बात, हर कार्य में राजनीति झलकती है। उन्होंने हमारे पास उद्धव को योग-संदेश देकर भिजवाने में भी राजनीति की चाल चली। अब उनका व्यवहार राजनीतिज्ञों जैसे छलपूर्ण हो गया है। अब वे सरल हृदय वाले कृष्ण नहीं रहे।

(ख) लक्ष्मण के अनुसार योद्धाओं की क्या-क्या विशेषताएँ
उत्तर:
लक्ष्मण ने वीर योद्धाओं की विशेषताएँ बताते हुए कहा है कि वीर योद्धा स्वयं अपनी वीरता का बखान नहीं करते। वे युद्ध-स्थल में वीरता का प्रदर्शन करते हैं। वे शत्रु को सामने देखकर अपनी वीरता की डींगें नहीं हाँकते, अपशब्दों का प्रयोग नहीं करते, गाली नहीं देते। वीर योद्धा कभी क्रोध नहीं करते, अपितु धैर्य से काम लेते

(ग) मुस्कान के लिए कवि ने ‘दंतुरित’ शब्द का प्रयोग क्यों किया है?
उत्तर:
जब बच्चा मुस्कुराता है तो उसके नए-नए दांत देखकर किसी के भी मन में ममता का भाव फटने लगता है। उसकी मुस्कान जीवित व्यक्ति को तो प्रसन्नता से भर देती है साथ ही मृतक के शरीर में भी संजीवनी का काम करती है। बच्चे की सुंदर दंतावली देखकर उसकी मुस्कान का प्रभाव भी बढ़ जाता है। पाषाण हृदय भी उस मुस्कान को देखकर पिघल जाता है, इसलिए मुस्कान के लिए कवि ने ‘दंतुरित’ शब्द का प्रयोग किया है।

(घ) संगतकार की भूमिका का महत्व कब सामने आता है?
उत्तर:
संगतकार की भूमिका का महत्व तब सामने आता है, जब वह मुख्य गायक के स्वर में स्वर मिलाकर उसके स्वरों को संभालता है। जब मुख्य गायक ऊँचे स्वर में गाता है, तब उसका स्वर भटकने लगता है। उसका उत्साह कम होने लगता है। ऐसे में संगतकार अपने मद्धिम सुर को उसके स्वर में मिलाकर उसे सहारा देता है। उसके ऊँचे स्वर में खोने पर संगतकार ही स्थायी को संभाले रहता है।

प्रश्न 13.
निम्नलिखित प्रश्नों में से किन्हीं दो प्रश्नों के उत्तर लगभग 50-60 शब्दों में लिखिए। (4 × 2 = 8)
(क) ‘कितना कम लेकर ये समाज को कितना अधिक वापस लौटा देती हैं।’ ‘साना-साना हाथ जोड़ि’ पाठ के इस कथन में निहित जीवन मूल्यों को स्पष्ट कीजिए और बताइए कि देश के विकास में आम नागरिक की क्या भूमिका है?
उत्तर:
‘कितना कम लेकर ये समाज को कितना अधिक वापस लौटा देती हैं’ यह कथन लेखिका मन-ही-मन गुनती है। यह कथन पर्वतीय क्षेत्र की कामकाजी महिलाओं की दुर्दशा को भी झलकाता है। यहाँ काम करने वाली स्त्रियाँ मजदूरी के रूप में अपर्याप्त पाती हैं, पर बदले में हमें बहुत कुछ देती हैं। इन्हीं के परिश्रम के बल पर पर्यटन स्थलों का सौन्दर्य बना रहता है। इन्होंने ही पर्वतीय स्थलों को काटकर रास्ते बनाए हैं। ये प्रकृति का संतुलन बनाने में महत्त्वपूर्ण भूमिका निभाती हैं। देश की प्रगति में आम नागरिकों की भूमिका अत्यन्त महत्त्वपूर्ण हो सकती है। सबसे पहले तो उन्हें अपने देश पर गर्व करना होगा। देश के प्रत्येक भाग को स्वच्छ बनाना होगा। पर्यावरण की रक्षा करनी होगी। देश में अधिकाधिक वृक्ष लगाना तथा उनका संरक्षण करना होगा। किसी भी काम को छोटा नहीं समझना होगा। बनाई जाने वाली नीतियाँ आम जनता के हाथों ही सफल हो सकती हैं।

(ख) ‘मैं क्यों लिखता हूँ, पाठ के आधार पर बताइए कि लेखक को लिखने की प्रेरणा कहाँ से प्राप्त होती
उत्तर:
‘मैं क्यों लिखता हूँ’ पाठ के अनुसार लेखक अपनी आन्तरिक विवशता को प्रकट करने के लिए लिखते हैं। वे तटस्थ होकर देखना चाहते हैं कि उनका मन क्या सोचता है। वे लिखकर अपने मन की बेचैनी और उसकी छटपटाहट से मुक्ति पाने के लिए लिखते हैं। वे स्वयं को जानने और समझने के लिए लिखते हैं। वे यह भी मानते हैं कि कई बार कुछ लेखक आर्थिक कारणों से भी लिखते हैं या कुछ संपादक के दबाव और प्रसिद्धि की कामना के लिए भी लिखते हैं।

(ग) ‘माता का अंचल’ पाठ में निहित जीवन-मूल्यों की चर्चा कीजिए।
उत्तर:
‘माता का अंचल’ पाठ वात्सल्यता से परिपूर्ण है। सबसे बड़ा जीवन मूल्य तो इसमें माता-पिता का बच्चे के प्रति दर्शाया गया प्रेम है। अपने बच्चे को पालने-पोसने में एक माँ अपने सारे सुख न्योछावर कर देती है और पिता उसे आत्मनिर्भर और स्वाभिमानी बनाने के लिए अपना सारा प्यार उस पर लुटा देता है। माता-पिता बच्चे के सुख में सुखी होते हैं और उसके दुःख में दुखी। ये सब माता-पिता के वात्सल्य, प्रेम, त्याग और तपस्या पर आधारित होता है जो आगे चलकर बच्चों को अपने माता-पिता की सेवा के लिए तैयार करता हैं।

प्रश्न 14.
निम्नलिखित में से किसी एक विषय पर दिए गए संकेत बिन्दुओं के आधार पर लगभग 120 शब्दों में एक अनुच्छेद लिखिए।
(क) मन की एकाग्रता-सफलता का मूल मंत्र

  • मन की एकाग्रता क्या और क्यों?
  • सफलता की कुंजी
  • सतत् अभ्यास।

अथवा

(ख) पश्चिम का आकर्षण पश्चिम की चमक-दमक आकर्षण के कारण उपाय।

अथवा

(ग) इण्टरनेट और उसके प्रभाव इण्टरनेट क्या है? (6)

  • मानव मन पर प्रभाव
  • सुझाव।

उत्तर:
(क) मन की एकाग्रता-सफलता का मूल मंत्र
मन की एकाग्रता का क्या अर्थ है? मन को सर्वाधि क चंचल माना गया है। मन में अनेक प्रकार के विचार आते रहते हैं। मन को पूर्णतः स्थिर करके तो नहीं रखा जा सकता, पर उस पर नियंत्रण का प्रयास अवश्य किया जा सकता है। मन पर यह नियंत्रण ही मन की एकाग्रता कहलाता है। जब हम अपने मन को किसी एक दिशा या एक बिन्दु पर टिका देते हैं, तब मन एकाग्र हो जाता है। ऐसी मनःस्थिति में हम किसी एक विचार पर ही अपना पूरा ध्यान देते हैं। उस समय हमें इधर-उधर की बातें नहीं सूझती। यह स्थिति ध्यान लगाने जैसी है। ध्यान लगाना अर्थात् मन को टिकाना आवश्यक है क्योंकि चाहे वह आत्म-चिंतन के लिए हो या किसी काम में सफल होने के लिए, इसके लिए सतत् अभ्यास की आवश्यकता होती है। ध्यान लगाने में अभ्यास आवश्यक है। मन की एकाग्रता को प्रायः साधु-सन्तों, योगियों, महात्माओं का विषय माना जाता है. पर जब से योग-प्राणायाम- ध्यान की ओर सामान्य लोगों का ध्यान गया है, तब से मन की एकाग्रता पर भी ध्यान जाने लगा है। ध्यान लगाने से मन एक बिन्दु पर टिक जाता है। जब हम अपने मन को एकाग्र कर लेते हैं तब विषय विशेष पर गहराई से सोच पाते हैं और सोचे हुए काम में सफलता अवश्य मिल जाती है। ऐसी मनोदशा में हमारे सारे प्रयत्न एक दिशा में लग जाते हैं। तब हम इधर-उधर अपना मन भटकने नहीं देते। मन में एक ही विचार आना तथा उसी को क्रियान्वित करना और उसमें अपनी पूरी शक्ति लगा देना ही मन की एकाग्रता है। मन की एकाग्रता कार्य में सफलता दिखाती है। इसे हम सफलता का मूल मंत्र भी कह सकते हैं। यह मंत्र हमारे पास है। हम जब चाहें तभी इसे घुमा सकते हैं। हमारा अपने मन पर नियंत्रण होना चाहिए। इस एकाग्रता को अभ्यास के द्वारा पाया जा सकता है। सफलता पाने का मूल मंत्र मन की एकाग्रता ही है। चंचल मन डगमगाती नाव की तरह है जिस पर सवार व्यक्ति कभी भी डूब सकता है। यदि पार उतरना है तो नाव रूपी मन को स्थिर करना ही होगा।

अथवा

(ख) पश्चिम का आकर्षण
एक पुरानी उक्ति है-दूर के ढोल सुहावने। यह बात आज के सन्दर्भ में भी उतनी ही सही है। वर्तमान समय में लोगों का पश्चिम की ओर आकर्षण निरन्तर बढ़ता जा रहा है। पश्चिमी जगत की चकाचौंध हमें आकर्षित करती है। वहाँ का रहन-सहन, खान-पान, स्वच्छता एवं सुविधाओं से भरपूर जीवन हमें लुभाता है। पाश्चात्य जगत के प्रति हमारे आकर्षण के पीछे अनेक कारण निहित हैं। पश्चिमी देश सम्पन्न हैं। वहाँ के निवासियों को रहने-सहने और कार्य करने की बेहतर सुविधायें उपलब्ध हैं। उनका जीवन स्तर हमसे कहीं बेहतर है, जबकि भारत 70 साल के उपरांत भी सभी को आवास, भोजन, वस्त्र और चिकित्सा जैसी मूलभूत आवश्यकतायें पूरी नहीं कर पाया है। पश्चिमी देशों में समृद्धि है, वे विकसित हैं तथा वहाँ विज्ञान के नवीनतम उपकरण उपलब्ध हैं, वहाँ काम की कद्र है। इसलिए भारत के प्रतिभावान डॉक्टर, इंजीनियर, आई. टी. क्षेत्र के विशेषज्ञ वहाँ जाकर बसने की आकांक्षा मन में पाले रहते हैं। यह एक प्रकार का प्रतिभा पलायन है। जब तक हमारा देश ये सुविधायें नहीं दे पाता, तब तक उनके लिए पश्चिमी देशों का आकर्षण बना रहेगा। इस प्रतिभा पलायन को रोकना मुश्किल बना रहेगा। न केवल उच्च शिक्षित लोगों के ही कदम पश्चिमी देशों की ओर बढ़ रहे हैं, अपितु छोटे-मोटे कामगार भी वहाँ जाकर काम करना चाहते हैं। वहाँ इनकी आवश्यकता है तथा छोटे कामगारों का सम्मान भी है। वहाँ उन्हें काम के बदले भरपूर वेतन मिलता है। कुछ कुशल उद्योगपतियों ने पश्चिमी देशों में अपना उद्योग भी जमा लिया है। अपने नागरिकों को इस पश्चिमी सभ्यता के आकर्षण से बचाने के लिए भारत को भी विकास की प्रक्रिया में तेजी से आगे बढना होगा। पिछले 3.4 वर्षों में भारत निरन्तर विकास की दिशा में आगे बढ़ रहा है।

अथवा

(ग) इण्टरनेट और उसके प्रभाव
कम्प्यूटर और इण्टरनेट के सहयोग से आज का मानव विश्व के किसी भी भाग से किसी भी प्रकार की सूचना प्राप्त कर सकता है। आज की सूचना प्रौद्योगिकी कम्प्यूटर और इण्टरनेट पर आधारित है। अब दफ्तरों में फाइलों का ढेर रखने की आवश्यकता नहीं रह गई है, इसके लिए एक छोटी-सी फ्लॉपी ही पर्याप्त है। अब संदर्भ ग्रन्थों से भरी आलमारियों की आवश्यकता नहीं है। कहीं की टिकट बुक करानी हो, कोई बिल जमा करवाना हो, बैंक से लेन-देन करना हो अथवा मनचाही सूचना पानी हो तो इण्टरनेट का सहारा लेना ही काफी होता है। यह मात्र तकनीक होते हुए भी जीवन्त शक्ति, मित्र व सहयोगी प्रतीत होती है। हम इसके माध्यम से सूचना, मनोरंजन, ज्ञान और व्यक्तिगत तथा सामाजिक विचारों का आदान-प्रदान कर सकते हैं। अब इण्टरनेट पत्रकारिता का युग है। इण्टरनेट का प्रयोग खबरों के सम्प्रेषण के लिए किया जा रहा है। इण्टरनेट ने शोध का काम बहुत आसान कर दिया है। इण्टरनेट का ‘ब्राउजर’ वह औजार है जिसके जरिए हम विश्वव्यापी जाल में गोते लगा सकते हैं। इण्टरनेट के जहाँ अनेक लाभ हैं, वहीं इसके कुछ दुष्प्रभाव भी देखने में आ रहे हैं। इण्टरनेट अश्लीलता, दुष्प्रचार और गन्दगी फैलाने का माध्यम बनता जा रहा है, किन्तु यह कमी उस तकनीक की नहीं, अपितु मानव मस्तिष्क की है जो उसका दुरुपयोग कर रहा है। इण्टरनेट जनसंचार का सबसे नया तथा लोकप्रिय माध्यम है। यह एक ऐसा माध्यम है जिसमें प्रिण्ट मीडिया, टेलीविजन, पुस्तक, सिनेमा तथा पुस्तकालय के सभी गुण विद्यमान हैं। इसकी पहुँच दुनिया के कोने-कोने तक है। यह अनेक माध्यमों का संगम है। इण्टरनेट पर बैठकर आप देश-विदेश में वार्तालाप कर सकते हैं, मन के मुताबिक ब्लॉग बना सकते हैं। इसकी सहायता से किसी भी वार्ता में भाग लिया जा सकता है। व्हाट्सएप, फेसबुक, ट्विटर आदि ने परस्पर सम्पर्कों को नया आयाम दिया है। यह एक अंतर क्रियात्मक माध्यम है। हम मक दर्शक बनकर नहीं बैठते, अपितु बहस के सूत्रधार हो सकते हैं।

CBSE Sample Papers for Class 10 Hindi A Set 7 with Solutions

प्रश्न 15.
प्लास्टिक की चीजों से हो रही हानि के बारे में किसी समाचार-पत्र के सम्पादक को पत्र लिखकर अपने सुझाव दीजिए।
अथवा
रचित आपका मित्र है और उसने राष्ट्रीय स्तर पर खेल प्रतियोगिता में स्वर्ण पदक प्राप्त कर देश का नाम रोशन किया है, उसे बधाई देते हुए पत्र लिखिए। (5)
उत्तर:
सम्पादक महोदय,
नव निर्माण समाचार-पत्र
नई दिल्ली।
विषय-प्लास्टिक की हानियाँ बताते हुए उसे रोकने हेतु सुझाव।
महोदय,
मैं नव निर्माण समाचार-पत्र की एक नियमित पाठिका रही हूँ। आपके लोकप्रिय समाचार-पत्र के माध्यम से मैं प्लास्टिक की चीजों से हो रही हानि के बारे में लोगों को सावधान करना चाहती हूँ। आशा है आप जनहित में मेरा पत्र अवश्य प्रकाशित करेंगे।

प्लास्टिक कभी भी मूल रूप को नहीं छोड़ता। वह अपघटित होकर जहरीली गैसें छोड़ता है। यह हमारे स्वास्थ्य के लिए बहुत हानिकारक है। प्लास्टिक की थैलियाँ नालियों को अवरुद्ध कर देती हैं। इसे पशु खा जाते हैं तथा उनके लिए यह अत्यधिक हानिकारक सिद्ध होती है। प्लास्टिक भूमि प्रदूषण का सबसे बड़ा कारण है। पर्वतीय स्थलों पर भी प्लास्टिक के अवशेष जमा होते जा रहे हैं। सरकार ने प्लास्टिक की थैलियों पर प्रतिबंध लगा दिया है, पर उसका असर तभी नजर आएगा जब जनता इसमें सहयोग देगी। केवल योजनायें बनना पर्याप्त नहीं हो सकता, हम सबको मिलकर इसे सफल बनाना है।
भवदीया,
क ख ग
संयोजिका, स्वच्छ भारत अभियान, नई दिल्ली।
दिनांक……………………….
अथवा
परीक्षा भवन,
नई दिल्ली।
दिनांक…………………………
प्रिय मित्र, सप्रेम नमस्ते। मुझे समाचार-पत्र में यह पढ़कर हार्दिक प्रसन्नता हुई कि मेरे मित्र ने राष्ट्रीय स्तर पर स्वर्ण पदक प्राप्त करके अपना, अपने परिवार तथा पूरे देश का नाम रोशन कर दिया है। इस शानदार सफलता पर मेरी हार्दिक बधाई स्वीकार करो। मैं जानता हूँ कि इस सफलता के पीछे तुम्हारी कड़ी मेहनत व संघर्ष छिपा है। आज तुम्हारा वर्षों का सपना पूरा हुआ है। तुमने इस सफलता से हमारे प्रदेश का मस्तक गर्व से ऊँचा किया है। वास्तव में हमें तुम पर गर्व है। एक बार पुनः बधाई।
तुम्हारा अभिन्न मित्र
क ख ग।

प्रश्न 16.
आप दुर्गापुर निवासी हेमलता जैन हैं। आंगनबाड़ी में सुपरवाइजर के पद के लिए लगभग 80 शब्दों में स्ववृत्त तैयार कीजिए।
अथवा
यूनियन बैंक, महावीर नगर, जयपुर में आपका बचत खाता है। आपकी चैक बुक समाप्त हो गई है। बैंक के मैनेजर को नई चैक बुक जारी करने के लिए ई-मेल लिखिए। (5)
उत्तर:
सेवा में
अधिकारी महोदय
आंगनबाड़ी केंद्र,
दुर्गापुर, जयपुर
विषय-सुपरवाइजर पद के लिए आवेदन-पत्र।
महोदय,
आज दिनांक 12 अगस्त, 20XX को प्रकाशित दैनिक भास्कर में प्रकाशित समाचार-पत्र में आंगनबाड़ी में सुपरवाइजर की आवश्यकता है। इस पद हेतु मैं अपना आवेदन प्रस्तुत कर रही हूँ। विज्ञापन में वर्णित सभी योग्यताओं को मैं पूरा करती हूँ। मेरा संक्षिप्त विवरण इस प्रकार है
अतः श्रीमान से निवेदन है कि मुझे आंगनबाड़ी में सुपरवाइजर के पद पर नियुक्त करने की कृपा करें।
धन्यवाद
निवेदक

हेमलता जैन
त्रिवेणी नगर चौराहा,
जयपुर
12 अगस्त, 20XX
मेरा स्ववृत्त आवेदन-पत्र के साथ संलग्न है।

स्ववृत्त

नामः हेमलता जैन
पिता का नामः राम प्रकाश जैन
माता का नामः संध्या जैन
जन्मतिथि: 06 अगस्त, 1994
वर्तमान पताः 3, त्रिवेणी नगर चौराहा, जयपुर
मोबाइलः 2XX0000546
ईमेल: [email protected]

शैक्षणिक योग्यता
CBSE Sample Papers for Class 10 Hindi A Set 7 with Solutions 1
अन्य योग्यताएँ

  • कम्प्यूटर में 1 वर्ष का डिप्लोमा
  • हिंदी, अंग्रेजी व स्थानीय स्पेनिश भाषा की जानकारी।

उपलिब्धयाँ

  • विद्यालय स्तर पर लेखन प्रतियोगिता में प्रथम पुरस्कार।
  • राष्ट्रीय स्तर पर समाजसेवा हेतु चयन।
  • अखिल भारतीय हिंदी भाषण प्रतियोगिता में प्रथम पुरस्कार

कार्येत्तर गतिविधियाँ और अभिरुचियाँ
हिंदी लेखन में विशेष रुचि।
सांस्कृतिक कार्यक्रम को आयोजन करने का अनुभव तथा रुचि
हस्ताक्षर
हेमलता जैन

दिनांक: X/XX/XXXX
अथवा
From : [email protected]
To : [email protected]
Cc: …………..
Bcc : ……
विषय : नई चैक बुक जारी करने हेतु महोदय निवेदन है कि आपके बैंक में मेरा बचत खाता है। इसका विवरण निम्नानुसार है
नामः रवि गुप्ता
पता: 12, महावीर नगर, जयपुर
खाता संख्या: 23455XXXX5
अतः आपसे अनुरोध है कि यथाशीघ्र 20 चैकों वाली एक नई चैक-बुक मेरे आवास पर भिजवाने की व्यवस्था करें।
धन्यवाद
भवदीय
क. ख. ग.

CBSE Sample Papers for Class 10 Hindi A Set 7 with Solutions

प्रश्न 17.
किसी दर्दनिवारक तेल के लिए लगभग 60 शब्दों में एक आकर्षक विज्ञापन तैयार कीजिए।
अथवा
सभी प्रदेशवासियों को होली के शुभ अवसर पर लगभग 80 शब्दों में एक शुभकामना सन्देश तैयार कीजिए। (4)
उत्तर:
CBSE Sample Papers for Class 10 Hindi A Set 7 with Solutions 2
अथवा

शुभकामनाएँ
रंगों और प्रेम से सराबोर विविध रंगों में अभिव्यक्त करने वाले महापर्व की सभी को बधाई

होली के पवन पर्व पर सभी प्रदेशवासियों को हार्दिक शुभकामनाएँ !
आप सभी इस अवसर पर हमेशा खुशी के रंगों से सराबोर रहे।
प्रदेशाध्यक्ष

CBSE Sample Papers for Class 11 Political Science with Solutions

CBSE Sample Papers for Class 11 Political Science with Solutions 2022-2023

Solved CBSE Sample Paper Class 11 Political Science 2022-2023 with Solutions: Solving Pre Board CBSE Sample Papers for Class 11 Political Science with Solutions Answers 2022-2023 Pdf Download helps to understand the pattern of questions asked in the board exam. Know about the important concepts to be prepared for CBSE Class 11 Political Science board exam and Score More marks. Here we have given CBSE Class 11 Political Science Sample Papers 2023.

Board – Central Board of Secondary Education (CBSE)
Subject – CBSE Class 11 Political Science
Year of Examination – 2022-2023

CBSE Sample Paper Class 11 Political Science 2022-2023 with Solutions

According to the new CBSE Exam Pattern, MCQ Questions for Class 11 Political Science Carry 20 Marks. Click on the link below to access the CBSE Class 11 Political Science Sample Paper 2023 Solved.

CBSE Class 11 Political Science Sample Papers 2023 with Solutions

Political Science Sample Paper Class 11 Question Paper Design 2022-23

Class – XI
Political Science (Code No. 028)
One Theory Paper
Total Marks: 100(80 + 20)
Time: 3 Hours

Units Marks
Part A: Indian Constitution at Work
1. Constitution 12
2. Election and Representation 10
3. The Legislature
4. The Executive 8
5. The Judiciary
6. Federalism 10
7. Local Governments
Total 40
Part B: Political Theory
8. Political Theory: An Introduction 4
9. Liberty 10
10. Equality
11. Justice 8
12. Rights
13. Citizenship 10
14. Nationalism
15. Secularism 8
Total 40
Project Work 20
Grand Total 100

We hope these CBSE Sample Papers for Class 11 Political Science with Solutions 2022-2023 will help in self-evaluation. Stay tuned for further updates on the CBSE Sample Paper of Political Science Class 11 2023 for their exam preparation.

CBSE Sample Papers for Class 12 English Set 11 with Solutions

CBSE Sample Papers for Class 12 English Set 11 with Solutions

Students must start practicing the questions from CBSE Sample Papers for Class 12 English with Solutions Set 11 are designed as per the revised syllabus.

CBSE Sample Papers for Class 12 English Set 11 with Solutions

Time : 3 Hours
Maximum Marks: 80

General Instructions:

  1. 15 Minute prior reading time allotted for Q-paper reading.
  2. The Question Paper contains THREE Sectians-Reading, Grammar & Writing and Literature.
  3. Attempt questions based on specific instructions for each part.

Section – A
Reading Skills (20 Marks)

Question 1.
Read the passage given below. (1 x 10 = 10)
Phases of Grief
(1) There is a time when everyone has to face grief. When someone dear to oneself dies, one is left in a state in which grief and shock overcome the ordinary life. One is left in a situation when one feels there is nothing one can do. Grief and mourning are always associated with death. It also happens with other kinds of losses. It could be the loss of one’s job, loss of one’s house or the loss of a close friend.

(2) The best people able to cope with these are those, who come from cultures that have strict, formal and intensive mourning rituaL In India, we have diverse rituals which nowadays are being discarded. But these help in surmounting the crisis. The near and dear ones come to share the grief and apply balm with their words of solace.

In a modern society, one is expected to act as if nothing has happened, but research has shown that mourning is an essential part of coming to terms with loss. There comes a time when one has to give up one’s grief and rejoin the main stream of life. Mourning in itself passes through these phases. Firstly, it is in the nature of shock and disbelief. A feeling of numbness overtakes. The brain is not in a position to creeps in.

(3) Here, the affected person displays odd behaviour and has difficulty in activities like eating and sleeping. One may remain in this stage for weeks, months and sometimes years. In the next phase, one experiences relief from pain and negative feelings. This leads to the positive side. Then one feels one must make alternative arrangements or replacements. Here, one is ready to cope with the situation. One knows one cannot recover what one has lost, but is conscious of the future, accepts the loss and is ready for the alternative.

(4) On passing through all the stages of grief it seems that time has passed like a river under the bridge. This shows that mourning has been successful. One cannot forget the loss but one comes to terms with the reality..

On the basis of your reading of the above passage, answer the following questions:
(A) Complete the sentence with reference to the passage.
After one has passed all the phases of grief there is a relief from ……………….. [1]
Answer:
Pain
Explanation: To overcome grief one has to go through certain stages. After the Last stage, the affected person accepts reality and is relieved from the pain.

(B) “…rejoin the main stream of life” Pick the option in which the meaning of ‘stream’ is NOT the same as it is in the passsge:
(a) After meeting her estranged sister she experienced a stream of emotions inside her.
(b) Since there was no internet, I couldn’t stream the movie.
(c) You can always see a stream flowing in the mountains.
(d) When her dog died she sat with tears streaming down her face. [1]
Answer:
(a) After meeting her estranged sister she experienced a stream of emotions inside her.
Explanation: In the passage stream means flow of feelings. In (b) stream means buffering, (c) stream means flowing water and (d) stream means flowing down of tears. Hence, (a) is correct as here stream means flow of emotions.

(C) Based on your understanding of the passage, choose the option that lists the correct sequence of the phases of grief:
(I) The person has difficulty in activities like eating and sleeping.
(II) The person recollects the old moments and the memory of guilt creeps in.
(III) This leads to the positive side.
(IV) The person is ready to cope with the situation.
(V) A feeling of numbness overtakes.
(a) (IV), (II) (I), (III), (V)
(b) (I), (III), (II), (V), (IV)
(c) (IV), (I); (V), (III), (II)
(d) (V), (I), (II), (IV), (III) [1]
Answer:
(d) (V), (I), (II), (IV), (III)
Explanation: While going through grief firstly, one experiences a feeling of numbness. Then the affected person has difficulty inactivities like eating and sleeping After that, there is recollection of old memories. Then the person is ready to accept reality leading to positive thinking. Hence, (d) is correct.

(D) Choose the option that best captures the central idea of the passage from the following:
(a) “When you are sorrowful, look again”. – Khalil Gibran
(b) “There is no grief like the grief that does not speak”. – Henry Wadsworth Longfellow
(c) “The only cure for grief is action”. – George Henry Lewes
(d) “Tears are the silent language of grief”. – Voltaire [1]
Answer:
(a) “When you are sorrowful, look again”. – Khalil Gibran
Explanation: The central idea of the passage is that grief is inevitable and one has to face it to overcome it. Once you get over it, you realize that life still goes on. Hence, (a) is correct.

(E) The affected person lets go of grief:
(a) with time
(b) when he is distracted from grief
(c) when he forgets the lost one
(d) when he goes on an outing [1]
Answer:
(a) with time
Explanation: The person affected by grief gets over it with time doesn’t subside with distraction or by forgetting the lost one. Going on an outing is also not the solution. Hence, (a) is correct.

(F) According to the passage, ‘grief and ‘mourning’ are associated with death. True/False [1]
Answer:
True

(G) Pick the option showing the CORRECT use of the word ‘overcome’ as used in the passage.
(a) We need to overcome this reluctance to talk openly.
(b) She stopped struggling, overcome by fear.
(c) In order to move ahead in life, one has to overcome all the hurdles.
(d) Nature is hard to be overcome. [1]
Answer:
(c) In order to move ahead in life, one has to overcome all the hurdles.
Explanation: In the passage, overcome means to defeat. In (a) overcome means to get over, (b) overcome means overpower and (d) means succeed in dealing with a problem. In (c) overcome means to defeat all hurdles. Hence, (c) is correct.

(H) Pick out the option that states what DOES NOT happen before the third stage of grief.
(a) acceptance of reality
(b) feeling of numbness
(c) denial
(d) feeling of pain [1]
Answer:
(a) acceptance of reality
Explanation: First there is feeling of numbness. Then, follow denial and feeling of pain. In the last stage there is acceptance of reality. Hence, (a) is correct.

(I) Which people are able to cope with the loss better? [1]
Answer:
People who follow strict and formal mourning rituaLs
Explanation: The people who follow all the formal rituals are able to cope with loss better.

(J) Pick the option that correctly lists the final feelings of the person in grief.
(I) frustrated
(II) remorse
(III) agitated
(IV) acceptance
(V) anguish
(VI) reconciliation
(a) (I) and (IV)
(b) (II) and (V)
(c) (IV) and (VI)
(d) (I) and (III) [1]
Answer:
(c) (IV) and (VI)
Explanation: A person in grief experiences a lot of emotions like pain, sorrow, acceptance and reconciliation. Hence, (c) is correct.

CBSE Sample Papers for Class 12 English Set 11 with Solutions

Question 2.
Read the passage given below: (1 x 10 = 10)
Looking For A New Clarity
(1) The Supreme Court, this past month, provided us with a useful reminder about its worth to our constitutional democracy. Its intervention in the imbroglio over government formation in Karnataka was flawless. The hearings conducted in the early hours of the morning may have been theatrical, but the court’s ultimate decision certainly helped avert a subversion of the Constitution.

Yet, much as its decision here deserves appreciation, we must be careful not to allow any ascription of credit to veil the deeper wounds that afflict it for a litany of problems continues to strike at the court’s independence.

(2) Three of these are especially salient The first involves the rejection by the government of the collegium’s recommendation of KM. Joseph, currently Chief Justice of the Uttarakhand High Court, for elevation to the Supreme Court The second concerns the need for a systemic mechanism to deal with allegations of corruption in the higher judiciary.

The third area of worry concerns the embroiled state of Chief Justice of India, Dipak Misra, his position as the master of the roster, and the critical question of whether such powers ought to be vested in the hands of one individual. Recurring problems:

(3) At first glance, these issues might strike us as unique to the times that we live in, as examples of crises that will eventually pass. But, on closer examination, it becomes clearer that these are, in fact recurring problems left unaddressed far decades. In trying to resolve the issues, therefore, we must ask ourselves how we got here. As AG. Noorani recently wrote in Frontline magazine fCrisis in Judiciary,” May 11, 2018): ‘V/e have not reached the nadir all of a sudden. The decline was long in process.”

(4) In his seminal book, America’s Unwritten Constitution: The Precedents and Principles We Live By, Akhil Reed Amar points to how the written constitution often invites us to heed what’s unwritten, which in turn, he writes, “refers us back in various ways to its written counterpart Like the Chinese symbols yin and yang, America’s written Constitution and America’s unwritten Constitution form two halves of one whole, with each half gesturing toward the other.”
(Source: The Hindu) On the basis of your reading of the above passage, answer the followingquestions:
(A) According to the passage, the Supreme
Court has reminded us of ………………. [1]
Answer:
constitutional democracy

(B) Pick the option that lists statements that are NOT TRUE according to the passage.
(I) The current Chief Justice of the Uttarakhand High Court is K.M. Joshi
(II) The hearings were conducted in the afternoon.
(III) The third area of worry concerns the embroiled state of Chief Justice of India, Dipak Misra.
(IV) The court’s ultimate decision did not help avert a subversion of the Constitution.
(a) (I) and (II)
(b) (III) and (IV)
(c) (II) and (IV)
(d) (I) and (IV) [1]
Answer:
(c) (II) and (IV)
Explanation: The passage does not say that the hearings were conducted in the afternoon as they were conducted in the morning. The court’s ultimate decision helped to avert a subversion of the Constitution. Hence, (c) is correct.

(C) The word ‘veil’, as used in paragraph 1, can be SUBSTITUTED with ………………. [1]
Answer:
‘cover’
Explanation: In paragraph 1, veil means to cover.

(D) The third area of worry concerns
(a) Dipak Misra’s position as the master of the roster.
(b) The rejection by the government of the collegium’s recommendation of K.M. Joseph.
(c) The need for a systemic mechanism.
(d) How to deal with allegations of corruption in the higher judiciary. [1]
Answer:
(a) Dipak Misra’s position as the master of the roster.
Explanation: The third area of worry concerns Dipak Misra’s position as the master of the roster. Hence, (a) is correct.

(E) A.G. Noorani recently wrote what in Frontline magazine? [1]
Answer:
Crisis in Judiciary
Explanation: A.G. Noorani recently wrote in Crisis of Judiciary.

(F) What did A.G. Noorani recently write in Frontline magazine? [1]
(a) “We have not reached the nadir all of a sudden. The decline was short in process.”
(b) “We have not reached the nadir all of a sudden. The decline was long in process.”
(c) “We have reached the nadir all of a sudden. The decline was long in process.”
(d) “We have not reached the nadir all of a sudden. The rise was long in process.” [1]
Answer:
(b) “We have not reached the nadir all of a sudden. The decline was long in process.’’
Explanation: A.G. Noorani recently wrote in Frontline magazine, “We have not reached the nadir all of a sudden. The decline was long in process.” Hence, (b) is correct.

(G) Who is the writer of the book ‘America’s Unwritten Constitution: The Precedents and Principles We Live By’? [1]
Answer:
Akhil Reed Amar
Explanation: The writer of the book is Akhil Reed Amar.

(H) How many salient problems are related to the given issue in the passage? [1]
Answer:
three

(I) Does the following statement agree with the information given in the passage.
The first problem involves the acceptance by the government.
True-if the statment agrees with the information.
False-if the statement contradicts with the information.
Not Given-If there is no information on this. [1]
Answer:
False
Explanation: The first problem involves the rejection by the government.

(J) The second problem concerns the need to deal with what of the following?
(a) Allegations of corruption in the lower judiciary
(b) Allegations of corruption in the higher judiciary
(c) An unsystemic mechanism
(d) None of these [1]
Answer:
(b) allegations of corruption in the higher judiciary
Explanation: The second problem concerns the need to deal with allegations of corruption in the higher judiciary. Hence, (b) is correct.

Section – B
Creative Writing Skills (20 marks)

Question 3.
Answer ANY ONE of the following
Analysis, using appropriate format and fluency, appropriacy of style and tone.
Your school, ‘Woodland College’ is going to have a class on ‘Social Distancing’ keeping in the mind, the pandemic and the precautions to be taken by the students. Draft a notice in not more than 50 words to be sent to the students digitally, inviting students to attend it online. Give all the necessary details. You are Roma/Romi, Head girl/Head boy of the school.
OR
You are Vaishnavi Dutt, a student of class XII, Aligarh Public School, Aligarh. You have found an expensive wrist watch in the school playground. Draft a notice in not more than 50 words for the school notice board. [5]
Answer:
Woodland College
Notice
12th August, 20XX
Online Class on Social Distancing
This is to inform the students that an online class on ‘Social Distancing’ is going to be held on 20th August 20XX. The students will be taught about the importance of the step taken by the government of India and the ways to maintain it along with carrying on the other necessary tasks of life. All of you are required to attend it online through the school web portal.
The students will receive the email regarding the details of the class soon. For any related query, contact the undersigned.
Roma/Romi
Head girl/Head boy
OR
ALIGARH PUBLIC SCHOOL, ALIGARH
NOTICE
27th August, 2OXX
Lost and Found
I have found a silver-colored ‘Casio’ wrist -watch in the school playground during the recess yesterday. The watch is analog with a black dial for males. Owner may please contact the undersigned during the school hours in her respective class.
Vaishnavi Dutt
Class XII-C

Question 4.
Attempt ANY ONE of the following:
Analysis, using appropriate format and fluency, appropriacy of style and tone.
You are Sarit Agarwal of 31-D, Kamala Nagar, Bhopal. You have purchased a new house and want to give a house warming party for the same. Draft an invitation in not more than 50 words.
OR
You have received an invitation from your friend to attend her birthday party. Write a reply in not more than 50 words accepting the invitation and expressing your excitement for the same. [5]
Answer:
Mr. Sarit Agarwal
requests your presence at
the House Warming Party
of his new home
at 9.00 p.m. on 24th September, 20XX
at his residence 31- D, Kamala Nagar,
Bhopal
RSVP
Mr. Sarit Agarwal 973296XXX
OR
25th September, 20XX
Dear Pinky,
Thank you for sending the lovely invitation to your birthday party. I am really excited to attend the party and meet all our friends. On this precious occasion, I want to wish you a healthy and a happy life. May you celebrate many more birthdays ahead.
Thank you once again for the kind invitation.
Best Wishes
XYZ

CBSE Sample Papers for Class 12 English Set 11 with Solutions

Question 5.
Attempt ANY ONE of the following:
Analysis, using appropriate format and fluency, appropriacy of style and tone.
You are Anirudh/ Anuradha. You come across the following advertisement. You consider yourself an ideal candidate for the same. Apply for the job giving all pertinent details in not more than 120-150 words.
Situation Vacant
Wanted a young energetic HR Manager for Gurgaon Office of Uber India Ltd. with degree from a reputed college and minimum three years’ experience. Contact, the Director, Uber, Ltd
OR
There have been increased reporting of cases of domestic violence during the period of nationwide lockdown. This shows a grave scenario that being forced to stay indoors is proving to be harmful for the mental health of many people. Write a letter to the editor of a local daily in not more than 120-150 words and create awareness about this issue. You are Ram/Rama of 12C, Sarita Vihar, Patna. [5]
CBSE Sample Papers for Class 12 English Set 11 with Solutions 1
Answer:
1204, AWHO
Ram Garden
Noida ( UP)
7th April, 20XX
The Director
Uber India Ltd
Gurgaon
Subject: Application for the Post of HR Manager
Madam/Sir
This is with reference to the advertisement published in The Times of India, dated 4th April 20XX, for the post of HR Manager. I wish to apply for the same.
My qualifications are as per your requirement. I have done my MBA in Human Resource from a reputed college and I am presently working with HR Solutions, Faridabad. I am willing to relocate as I am aware that your company is known to provide opportunities for employee growth.
If I am given the opportunity to serve your reputed company, I assure you that I would prove to be an asset. My bio data is attached for your kind perusal.
Yours sincerely
Anirudh/Anuradha
Enclosure : Bio-data

Bio-Data
Anirudh/ Anuradha
Father’s name : Narendra Nath
Date of birth : 02-12-19XX
Contact address : 1204-AWHO, Noida
Email: [email protected]
Educational qualifications :

S. No. Class/ Degree Board/ University Institution Year Percentage
1. Class X CBSE SDA School 19XX 84%
2. Class XII CBSE SDA School 20XX 94%
3. BBA Delhi University Mt Carmel College 20XX 88%
4. MBA Delhi University Lala Lajpat Rai Institute of Management 20XX 80%

Work Experience: HR Manager at HR Solutions from March 2015 to April 2020
References: (i) Mr Rohit Lowe, Addtl Director, HR Solutions 9925XXXXXX
(ii) Ms Asha Khana, HR Manager, Sapna Solutions 7654XXXXXX
Testimonials enclosed herewith
The information given above is true to the best of my knowledge.
Anirudh/Anradha
7th April 20XX
OR
12C,
Sarita Vihar,
Patna
21st May, 20xx
The Editor
The National Times Patna
Subject: Rising cases of domestic violence during lockdown Ma’am/ Sir,
I would like to draw the attention towards the rise in the number of cases of domestic violence during the period of nationwide lockdown through the medium of your prestigious newspaper.

Being locked up at homes is proving to be bothersome for some people. Their behavior is becoming erratic and restless, resulting in domestic violence. The situation is getting worse day by day. For those people who are finding it hard to deal with the current situations, there needs to be some sort of help given. It can be in the form of providing online counseling sessions or airing informative commercials on keeping good mental health on mass media so that they stay away from any such activity.

I request you to publish my letter in your newspaper to create awareness and motivate government authorities to step up and address this issue at the earliest.
Your faithfully
Ram/Rama

Question 6.
Attempt ANY ONE of the following:
Evaluation, using appropriate format and fluency, appropriacy of style and tone.
There is a lot of talk on saving the environment but we are not doing enough as a society. Things will not change overnight and there is a need for the youth to take charge and bring about the much needed change. As Aditya/Aditi, write an article in 120-150 words to be published in the local daily on the need to change our approach towards the environment with the help of the given cues.

  • Nature give us warning and reminders
  • Youth are the leaders of the future
  • Time has come to ensure the protection of the environment

OR
Your school sent you to attend a workshop organised by the Forum for Special Awareness on the use of smart phones by school going children. The workshop also informed the participants on the judicious use of smart phones. Write a report in 120150 words on the workshop to be published in your school magazine. You are Prakhar/ Siya, a student of Class XII, with the help of the given cues

  • Workshop attended by students from all across the city.
  • Students spend too much time surfing worthless sites or interacting on the social media.
  • Use the smart phones judiciously so that it can be a boon and not a bane.

Answer:
The Time for Change Is Now
By Aditi/Aditya
Every day nature gives us warnings and reminders on how our callous and thoughtless attitude is destroying the environment. We take notes, conduct seminars and organise marches and then we go back to our old ways. But now the time has come to step up and get proactive.

The youth must lead by example, since they have long been called ‘the leaders of the future’, and now is the time for them to take charges of the situation. Taking care of the environment must be our duty and the youth must ensure that everyone performs this duty diligently.

The training and conditioning to care for the environment must be instilled in students’ right from the beginning. If this is made a part of the curriculum, where in each child is told that she/he is responsible for a tree, cleanliness of surroundings, preventing pollution, the child grows up with an attitude of nurturing nature.

We need to believe that the change begins with us, so let us reduce the use of air-conditioners, vehicles and, go back to the basics. Time has come to ensure all thoughts of protecting the environment are not just to show but for real. If the children grow up knowing their responsibility towards the environment, they will truly be the torchbearers of change and give the environment the nurturing it needs.
OR
Use the Smart Phone Smartly
By Prakhar/Siya, Class XII E
The workshop was conducted by the Forum for Special Awareness on the use of smart phones by school going children which was a wonderful learning experience. The workshop provided the participants an insight into how smart phones can become our best friends instead of the nuisance everyone thinks them to be.

The workshop was attended by 250 students from all across the city. The chief guest Mr Narayan Murthy told the students, “Smart phones can be your best friends if you know how to use them smartly. A smart phone can provide you all the information you need, provided you like in the right place.” The other speakers agreed that phones are meant to be means of communication, which can be perfect for that role.

The problem usually begins when students spend too much time surfing worthless sites or interacting on the social media. A presentation on how the very same social media sites can be used to gain information and update knowledge was not only informative but also helped the students understand their mistakes.

The speakers informed and guided the students on how they can use the smart phones judiciously so that it can be a boon and not a bane for then. The students went back armed with various tools to help and guide them use the smart phones to their advantage.

Section – C
Literature (40 marks)

Question 7.
Read the extracts given below and attempt ANY ONE of the two given bg answering the questions that follow: (1 x 6 = 6)
When Aunt is dead, her terrified hands will lie
Still ringed with ordeals she was mastered by
The tigers in the panel that she made
Will go on prancing, proud and unafraid.
(A) ‘When Aunt is dead.’ Who is the ‘aunt’ being talked about here?
(a) Poet’s aunt
(b) Aunt Jennifer
(c) Uncle’s wife
(d) Aunt in general [1]
Answer:
(b) Aunt Jennifer
Explanation: The aunt mentioned here is Aunt Jennifer, the protagonist of Adrienne Rich’s poem ‘Aunt Jennifer’s Tigers’. It doesn’t refer to any of the options (a), (c) or (d). Hence, (b) is the right answer.

(B) Why is she “ringed with ordeals”?
(a) As she has been tied with a lot of marriage responsibilities.
(b) As she is burdened with a lot of rings and ornaments on her fingers.
(c) As she wearing a ring that spells ‘ordeals’.
(d) As she is a new bride. [1]
Answer:
(a) As she has been tied with a lot of marriage responsibilities.
Explanation: The ‘ring’ here refers to her wedding band or ring, which has brought with it a host of family responsibilities. She feels so opressed and tied by her marital liabilities that it seems like an ordeal to her. So, the correct option is (a).

(C) Complete the given sentence.
Aunt Jennifer is ……………….. but her tigers are ………………… [1]
Answer:
Coward; brave

(D) The tigers created by Aunt Jennifer reflects her:
(a) inner strength
(b) fantasy
(c) creativity
(d) deepest hidden desire [1]
Answer:
(d) deepest hidden desires
Explanation: The tigers created by Aunt Jennifer are an expression of her hidden desire to free herself from the constraints of her married life. She wants to be bold and fearless like her tigers. Hence, (d) is the right answer.

(E) List the correct rhyme scheme of the given extract. [1]
Answer:
aabb
Explanation: The words ‘lie’ and ‘by’ sounds similar while the words ‘made’ and ‘unafraid’ sounds similar. Hence, (a) is the right answer.

(F) Choose the correct option based on the given statements.
(1) Aunt Jennifer seems to be an suppressed lady.
(2) The tigers that Aunt creates are in contrast to her real personality.
(a) Statement 1 is true but 2 is false.
(b) Statement 2 is true but 1 is false.
(c) Both statements 1 and 2 are true.
(d) Both statements 1 and 2 are false. [1]
Answer:
(c) both statements 1 and 2 are true.
OR
Therefore, on every morrow, are we wreathing
A flowery band to bind us to the earth,
Spite of despondence, of the inhuman dearth
of noble natures, of the gloomy days,
of all the unhealthy and o’er-darkened ways
Made for searching.
(A) According to the poet, what are we doing every morning?
(a) weave a flowery band that bound us to the earth
(b) weave a flowery band that repels us from the earth
(c) weave a flowery band that bound us to life
(d) none of these [1]
Answer:
(a) weave a flowery band that bound us to the earth
Explanation: According to the poet, every morning we weave a flowery band that bound us to the earth. Hence, (a) is correct.

(B) ‘Spite of despondence’ evokes the feeling oft
(a) the loss of friendship
(b) the loss of hope
(c) the loss of love
(d) none of these [1]
Answer:
(b) the loss of hope
Explanation: ‘Spite of despondence’ refers to a mental state when we cannot see anything good about life. When we lose all hopes. Hence, (b) is correct.

(C) List the rhyme scheme of the above extract is: [1]
Answer:
abbcca
Explanation: The sound of the first and last line’s last word is the same, ‘earth’ and ‘dearth’ rhyme with each other and ‘days’ and ‘ways’ are rhyming words. Hence, (a) is correct.

(D) ‘unhealthy and o’er-darkened ways’ refers to
(a) unhygienic lifestyle
(b) dark roads
(c) harmful and illicit methods adopted by human beings
(d) beauty binds us to the earth [1]
Answer:
(c) harmful and illicit methods adopted by human beings
Explanation: The words refer to the harmful and illegal ways adopted by people. Hence, (c) is correct.

(E) List the correct synonym of the word ‘gloomy’. [1]
Answer:
sad

(F) By ‘wreathing’, the poet means ………………….
(a) To destroy
(b) To braid
(c) To pluck
(d) To snatch [1]
Answer:
(b) To braid

CBSE Sample Papers for Class 12 English Set 11 with Solutions

Question 8.
Read the extracts given below and attempt ANY ONE of the two given by answering the questions that follow. (1 x 4 = 4)
The visual scale ranges from the microscopic to the mighty: midges and mites to blue whales and icebergs as big as countries (the largest recorded was the size of Belgium). Days go on and on and on in surreal 24-hour austral summer light, and a ubiquitous silence, interrupted only by the occasional avalanche or calving ice sheet, consecrates the place. It’s an immersion that will force you to place yourself in the context of the earth’s geological history.
(A) Which of the given options means ‘microscopic to the mighty’?
(a) largest to the smallest
(b) smallest to the largest
(c) small to smaller
(d) large to larger [1]
Answer:
(b) smallest to the largest
Explanation: Anything that is ‘microscopic’ means very small while a ‘mighty’ thing is very big. Hence, (b) is the right answer.

(B) Complete the given sentence.
A …………… and ……………… made author think about the Earth’s physicality.
(a) Breakage of iceberg; summer light in the Southern Hemisphere
(b) Breakage of stars; summer light in the Southern Hemisphere
(c) Droughttfoods
(d) Drought; earthquakes [1]
Answer:
(a) Breakage of iceberg; summer light in the Southern Hemisphere

(C) List the correct synonym for the word ‘immersion’. [1]
Answer:
absorption

(D) What does the author mean by Visual ‘ scales’?
(a) a measuring scale
(b) a vision board
(c) cinematic panorama
(d) artificial views [1]
Answer:
(c) cinematic panorama
Explanation: The phrase ‘visual scales’ refers to the cinematic views or panorama of the place. Hence, (c) is the right answer.
OR
“I wondered, Your Excellency,”Sadao murmured. “It was certainly very Ncareless of me,” the General said. “But you understand it was not ” lack of patriotism or dereliction of duty.” He looked anxiously at his doctor. ‘If the matter should come out you would understand that, wouldn’t you?”
‘Certainly, Your Excellency,” Sadao said. He suddenly comprehended that the General was in the palm of his hand and that as a consequence he himself was perfectly safe. “I swear to your loyalty. Excellency,” he said to the old General, “and to your zeal against the enemy.”
(A) Which of the given words refer to ‘dereliction’ as per the given context?
(a) irresponsibility
(b) negligence
(c) tolerance
(d) Performance [1]
Answer:
(b) negligence
Explanation: The term ‘dereliction’ means negligence or carelessness of something. Hence, (b) is the correct answer.

(B) At the end of the conversation with the General, what was the reaction of Sadao? [1]
Answer:
hopefully relaxed
Explanation: Sadao said that he felt like the General was ‘in his palms’. This shows that Sadao was relaxed and hopeful of the enemy being taken away without any issue.

(C) Pick the option that states the correct reason of Sadao being at the stake of General’s understanding.
(a) He saved an American soldier at his home.
(b) He killed the enemy.
(c) He refused to treat the old General
(d) He didn’t want to continue his service. [1]
Answer:
(a) He saved an American soldier at his home.

(D) Select a word that DOES NOT mean the same as ‘zeal’.
(a) enthusiasm
(b) passion
(c) excitement
(d) nervousness [1]
Answer:
(d) nervousness

Question 9.
Read the extracts given below and attempt ANY ONE of the two given by answering the questions that follow. (1 x 6 = 6)
She still has bangles on her wrist, but no light in her eyes. “Ek waqt ser bhar khana bhi nahin khaya.” she says, in a voice drained of joy. She has not enjoyed even one full meal in her entire lifetime-that’s what she has reaped! Her husband, an old man with a flowing beard says, “I know nothing except bangles. All I have done is make a house for the family to live in.” Hearing him one wonders if he has achieved what many have failed in their lifetime. He has a roof over his head!

The cry of not having money to do anything except carry on the business of making bangles, not even enough to eat, rings in every home. The young men echo the lament of the elders. Little has moved with time, it seems in Firozabad, years of mind-numbing toil have killed all initiative and the ability to dream.
(A) What does the phrase ‘She still has bangles on her wrist, but no light in her eyes’ implies about the girl?
(a) That she is married but blind.
(b) That she is a married and happy.
(c) That though she is married, her eyes are devoid of any clue of joy.
(d) That she is a married but lost her natural beauty. [1]
Answer:
(c) That though she is married, her eyes are devoid of any clue of joy.
Explanation: The given phrase means that though the girl is married, her eyes are devoid of happiness. Hence, (c ) is the correct answer.

(B) “Ek waqt ser bhar khana bhi nahin ’ khaya.’ The tone of the speaker is ……………….. [1]
Answer:
disdainment

(C) The phrase ‘The young men echo the lament of their elders’ represents the …………….. between the generations.
(a) acceptance of poverty
(b) similarity of fate
(c) conclusion
(d) chaos [1]
Answer:
(b) similarity of fate

(D) ‘Years of mind-numbing toil have killed all initiative and the ability to dream’. This shows that:
(a) The bangle makers are exhausted yet they are working continuously.
(b) The drudgery of work has destroyed their willingness to improve their lot.
(c) The daily grind has stolen the dreams of them.
(d) They have been working so hard that there’s no time to dream. [1]
Answer:
(b) The drudgery of work has destroyed their willingness to improve their lot.
Explanation: This shows that the drudgery of work has destroyed their willingness to improve their lot. Hence, (b) is the correct answer.

(E) The word that means the same as ‘toil’ is ……………….
Answer:
hardwork

(F) Choose the word that does NOT mean the same as ‘drained’.
(a) weary
(b) emptied
(c) full
(d) used [1]
Answer:
(c) full

OR
From the beginning, however, I had an aversion to the water when I was in it This started when I was three or four years old and father took me to the beach in California. He and I stood together in the surf. I hung on to him, yet the waves knocked me down and swept over me. I was buried in water. My breath was gone. I was frightened. Father laughed, but there was terror in my heart at the overpowering force of the waves. My introduction to the Y.M.CA. swimming pool revived unpleasant memories and stirred childish fears. But in a little while I gathered confidence.
(A) At which place did the incident in childhood of Douglas had taken place? [1]
Answer:
California

(B) Pick the word that means the same as ‘aversion’.
(a) Love
(b) inclination
(c) dislike
(d) influence [1]
Answer:
(c) dislike
Explanation: The word ‘aversion’ means to dislike something or to repel it. Hence, (ii) is the right answer.

(C) The writer decided to learn to swim when he was about:
(a) ten or eleven gears old
(b) fifteen or sixteen gears old
(c) twenty gears old
(d) eighteen gears old [1]
Answer:
(a) ten or eleven years old

(D) His mother warned him against swimming in the Yakima River because ………………. [1]
Answer:
many people had drowned there and lost their lives

(E) The pool’s depth at the deep end was ………………
(a) seven feet
(b) nine feet
(c) sixteen feet
(d) four feet [1]
Answer:
(b) nine feet

(F) The term that matches the meaning of ‘being scared’ from the extract is:
(a) stirred
(b) aversion
(c) frightened
(d) unpleasant [1]
Answer:
(c) frightened

CBSE Sample Papers for Class 12 English Set 11 with Solutions

Question 10.
Attempt ANY FIVE out of the six questions given below, in 40-50 words each. (5 x 2 = 10)
(A) Neruda sags that he does not advocate total inactivitg and death. Whg did he sag so? [2]
(B) Explain the pain that Kamala Das speaks of when she sags she began ‘looking outside the car’. [2]
(C) Explain whg Danng Casey was popular among people in the storg ‘Going Places’. [2]
(D) Subbu was a very generous man but still he had rivals. Comment. [2]
(E) The reader’s sympathy is with the peddler right from the beginning? Is it justified? State with a textual evidence. [2]
(F) What was the reason behind Umberto Eco’s writing a novel? [2]
Answer:
(A) Neruda makes it very clear that his call for keeping quiet should not be mistaken with inactivity and death because life is meant to be lived. Life is a continuous process and one needs to be involved with a positive attitude.

(B) The poet, Kamala Das, is in agony and pain when the realization hits her that her mother was getting old and might die. To distract herself from the fear she begins looking outside the car. The sight of merry children and the lush green trees present a sharp contrast, doing nothing but to make her feel better. It only draws her attention to the energy and life outside which is in direct contrast to her ageing mother.

(C) Danny Casey was a young football player from Ireland. He was popular among the people especially girls in the chapter ‘Going Places’ due to his playing skills, looks and his successful career. Teenage girls were his huge fans.

(D) Subbu was really an improvident man but his habit of pleasing the boss as a way of being successful in his career was disliked by the people in the Gemini Studios. Also, Suubu was a non-critical person and so, he was believed to be a clever person. This made people like office boy get envious of him as he would blame Subbu for the failure of his life at the studios.

(E) The rattrap peddler draws the reader’s sympathy because of his poverty and destitute state. The author’s description of his clothes and appearance like – ”his clothes are in rags, his cheeks are sunken and hunger gleams in his eyes” and his resorting to begging and petty thievery to keep his body and soul together evoke the reader’s sympathy.

(F) It was by an accident that Umberto Eco started writing a novel. As he explained in the interview that one day he was having nothing to do and so, he started writing a novel as a way to satisfy his taste to narrate his stories.

Question 11.
Attempt ANY TWO out of the three questions given below in 40-50 words: (2 x 2 = 4)
(A) When the psychiatrist says that Charley’s desire is an escape, Charley’s wife gets upset with him. Why do you think so? [2]
(B) Sadao was confident that despite his actions of treating a prisoner, nothing would happen to him. Support this statement with a rationale. [2]
(C) Bama’s brother made a huge impression on Bama. Which words of her brother influenced Bama to study hard? [2]
Answer:
(A) When the psychiatrist suggests that Charley’s desire to find the third level is an escape mechanism to get away from the stress and worries of the modern world, Charley’s wife considers it as an insult and a comment on their family life. Sam tries to tell her that contrary to the psychiatrist’s prognosis, Charley is trying to get away from the insecurity, wars, fears and worries of the modern world.

(B) Dr. Sadao informed the General about the prisoner in his house and the General promised that he would take care of him. Being a careless and self-centred person, the General forgot to send assassins to kill the prisoner. Since it was the General’s fault, combined with the fact that the General needed him for the surgery, this knowledge gave Dr. Sadao the power and also the confidence that he was safe and no action would be taken against him.

(C) Bama’s brother told her that they belonged to a family that was known as ‘untouchables’ which means the ones who were low caste. They were looked down upon by the society. And, it can be done only by studying hard, and eradicating this evil of untouchability from the society for people like them.

Question 12.
Attempt ANY ONE of the following questions in 120-150 words.
The storg ‘The Lost Lesson’ talks about the realization regarding the perfect teaching-learning process. Elaborate with reference to the text.
OR
The storg ‘The Rattrap’ focuses on human loneliness and the need to bond with others. Support your rationale with textual evidence. [5]
Answer:
Franz who always considers his books a nuisance and his bag a dead weight gets a new perspective on education and learning. Franz was not fond of his teacher, M Hamel, and always found an excuse to miss his lesson. The day he learns that French would not be taught again, he realises that he had taken his education and learning for granted. M Hamel explains the importance of one’s language. That day Franz is able to understand everything his teacher taught and he realises that the teaching-learning experience is a two¬way process. The student has to be interested and also should have a desire to learn.

The teachers must also be passionate about teaching and also be patient and sincere in his or her intent of teaching and bestowing knowledge. When both parties show similar enthusiasm, the teaching-learning process is perfect.
OR
The Rattrap deals with the issues of human loneliness and the need to bond with others. Not only the peddler but also other characters like the crofter, the ironmaster and Edla emphasize upon this fact. The peddler’s conscience had left him because he had been lonely in his predicament, for a long time.

But Edla’s kindness and hospitality changed him. The crofter, on the other hand, was a lonely fellow whose craving for a company led him to give shelter to a vagabond, and he ended up getting robbed. Even the ironmaster and his daughter suffer from loneliness. They crave for a company on Christmas Eve and were excited when they got the opportunity to serve a guest.

CBSE Sample Papers for Class 12 English Set 11 with Solutions

Question 13.
Attempt ANY ONE of the following questions in 120-150 words.
After Mr. Lamb dies, will Derry get back to his old seclusion or will Mr. Lamb’s brief association with Derry lead to a change in the land of life he leads in the future?
OR
There are moments in life when we have to make hard choices between our roles as private individuals and as citizens with a sense of national loyalty. Discuss with reference to the story ‘The Enemy’. [5]
Answer:
In the play, we have seen the huge impact that Mr. Lamb has created on Derry in a fraction of time. This impact of Mr. Lamb’s on Derry is evident through the scene where Derry’s mother stops him from meeting the former but Derry doesn’t pay heed to what his mother tells him about Mr. Lamb’s reputation in the area. The transformation that Mr. Lamb has successfully brought in Derry from negativity to positivity in life, will grow stronger after Mr. Lamb dies.

It is very unusual for Derry to connect to a person as he is an introvert who doesn’t like to get close to anyone especially due to his embarrassment from his burnt face. But when he meets, Mr. Lamb, he feels alive again. Mr. Lamb’s positive approach towards life and physical impairment is something Derry has just imagined. Derry starts to trust and confide in Mr. Lamb which shows that he is having a change of heart towards people. All these changes hints towards Derry becoming a motivated different person after Mr. Lamb dies.
OR
Life has many facets. We live it by maintaining a delicate balance between the various facets it offers to us as part of our existence as an individual in a society. Living for our own self, family, profession and country are just a few of them. However at times, it becomes difficult to maintain this balance, and one gets overpowered by confusion and dilemmas.

The story about Dr. Sadao, Hana and the war prisoner exemplified that. On finding a wounded war prisoner washed ashore, Dr Sadao and his wife were unable to decide what to do. They were confused whether they should save and treated the injured man, leave him to die or inform the army. Eventually, he struck the balance by deciding to save his life before handing him over to the army.

He and his wife sympathetically treated him but secretly hid a war prisoner in their home going against the rule of the law and subdued self conscience. In a bid to get rid of that burden, he revealed the fact to the General who promised to get the prisoner killed through assassins. His inner conscience did not allow him to get the war prisoner killed and he helped the enemy soldier flee to safety.

CBSE Sample Papers for Class 12 Physical Education Term 2 Set 2 with Solutions

CBSE Sample Papers for Class 12 Physical Education Term 2 Set 2 with Solutions

Students can access the CBSE Sample Papers for Class 12 Physical Education with Solutions and marking scheme Term 2 Set 2 will help students in understanding the difficulty level of the exam.

CBSE Sample Papers for Class 12 Physical Education Term 2 Set 2 with Solutions

General Instructions :

  • There are three sections in the Question paper namely Section A, Section B and Section C.
  • Section A consists of 9 questions amongst which 7 questions have to be attempted. Each question carries 2 marks and should have 30-50 words.
  • Section B consists of 5 questions amongst which 3 questions have to be attempted. Each question carries 3 marks and should have 80-100 words.
  • Section C consists of 4 questions amongst which 3 questions have to be attempted. Each question carries 4 marks and should have 100-150 words.

Time : 2 Hrs.
Max. Marks : 35

Section A

Question 1.
What is the main physiological cause of Asthma?
Answer:
The physiological cause of asthma is narrowing of airway passage and a subsequent interference with airflow.

Question 2.
Define acceleration runs.
Answer:
Acceleration runs is a special kind of training in which running speed is gradually increased from jogging to striding and finally to sprinting at maximum speed. Each component is about 50 meter long. Its progressive nature reduces the risk of muscles injuries.

Question 3.
Define sports psychology.
Answer:
Sports psychology is the study of how psychology influences sports, athletic performance, exercise and physical activity. Some sports psychologists work with professional athletes and coaches to improve performance and increase motivation.

Question 4.
What is Attention Deficit or Hyperactivity Disorder (ADHD)?
Answer:
It is a brain disorder that includes inattentiveness, hyperactivity and impulsiveness. It is a medical condition that affects how well can someone sit still, focus.and pay attention.

Question 5.
What do you mean by soft tissue injuries?
Answer:
Soft-tissue injury includes damage of muscles, ligaments, and tendons throughout the body. The result can be pain, swelling, bruising, and damage. Soft-tissue injuries are classified as Contusions, Sprains, Tendonitis, Bursitis, Stress injuries, Strains.

Question 6.
What is coordinative ability?
Answer:
Abilities of an individual which assist him to do a variety of interconnected activities correctly and competently are called coordinative abilities.

Question 7.
What are the symptoms of ODD?
Answer:
Symptoms of ODD are as follows:

  • Rude, aggressive and non-cooperative.
  • Difficulty making friends.
  • Often and easily loses temper.
  • Is frequently touchy and easily annoyed by others.
  • Is often angry and argumentative.
  • Often argues with adults or people in authority.
  • Often actively refuses to comply with adults’ requests or rules.

Question 8.
What kind of sports injury can be termed as ‘Abrasion”?
Answer:
Abrasion is injury on the surface of the skin. In this injury skin is scrapped or rubbed by friction. It causes severe pain and sometimes bleeding from the affected part. Abrasions are very common sports injuries caused by fall on hard surfaces.

Question 9.
Name the type of Disorders.
Answer:
Types of disorders are:

  • Attention Deficit Hyperactivity Disorder (ADHD)
  • Sensory Processing Disorder (SPD)
  • Autism Spectrum Disorder (ASD)
  • Oppositional Defiant Disorder (ODD)
  • Obsessive-Compulsive Disorder (OCD)

Section B

Question 10.
What is the role of a spectators in creating a positive sports environment? Explain.
Answer:
Positive sports environment is the conditions and circumstances which are favorable and beneficial for the sportspersons who perform sports activities. Behaviour and attitude of the spectators towards coaches, players and officials should be positive. They should not pass any negative comment towards players, referee, umpires and any other game officials. They should not indulge themselves in any type of violence. They should try to motivate the players so that they may put up better performance. In this way the spectators can play a vital role for creating a positive sports environment.

Question 11.
What are the various causes of ADHD?
Answer:
The various causes of ADHD are as follows:

  1. Genetic Factors: It is not a disorder that passed socially. Studies show that parents, siblings, and children of people with ADHD may be up to five times more likely to have the disorder than the people who are not related to someone with ADHD.
  2. Brain Injuries: When a baby’s brain is damaged before or after birth this could make the baby v more likely to develop ADHD later on.
  3. Low Birth Weight: It is observed that children with low birth weight are more likely to develop ADHD.
  4. Trauma and brain diseases: Trauma during birth and brain diseases may lead to developing ADHD.
  5. Diet: There are a number of evidence which shows that taking a particular type of food or food additives plays a significant role in causing ADHD.

Question 12.
Discuss the preventive measure of sports injuries.
Answer:
“Sports injuries” are the type of injuries that occur during participating in sports or competitions, training sessions or sports activities. One of the important objectives of sports medicine is preventing injuries. It also prevents other physical, mental, social and financial harm accompanying sports injuries.

General techniques that can prevent sports injuries are:

  1. Warm-up and Cool-down: A well-structured warm-up and cool-down is necessary to increase blood and nutrient flow and concentration. Also it helps in relaxation, improved flexibility and recovery of muscles.
  2. Planning a Session: Careful planning of training and rehabilitation sessions allows gradual specific adaptations. It reduces the damage to the tissues as a result of training.
  3. Using Protective Equipment: The use of protective equipment like proper footwear, helmets, goggles, gum shield, shin pads and gloves prevent many sports injuries.
  4. Psychological Training: Some form of mental skills training and practice could reduce injuries by reducing anxiety and improving concentration.
  5. Adherence to the Rules: If all performers are aware of and adhere to the rules and laws of the particular sport, then injuries can be reduced to great extent.

Question 13.
What is the role of Yoga in preventing lifestyle diseases?
Answer:
Yoga places a great importance on a proper and healthy lifestyle whose main components are healthy activities, healthy relationships, healthy thoughts, healthy food, and healthy recreation. The holistic art and science of yoga is the best lifestyle ever designed and is effective in managing prevalent lifestyle disorders such as diabetes and hypertension.

There are various lifestyle diseases like Obesity, Diabetes, Asthma, Hypertension and Backache. Through regular participation in yoga:

  • Bones and joints become strong.
  • Muscles become stronger and more flexible.
  • Circulation of blood becomes normal.
  • Respiratory organs become efficient.
  • The efficiency of the digestive system increases.
  • Better neuro muscular coordination.
  • Strengthens the immune system.

Question 14.
Explain any three principles of training in brief.
Answer:
Three principles of training are:

  1. Principle of Overload: The overload principle is a basic sports fitness training concept. It means that in order to improve, athletes must continually work harder as their bodies adjust to existing workouts. Overloading also plays a role in skill learning.
  2. Principle of Specificity: The principle of specificity states that the more specific a training activity ” is to a given sport (muscle group, work load, velocity and pattern of movement, body posture, and range of motion) the more it will contribute to increasing performance in that sport.
  3. Principle of Individualization: This could also be called the snowflake principle, since it highlights that no two climbers or their optimal conditioning programs are the same. The best training program for a person will target his/ her specific weaknesses, address past or present injuries, provide sufficient time for recovery, and be structured to provide the greatest output for the available training input.

Section C

Question 15.
What is personality? Explain its different dimensions.
Answer:

Personality is a combination of two words i.e., PERSONAL + IDENTITY = Personality.

Thus, any variable or component that makes a person unique, different from all other constitutes a part of one’s Holistic Personality.

Holistic personality means all round development of:

  1. BODY
  2. MIND (Knowledge, communication and emotion)
  3. SOUL

There are 11 domains of personality :

  • R = Regularity
  • S = Safety
  • S = Self-Responsibility
  • M = Mental Activity
  • O = Occupational Activities
  • V = Vital
  • E = Environmental
  • S = Social
  • P = Physical
  • E = Emotional
  • S = Spiritual
  • T = Temporal (Physical Growth)

Personality Dimensions: The five-factor model is comprised of five personality dimensions (OCEAN): Openness to experience, Conscientiousness, Extraversion, Agreeableness and Neuroticism. The five dimensions are held to be a complete description of personality.

Question 16.
Explain any two methods for speed development.
Answer:
Speed means the velocity with which an individual can execute his movements. In other words, it means the capacity of moving a body part or the whole body with the greatest possible velocity. For example, the movements of a smasher’s arm in volleyball, has the maximum speed or velocity at the time of smashing the volleyball. The following methods are usually adopted for the development of speed in sprinting events.

(i) Acceleration Runs: These are usually adopted to develop speed, especially in attaining maximum speed from stationary position. It should be kept in mind that the technique of any event should be learnt in the beginning. Only then, we should switch over to acceleration runs.

(ii) Pace Races: It means, running the whole distance of a race at a constant speed. In pace races, an athlete runs with uniform speed. Generally 800 meters and above races are included in pace races.

Question 17.
What is movement speed? Explain the methods to develop speed endurance.
Answer:
Movement speed is the time taken between the initiation of movement and the completion of the movement. It depends upon techniques, explosive strength, flexibility and coordinative abilities. It plays a vital role in boxing, gymnastics, swimming; throws jumps etc. where the minimum time is taken to complete the movement.

To develop the speed endurance more work has to be done on pace races because pace races means running the whole distance at a constant speed. Generally, 800 meters and above races are included in pace races. As a matter of fact, an athlete can run a distance of 300 meters at full speed but, in longer races such as 800 meters or above races he must conserve his energy by reducing his speed. For example, if there is a runner of 800 meters race his best time is 1 minute 40 seconds, so, he should nm first 400 meters in 49 seconds and next 400 meters in 51 seconds.

Question 18.
“Involvement in physical activities for longer period of time with moderate intensity can improve the quality of life.” Justify your answer.
Answer:
Involvement in physical activities for longer period of time with moderate intensity can improve the quality of life in the following ways:

(i) Exercise Helps in Healthy Growth and Development: Exercise is an important part of keeping children healthy. Encouraging healthy lifestyles in children and adolescents is important when they grow older. Participating in organised sports and games is not only of great fun but is very essential for healthy growth and development.

(ii) Exercises Improve Self-esteem: Exercise is necessary for the physical and mental health. Self-esteem can play a great role in how children feel about themselves and also how much they enjoy things or worry about things. Exercise reduces depressive symptoms and improves self-esteem in children.

(iii) Enhances Flexibility: The stiffness ofoints decreases due to exercising, in a way improving the flexibility. The elasticity of tendons, ligaments andoint capsules improves due to regular exercise.

(iv) Lessens Stress and Tension: Regular exercise has a distinctive capability to slow down the depression process by reducing stress and tension. Actually, regular exercise lessens the levels of body’s stress hormones like adrenaline and cortisol. The body’s natural painkillers and mood elevators i.e., the endorphins are produced due to regular exercise. These benefits of work out facilitate in delaying the process of ageing.

(v) Connect with Others: Spend time with positive people who enhance your life by instilling in you with positive thoughts. A strong support system will buffer you from the negative effects of stress.

(vi) Keep Your Sense of Humour: This includes the ability to laugh oneself. The act of laughing helps the body to fight stress in a number of ways.

CBSE Sample Papers for Class 11 Hindi Set 4 with Solutions

CBSE Sample Papers for Class 11 Hindi Set 4 with Solutions

Students must start practicing the questions from CBSE Sample Papers for Class 11 Hindi with Solutions Set 4 are designed as per the revised syllabus.

CBSE Sample Papers for Class 11 Hindi Set 4 with Solutions

समय :3 घण्टे
पूर्णाक: 80

सामान्य निर्देश :

  1. प्रश्न-पत्र दो खण्डों में विभाजित किया गया है- ‘अ’ और ‘ब’।
  2. खंड ‘अ’ में 45 वस्तुपरक प्रश्न पूछे जाएँगें, जिनमें से केवल 40 प्रश्नों के ही उत्तर देने होंगे।
  3. खंड ‘ब’ में वर्णनात्मक प्रश्न पूछे गए जाएँगें। प्रश्नों में उचित आन्तरिक विकल्प दिए जाएंगे।
  4. उत्तर लिखते समय प्रश्न का क्रमांक अवश्य लिखें।
  5. एक प्रश्न के सभी भाग एक साथ हल करें।
  6. उत्तर स्पष्ट एवं तर्कसंगत हों।

रखण्ड’अ’ : अपठित बोध

I. अपठित बोध- (15 अंक)

(अ) अपठित गद्यांश

निम्नलिखित गद्यांश को ध्यानपूर्वक पढ़कर नीचे दिए गए प्रश्नों के उत्तर दीजिए- (1 × 10 = 10)

इस संसार में कुछ व्यक्ति भाग्यवादी होते हैं और कुछ केवल अपने पुरुषार्थ पर भरोसा रखते हैं। प्रायः ऐसा देखा जाता है कि भाग्यवादी व्यक्ति ईश्वरीय इच्छा को सर्वोपरि मानते हैं और अपने प्रयत्नों को गौण मान बैठते हैं। वे विधाता का ही दूसरा नाम भाग्य को मान लेते हैं। भाग्यवादी कभी-कभी अकर्मण्यता की स्थिति में भी आ जाते हैं। उनका कथन होता है कि हम कुछ नहीं कर सकते सब कुछ ईश्वर के अधीन है। हमें उसी प्रकार परिणाम भुगतना पड़ेगा जैसा भगवान चाहेगा। भाग्योदय शब्द में भाग्य प्रधान है। एक अन्य शब्द है – सूर्योदय ।

हम जानते हैं कि उदय सूरज का नहीं होता सूरज तो अपनी जगह पर रहता है, चलती घूमती तो धरती ही है। फिर भी सूर्योदय हमें बहुत शुभ और सार्थक मालूम होता है। भाग्य भी इसी प्रकार है । हमारा मुख सही भाग्य की तरफ हो जाए तो इसे भाग्योदय ही मानना चाहिए। पुरुषार्थी व्यक्ति अपने परिश्रम के बल पर कार्य सिद्ध कर लेना चाहता है । पुरुषार्थ वह है जो पुरुष को सप्रयास रखे । पुरुष का अर्थ पशु से भिन्न है। बल – विक्रम तो पशु में अधिक होता है, लेकिन पुरुषार्थ पशु चेष्टा के अर्थ से अधिक भिन्न और श्रेष्ठ है । वासना से पीड़ित होकर पशु में अधिक पराक्रम देखा जाता है, किन्तु यह पुरुष से ही संभव है कि वह आत्मविसर्जन में पराक्रम दिखाए। पुरुषार्थ व्यक्ति को क्रियाशील रखता है। जबकि अकर्मण्य व्यक्ति ही भाग्य के भरोसे बैठता है। हमें भाग्य और पुरुषार्थ का मेल साधना है । यदि सही दिशा में बढ़ेंगे तो सफलता अवश्य हमारे कदम चूमेगी ।

1. ‘भाग्योदय’ शब्द में क्या प्रधान होता है ?
(क) उदय
(ख) ईश्वर
(ग) भाग्य
(घ) किस्मत
उत्तर:
(ग) भाग्य

2. ईश्वरीय इच्छा को कौन सर्वोपरि मानता है?
(क) भाग्यवादी
(ग) आलसी
(ख) कर्मणीय
(घ) गरीब
उत्तर:
(क) भाग्यवादी

CBSE Sample Papers for Class 11 Hindi Set 4 with Solutions

3. कौन उदय न होकर अपनी जगह पर रहता है ?
(क) भाग्य
(ख) धन
(ग) सूरज
(घ) इनमें से कोई नहीं
उत्तर:
(ग) सूरज

व्याख्या – सूरज उदय न होकर भी अपनी जगह पर रहता है।

4. पुरुषार्थी व्यक्ति किस आधार पर अपना कार्य सिद्ध करता है?
(क) परिश्रम
(ख) शक्ति
(ग) झगड़े
(घ) डराकर
उत्तर:
(क) परिश्रम

व्याख्या – इस संसार में कुछ व्यक्ति भाग्यवादी होते हैं और कुछ केवल अपने पुरूषार्थ पर भरोसा रखते हैं। पुरुषार्थी व्यक्ति परिश्रम के आधार पर अपना कार्य सिद्ध करता है।

5. पुरुष को कब पराक्रम दिखाना चाहिए? 1
(क) कार्य के समय
(ख) पढ़ाई में
(ग) आत्मविसर्जन में
(घ) खेल में
उत्तर:
(ग) आत्मविसर्जन में

6. पुरुषार्थ व्यक्ति को कैसे रखता है? 1
(क) नम्र
(ख) समान
(ग) आलसी
(घ) क्रियाशील
उत्तर:
(घ) क्रियाशील

7. भाग्य के भरोसे कौन बैठता है? 1
(क) अकर्मणीय
(ख) निकम्मा
(ग) मेहनती
(घ) श्रमिक
उत्तर:
(क) अकर्मणीय

CBSE Sample Papers for Class 11 Hindi Set 4 with Solutions

8. सही दिशा में बढ़ने के लिए किस की आवश्यकता है? 1
(क) साधना की
(ख) अराधना की
(ग) कामना की
(घ) निर्देश की
उत्तर:
(क) साधना की

व्याख्या – सही दिशा में बढ़ने के लिए साधनों की ज़रूरत है।

9. पुरुषार्थ का संधि-विच्छेद कीजिए- 1
(क) पुरुषा + अर्थ
(ख) पुरुष + आर्थ
(ग) पुरुष + अर्थ
(घ) ये सभी
उत्तर:
(ग) पुरुष + अर्थ

10. सफलता में प्रत्यय लगाइए- 1
(क) फलता
(ख) स
(ग) फालता
(घ) ता
उत्तर:
(घ) ता

(ब) अपठित पद्यांश-

निम्नलिखित पद्यांशों में से किसी एक पद्यांश से संबंधित प्रश्नों के उत्तर दीजिए- (1 × 5 = 5)

जिसमें स्वदेश का मान भरा
आज़ादी का अभिमान भरा
जो निर्भय पथ पर बढ़ आए
जो महाप्रलय में मुस्काए
जो अंतिम दम तक रहे डटे
दे दिए प्राण, पर नहीं हटे
जो देश राष्ट्र की वेदी पर
देकर मस्तक हो गए अमर
ये रक्त-तिलक – भारत – ललाट !
उनको मेरा पहला प्रणाम !
फिर वे जो आँधी बन भीषण
कर रहे आज दुश्मन से रण
बाणों के पवि संधान बने
जो ज्वालामुख-हिमवान बने
हैं टूट रहे रिपु के गढ़ पर
बाधाओं के पर्वत चढ़कर
जो न्याय-नीति को अर्पित हैं
भारत के लिए समर्पित हैं
कीर्तित जिससे यह धरा धाम
उन वीरों को मेरा प्रणाम ।
श्रद्धानत कवि का नमस्कार
दुर्लभ है छंद- प्रसून हार
इसको बस वे ही पाते हैं
जो चढ़े काल पर आते हैं
हुकृति से विश्व काँपते हैं
पर्वत का दिल दहलाते हैं
रण में त्रिपुरांतक बने सर्व
कर ले जो रिपु का गर्व खर्व
जो अग्नि-पुत्र, त्यागी, अकाम
उनको अर्पित मेरा प्रणाम !

1. कवि ने वीरों को किसकी उपमा दी है ? 1
(क) भारतमाता के ललाट के तिलक की
(ख) भारतमाता के चरणों की रज़ की
(ग) भारतमाता के उन्नत शिखर की
(घ) उपर्युक्त सभी
उत्तर:
(क) भारतमाता के ललाट के तिलक की

2. पद्यांश को उपयुक्त शीर्षक दीजिए। 1
उत्तर:
वीरों को प्रणाम ।

CBSE Sample Papers for Class 11 Hindi Set 4 with Solutions

3. कवि किन वीरों के प्रति श्रद्धा सुमन अर्पित करते हैं? 1
उत्तर:
कवि उन वीरों के प्रति श्रद्धा सुमन अर्पित करते हैं जो न्याय के पथ पर चलते हैं, जो देश के प्रति समर्पण का भाव रखते हैं तथा जिनके कारण देश को कीर्ति प्राप्त होती है।

4. वीर की हुंकृति से कौन काँप जाता है? 1
(क) देशद्रोही
(ख) विश्व
(ग) रिपु
(घ) आतंकवादी
उत्तर:
(ख) विश्व

5. कवि कैसे वीरों को अपना प्रणाम अर्पित करता है? 1
उत्तर:
कवि उन वीरों को अपना प्रणाम अर्पित करता है जो युद्ध भूमि में दुश्मन के गर्व को चूर करते हैं। जो निष्काम भाव से देश की सेवा में अपना सब कुछ न्यौछावर कर देते हैं।

अथवा

ब्रह्मा से कुछ लिखा भाग्य में
मनुज नहीं लाया है,
अपना सुख उसने अपने
भुजबल से ही पाया है।
प्रकृति नहीं डर कर झुकती है
कभी भाग्य के बल से,
सदा हारती वह मनुष्य के
उद्यम से, श्रम जल से ।
ब्रह्मा का अभिलेख पढ़ा-
करते निरुद्यमी प्राणी,
धोते वीर कु-अंक भाल के
बहा ध्रुवों से पानी।
भाग्यवाद आवरण पाप का
और शस्त्र शोषण का,
जिससे रखता दबा एक जन
भाग दूसरे जन का ।

1. मनुज किससे कुछ लिखा कर नहीं लाया ? 1
(क) भाग्य
(ख) ब्रह्मा
(ग) विष्णु
(घ) शिव
उत्तर:
(ख) ब्रह्मा

2. प्रकृति मनुष्य के आगे कब झुकती है? 1
उत्तर:
प्रकृति मानव के आगे तब झुकती है जब वह पसीना बहाकर कार्य करता है।

3. जीवन को भाग्य का खेल कौन मानता है? 1
(क) ईमानदार
(ख) कामचोर
(ग) परिश्रमी
(घ) वीर
उत्तर:
(ख) कामचोर

CBSE Sample Papers for Class 11 Hindi Set 4 with Solutions

4. ‘भाग्यवाद आवरण पाप का, और शस्त्र शोषण का ‘ पंक्ति का भाव स्पष्ट कीजिए । 1
उत्तर:
इस पंक्ति का भाव है कि भाग्य के नाम पर एक सम्पन्न व्यक्ति दूसरे गरीब तथा निर्बल का शोषण करता है। गरीब उसे अपना भाग्य मान लेता है।

5. पद्यांश का उपयुक्त शीर्षक दीजिए । 1
उत्तर:
(ख) परिश्रम का फल ।

II. पाठ्यपुस्तक अभिव्यक्ति और माध्यम की इकाई एक से पाठ संख्या 1 तथा 2 पर आधारित बहुविकल्पात्मक प्रश्न । (1 × 5 = 5)

निम्नलिखित प्रश्नों के उत्तर के लिए उचित विकल्प का चयन कीजिए ।

1. दृश्यों का किस माध्यम से अधिक महत्त्व होता है ?
(क) समाचार पत्र
(ख) रेडियो
(ग) टेलीविज़न
(घ) इंटरनेट ।
उत्तर:
(ग) टेलीविज़न

व्याख्या – संचार के माध्यम में दृश्यों का महत्त्व टेलीविज़न में अधिक होता है क्योंकि टेलीविज़न ऐसा माध्यम है, जो घर-घर उपलब्ध होता है और उसमें आवाज़ के साथ-साथ चित्रों को भी देखा जाता है।

2. उल्टा पिरामिड शैली में समाचार कितने भागों में बाँटा जाता है? 1
(क) तीन
(ग) पाँच
(ख) चार
(घ) दो
उत्तर:
(क) तीन

व्याख्या – उल्टा पिरामिड शैली में तीन हिस्से होते हैं-
(1) इंट्रो / लीड जिसे हिन्दी में मुखड़ा कहा जाता है।
(2) बॉडी – इसमें समाचार के विस्तृत ब्यौरे को घटते हुए क्रम में लिखा जाता है।
(3) समापन

3. सर्वाधिक खर्चीला जनसंचार माध्यम कौन-सा है ? 1
(क) रेडियो
(ख) टेलीविज़न
(ग) समाचार पत्र
(घ) इंटरनेट
उत्तर:
(घ) इंटरनेट

CBSE Sample Papers for Class 11 Hindi Set 4 with Solutions

4. भारत का पहला समाचार-पत्र है- 1
(क) बंगदूत
(ख) सरस्वती
(ग) उदन्त मार्तंड
(घ) इनमें से कोई नहीं
उत्तर:
(ख) उदन्त मार्तंड

5. ‘ऑल इण्डिया रेडियो’ की विधिवत् रूप से स्थापना कब हुई? 1
(क) 1936
(ख) 1948
(ग) 1952
(घ) 1964
उत्तर:
(ग) 1936

व्याख्या – सन् 1936 में ‘ऑल इण्डिया’ की स्थापना विधिवत् रूप से की गई।

III. पाठ्यपुस्तक आरोह भाग-1 (10 अंक)

(अ) निम्नलिखित काव्यांश को ध्यानपूर्वक पढ़कर दिए गए प्रश्नों के लिए उचित विकल्प का चयन कीजिए- (1 × 5 = 5)

हे सजीले हरे सावन,
हे कि मेरे पुण्य पावन,
तुम बरस लो वे न बरसें,
पाँचवें को वे न तरसें
मज़े में हूँ सही है,
घर नहीं हूँ बस यही है,
किन्तु यह बस बड़ा बस है,
इसी बस से सब विरस है,

1. कवि को कब जेल में बंद किया ? 1
(क) 1946
(ख) 1942
(ग) 1947
(घ) 1943
उत्तर:
(ख) 1942

2. कवि जल बरसाने के लिए किससे प्रार्थना करता है? 1
(क) सावन
(ख) माँ
(ग) पिता
(घ) चाचा
उत्तर:
(क) सावन

3. सावन को दूत बनाकर संदेश देना कैसी परम्परा है ? 1
(क) प्राचीन
(ख) नवीन
(ग) विकल्प (क) और (ख)
(घ) इनमें से कोई नहीं
उत्तर:
(क) प्राचीन

CBSE Sample Papers for Class 11 Hindi Set 4 with Solutions

4. ‘पुण्य पावन’ में कौन-सा अलंकार है ? 1
(क) उपमा
(ख) अनुप्रास
(ग) रूपक
(घ) अतिश्योक्ति
उत्तर:
(ख) अनुप्रास

5. पुण्य का अर्थ है- 1
(क) पवित्र
(ख) शुद्ध
(ग) शुभ
(घ) सभी
उत्तर:
(घ) सभी

(ब) निम्नलिखित गद्यांश को ध्यानपूर्वक पढ़कर दिए गए प्रश्नों के लिए उचित विकल्प का चयन कीजिए- (1 × 5 = 5)

पंडित अलोपीदीन ने हँसकर कहा- हम सरकारी हुक्म को नहीं जानते और न सरकार को । हमारे सरकार तो आप ही हैं। हमारा और आपका तो घर का मामला है, हम कभी आपसे बाहर हो सकते हैं? आपने व्यर्थ का कष्ट उठाया। यह हो नहीं सकता कि इधर से जाएँ और इस घाट के देवता को भेंट न चढ़ावें। मैं तो आपकी सेवा में स्वयं ही आ रहा था । वंशीधर पर ऐश्वर्य की मोहिनी वंशी का कुछ प्रभाव न पड़ा। ईमानदारी की नई उमंग थी कड़क कर बोले-हम उन नमक हरामों में नहीं हैं जो कौड़ियों पर अपना ईमान बेचते फिरते हैं। आप इस समय हिरासत में हैं। आपका कायदे के अनुसार चालान होगा। बस, मुझे अधिक बातों की फुरसत नहीं है। जमादार बदलू सिंह ! तुम इन्हें हिरासत में ले चलो, मैं हुक्म देता हूँ ।

1. अलोपीदीन कैसा व्यापारी है? 1
(क) सज्जन
(ख) भ्रष्ट
(ग) विकल्प (क) और (ख)
(घ) इनमें से कोई नहीं
उत्तर:
(ख) भ्रष्ट

2. अलोपीदीन बड़े अफ़सरों से सम्बन्ध बनाकर कैसे सामान को इधर से उधर करता ? 1
(क) वैध
(ख) अवैध
(ग) विकल्प (क) और (ख)
(घ) इनमें से कोई नहीं
उत्तर:
(ख) अवैध

3. अलोपीदीन, वंशीधर को क्या कहकर पुकारता था ? 1
(क) वंशी
(ख) सरकार
(ग) मित्र
(घ) प्रभु
उत्तर:
(ख) सरकार

4. वंशीधर कैसा दरोगा था ? 1
(क) सत्यनिष्ठ
(ख) ईमानदार
(ग) विकल्प (क) और (ख)
(घ) इनमें से कोई नहीं
उत्तर:
(ग) विकल्प (क) और (ख)

CBSE Sample Papers for Class 11 Hindi Set 4 with Solutions

5. ‘घाट के देवता को भेंट चढ़ाना’ से क्या आशय है? 1
(क) प्रेम देना
(ख) दया दिखाना
(ग) रिश्वत देना
(घ) बलिदान देना
उत्तर:
(ग) रिश्वत देना

IV. पूरक पाठ्य पुस्तक वितान भाग – 1 (10 अंक)

निम्नलिखित प्रश्नों को ध्यानपूर्वक पढ़कर दिए गए प्रश्नों के लिए उचित विकल्प का चयन कीजिए- (1 × 10 = 10)

1. मंदिरों और महलों में विकसित कलाएँ कौन-सा स्वरूप ग्रहण करती हैं? 1
(क) शास्त्रीय
(ख) पाश्चात्य
(ग) विकल्प (क) और (ख)
(घ) इनमें से कोई नहीं
उत्तर:
(क) शास्त्रीय

2. मध्यकाल में किन कलाओं को शासकों का संरक्षण मिला ? 1
(क) साहित्य
(ख) चित्र
(ग) संगीत
(घ) ये सभी
उत्तर:
(घ) ये सभी

3. किसने संगीत, नृत्य – अभिनय कलाओं को एक शास्त्रीय कला का स्वरूप दिया ? 1
(क) प्रकृति
(ख) मनुष्य
(ग) शास्त्र
(घ) इनमें से कोई नहीं
उत्तर:
(ग) शास्त्र

4. चित्रकारी किस काल से हमारे जीवन का अभिन्न अंग रही है? 1
(क) वीरगाथा काल
(ख) मध्यकाल
(ग) प्राचीनकाल
(घ) आधुनिक काल
उत्तर:
(ग) प्राचीनकाल

CBSE Sample Papers for Class 11 Hindi Set 4 with Solutions

5. सबसे प्राचीन चित्रों के नमूने किन चित्रों को माना है? 1
(क) ऐतिहासिक
(ख) हाइपर
(ग) तकनीकी
(घ) शैल
उत्तर:
(घ) शैल

6. भीमबेटका की गुफ़ाएँ कहाँ हैं? 1
(क) उत्तराखण्ड
(ख) राजस्थान
(ग) मध्य प्रदेश
(घ) उत्तर प्रदेश
उत्तर:
(ग) मध्य प्रदेश

7. चौथी से छठी सदी के बीच किस साम्राज्य कलाओं के लिए स्वर्ण युग कहलाता है? 1
(क) मौर्य
(ख) गुप्त
(ग) विकल्प (क) और (ख)
(घ) इनमें से कोई नहीं
उत्तर:
(ख) गुप्त

8. अजंता की दीवारों पर बने चित्रों को किसने बनाया है? 1
(क) प्रकृति ने
(ख) शासकों ने
(ग) बौद्ध भिक्षु ने
(घ) जैन भिक्षु ने
उत्तर:
(ग) बौद्ध भिक्षु ने

9. किस सदी में चट्टानों को काटकर एलोरा की गुफाएँ तैयार की गईं ? 1
(क) पांचवीं – छठीं
(ख) दूसरी-तीसरी
(ग) आठवीं-नौवीं
(घ) सातवीं-आठवीं
उत्तर:
(ख) दूसरी-तीसरी

10. एलोरा की गुफ़ाओं के बीच किसका विशाल मंदिर है ?
(क) कैलाश
(ख) कृष्ण
(ग) गणेश
(घ) सरस्वती
उत्तर:
(ग) गणेश

CBSE Sample Papers for Class 11 Hindi Set 4 with Solutions

खण्ड ‘ब’ : वर्णनात्मक प्रश्न

V. पाठ्य-पुस्तक अभिव्यक्ति और माध्यम से सृजनात्मक लेखन और व्यावहारिक लेखन । (20 अंक)

1. निम्नलिखित चार अप्रत्याशित विषयों में से किसी एक विषय पर रचनात्मक लेखन कीजिए – ( लगभग 120 शब्दों में) (5 × 1 = 5)

(क) योग के माध्यम से हम शरीर तथा मन दोनों को स्वस्थ कर सकते हैं, जीवन में योग की अनिवार्यता तथा उससे मिलने वाले लाभों का वर्णन करते हुए अपने विचार लिखिए । (5)
उत्तर:
सच ही कहा गया है- पहला सुख निरोगी काया । धर्मग्रन्थों में कहा गया है- “ कुर्वन्नेवेह कर्माणि जिजीविषेत् शतं समाः।” अर्थात् कर्मशील रहते हुए सौ वर्ष जीने की अभिलाषा करनी चाहिए। शरीर कर्मशील रहे उसके लिए स्वस्थ रहना आवश्यक है तथा स्वस्थ रहने के लिए योगाभ्यास सर्वश्रेष्ठ साधन है।

योगासन से व्यक्ति का शारीरिक, मानसिक व आध्यात्मिक विकास होता है। प्राणायाम से शरीर के अन्दर शुद्ध वायु फेफड़ों में जाती है जिससे व्यक्ति की जीवन शक्ति का विकास होता है। प्राणायाम से नाड़ियाँ शुद्ध होती हैं। मन शान्त और एकाग्र रहता है। रक्त का आवागमन नियमित रूप से होता है । प्राणायाम से जुकाम, खांसी, दमा, तपेदिक जैसे रोग नहीं होते हैं। योग का शाब्दिक अर्थ है- जोड़ना अर्थात् धार्मिक विचारधारा के अनुसार ईश्वर से मिलन का प्रयास ही योग कहलाता है। ईसा से लगभग दो शताब्दी पूर्व ज्ञानी पुरुष पतंजलि ने ‘योगशास्त्र’ ग्रन्थ की रचना की थी । योग के प्रमुख चार मार्ग – 1. भक्ति 2. ज्ञान, 3. कर्म तथा 4. राजयोग हैं। पतंजलि के अनुसार राजयोग की आठ शाखाएँ – यम, नियम, आसन, प्राणायाम, प्रत्याहार, धारणा, ध्यान तथा समाधि हैं।

मानव जीवन में योग का महत्त्वपूर्ण स्थान है। भगवान श्रीकृष्ण ने भी गोपियों की विरह वेदना को समाप्त करने के लिए योग – सन्देश लेकर उद्धव को गोपियों के पास भेजा था। बाबा रामदेव ने पतंजलि योग संस्थान के माध्यम से पतंजलि योग दर्शन को भारत में ही नहीं बल्कि पूरे संसार में स्थापित किया है। ‘योग भगाए रोग’ इस कथन को बाबा रामदेव ने अपने अभियान के माध्यम से प्रत्यक्ष चरितार्थ कर दिखाया है।

आज हज़ारों व्यक्ति योगाभ्यास के माध्यम से स्वास्थ्य लाभ ले रहे हैं। भले ही कुछ ईर्ष्यालु और निहित स्वार्थ वाले लोग उनकी आलोचना करते रहे, लेकिन उन्होंने सार्वजनिक स्वास्थ्य के अभियान को एक विश्वसनीय होगा । दिशा प्रदान की है अतः प्रत्येक व्यक्ति के लिए योग अनिवार्य है। योग की प्रमुख क्रियाएँ, जैसे-आसन, प्राणायाम, ध्यान आदि को चिकित्सा जगत में स्वास्थ्य सुधार, कुछ रोगों के उपचार, शारीरिक मुद्रा सुधार तथा तनावों को कम करने में प्रयोग किया जा रहा है। योग की प्रमुख क्रियाएँ पद्मासन, पश्चिमोत्तानासन, सर्वांगासन, धनुरासन, शवासन सूर्यनमस्कार आदि हैं।

योगासन से शरीर की हड्डियाँ, मस्तिष्क, यकृत, गुर्दे आदि शारीरिक अंग स्वस्थ तथा क्रियाशील बनते हैं। शरीर का रक्त संचार ठीक बना रहता है तथा स्मरण शक्ति बढ़ती है। शरीर का मोटापा दूर करने में योगासन बहुत उपयोगी होता है।

आज यह प्रमाणित हो चुका है कि योग के माध्यम से असाध्य रोगों पर भी नियन्त्रण किया जा सकता है अतः अस्पताल बढ़ाने को आतुर सरकार और अर्थलोलुप लोग, स्वास्थ्य रक्षा में योग की भूमिका को स्वीकार करें और उसे लोगों की दिनचर्या में उचित स्थान दिलाएँ, यह आज की महती आवश्यकता है। औषधियाँ कभी उत्तम स्वास्थ्य का रूप नहीं हो सकतीं।

CBSE Sample Papers for Class 11 Hindi Set 4 with Solutions

(ख) ‘जल ही जीवन है।’ इस कथन के आधार पर एक रचनात्मक लेख लिखिए | (5)
उत्तर:
“जल संरक्षण मेरा संकल्प इसका नहीं दूसरा विकल्प।” मानव संरक्षण के साथ-साथ सम्पूर्ण प्राणी जगत की ज़िन्दगी में पानी जरूरी तत्व है, इसके बिना जीवन की कल्पना भी नहीं की जा सकती। हमारा शरीर पाँच तत्वों से मिलकर बना है, जिनमें से एक पानी भी है “क्षितिक जल पावक गगन समीरा, पंच तत्व से बना शरीर ”

धरती पर लगभग 30 प्रतिशत जमीन एवं 70 प्रतिशत पानी है, लेकिन पीने के लायक एक प्रतिशत से भी कम है। हमें अपने प्राकृतिक जल स्रोतों जैसे; नदी, तालाब, पोखर, झील व झरने को बचाना होगा। इनको साफ-सुथरा बनाना होगा और ज़्यादा से ज़्यादा वृक्ष लगाकर आस-पास को हरा-भरा बनाकर पर्यावरण संतुलन का भी प्रयास करना होगा।

‘पानी बचाओ, जीवन बचाओ’ हमें पानी की बचत करनी होगी तथा प्राकृतिक जल स्रोतों को साफ-सुधरा बनाकर उनके पानी को पीने के लायक बनाना होगा। अधिकाधिक पेड़ लगाकर धरती को हरा-भरा बनाना होगा एवं पर्यावरण संतुलन हेतु प्रयास करते रहने होंगे। जल संरक्षण हेतु लोगों को जागरूक भी करना होगा। वर्षा जल का भंडारण एवं समुचित उपयोग करना होगा। पानी की बूंद-बूंद का संरक्षण एवं समुचित उपयोग करना ही प्राणी जगत के लिए लाभदायक ‘जल ही जीवन का आधार मत कीजिए इसको बेकार।’

(ग) भारत में विभिन्न धर्मों, जातियों एवं सम्प्रदायों के मानने वाले लोग रहते हैं । इसलिए भारतीय समाज में अनेक समस्याओं का होना भी लाज़मी है। अत: ‘ भारतीय समाज की समस्याएँ विषय पर एक रचनात्मक लेख लिखिए । (5)
उत्तर:
मनुष्य एक सामाजिक प्राणी है। उसे समाज का अंग माना गया है। मनुष्य का समस्याओं से अत्यन्त घनिष्ठ सम्बन्ध रहा है। मुनष्य को जिन समस्याओं से बार-बार दो चार होना पड़ता है, धीरे-धीरे वही सामाजिक समस्या बन जाती है। भारतीय समाज में विभिन्न सम्प्रदायों जातियों एवं धर्मों को मानने वाले लोग रहते हैं। जब भारतीय समाज में रहने वालों में विविधता बहुत है तो वहीं समस्याओं की अधिकता का होना स्वाभाविक भारतीय समाज में पाई जाने वाली समस्याओं का वर्णन निम्नलिखित हैं-

अन्धविश्वास एवं कुरीतियाँ :
भारतीय समाज अनेक तरह की कुरीतियाँ, रूढ़ियों एवं अन्धविश्वास से आज मुक्त नहीं हो पाया है। इनसे छुटकारा पाना कठिन हो गया है। लोग इनसे प्रताड़ित, शोषित होते रहते हैं पर इन्हें छोड़ने को तैयार नहीं आज भी भारतीय समाज में लोग कितने भी ज़रूरी काम के लिए घर से निकल रहे हों और बिल्ली रास्ता काट जाए या कोई छींक दे या एक आँख वाला आदमी सामने आ जाए तो इसे अपशकुन मानकर काम को छोड़कर बैठ जाते हैं। चाहे उन्हें नुकसान ही क्यों न उठाना पड़ा हो पर उन्हें इसकी फिक्र नहीं।

सिर दर्द, बदन दर्द या बुखार होने पर डॉक्टर की दवा से ज़्यादा तांत्रिक या मौलवी साहब के ताबीज़, गंडे, भभूत, पूजा स्थल की मिट्टी पर अधिक यकीन करते हैं। देखा जाए तो समाज के कुछ लोगों द्वारा वैज्ञानिक उन्नति एवं खोजों का घोर अपमान है। ऐसे अन्धविश्वासी एवं कुरीति वाले लोग वैज्ञानिकों को कम महत्ता देकर पाखण्डियों को ज़्यादा महत्व देते हैं।

जाति पाँति की समस्या जाति :
पांति की भावना ने समाज में कटुता और विषमता का जहर घोल रखा है। लोग चाहकर भी एक मंच पर आकर समता का भाव प्रदर्शित नहीं कर पाते, क्योंकि नीची जाति अथवा दलितों को सम्मान की दृष्टि से नहीं देखा जाता है। उच्च वर्ग अथवा अभिजात्य वर्ग के लोग इन्हें अपने साथ बिठाने, खाने पीने आदि में हीनता अनुभव करते हैं। इससे समाज विखण्डित होता जा रहा है।

दहेज प्रथा :
वर्तमान में यह भारतीय समाज की प्रमुख समस्याओं में से एक है। इससे सामाजिक संरचना प्रभावित हो रही है। देश के कई राज्यों में स्त्रियों का लिंगानुपात पुरुषों की तुलना में काफी तेजी से गिरा है। इसका मूल कारण दहेज की समस्या है। जिसके कारण लोग कन्या भ्रूण हत्या कराकर कन्या से छुटकारा पाना चाहते हैं

इसका समाधान होना बहुत जरूरी है, तभी समाज की प्रगति तेजी से हो सकती हैं। नारी जाति के प्रति असम्मान की भावना – आज नारी पर भिन्न-भिन्न अत्याचार हो रहे हैं तथा लिंगानुपात तेजी से गिरा है। इसका मूल कारण दहेज की समस्या है। जिसके कारण लोग कन्या भ्रूण हत्या कराकर कन्या से छुटकारा पाना चाहते हैं। इसका समाधन होना बहुत ज़रूरी है, तभी समाज की प्रगति तेजी से हो सकती है।

नारी जाति के प्रति असम्मान की भावना :
आज नारी पर भिन्न-भिन्न रूपों में अत्याचार किए जा रहे हैं। पुरुषों ने अपनी स्वार्थ पूर्ति हेतु तरह-तरह के हथकंडे अपनाएँ हैं। जिसके प्रभाव स्वरूप अनमेल विवाह, पर्दा प्रथा, बहुविवाह, बाल विवाह, विधवा विवाह जैसी अनेक समस्याएँ सामने आई हैं। इन सबका दंश नारी जाति को झेलना पड़ता है। नारी पुरुष की अर्धांगिनी है। उसकी उपेक्षा करके समाज की उन्नति की बात सोचना भी बेईमानी है।

समाज में अनेक समस्याओं के लिए उत्तरदा. ई कारक :
जनसंख्या वृद्धि जो स्वयं एक समस्या होने के साथ-साथ अनेक समस्याओं की जन्मदात्री है। बेरोजगारी इससे युवावर्ग निराश व हताश होकर असामाजिक कार्य करने का दुस्साहस कर बैठता है। इसके अलावा अशिक्षा; यह भारतीय समाज की बड़ी समस्या है। अगर सामाजिक समस्याओं पर जीत हा. सिल कर उत्तरोत्तर प्रगति पथ पर बढ़ना है तो लोगों को शिक्षित व जागरूक करना चाहिए तभी साहस व तत्परता से प्रयास करके जीत पा सकते हैं।

CBSE Sample Papers for Class 11 Hindi Set 4 with Solutions

(घ) ‘झरोखे से बाहर’ विषय पर रचनात्मक लेख लिखिए । (5)
उत्तर:
‘झरोखे से बाहर’ झरोखा भीतर से बाहर की ओर देखने का माध्यम है और बाहर से भीतर देखने का रास्ता हमारी आँखें भी एक तरह से झरोखा ही हैं। ये मन-मस्तिष्क को संसार से और संसार को मन-मस्तिष्क से जोड़ने का माध्यम मन रूपी झरोखे से किसी भक्त को संसार में ईश्वर के दर्शन होते हैं तो किसी डाकू लुटेरे को किसी सेठ की धन सम्पत्ति दिखाई देती है। झरोखा स्वयं कितना छोटा होता है पर उसके पार बसने वाला संसार इतना व्यापक है कि जिसे देखकर तन-मन की भूख जाग जाती है और कभी-कभी शान्त हो जाती है।

किसी पर्वतीय स्थल पर किसी घर के झरोखे से गगन चुंबी पर्वत मालाएँ, ऊँचे-ऊंचे पेड़, गहरी गहरी घाटियाँ, डरावनी खाइयाँ पर्यटकों को अपनी ओर खींचती है तो दूर-दूर तक घास चरती भेड़-बकरियाँ, बाँसुरी बजाते चरवाहे, पीठ पर लम्बे टोकरे बाँधकर इधर-उधर जाते सुन्दर पहाड़ी के युवक-युवतियाँ मन को मोह लेते हैं। झरोखे बहुत तरह के होते हैं पर झरोखे के पीछे बैठ प्रतीक्षारत आँखों में सदा एक ही भाव रहता है- कुछ देखने का, कुछ पाने का।

2. संगीत निदेशालय लखनऊ में प्रशिक्षित शास्त्रीय संगीत कलाकारों की आवश्यकता है। इस पद के लिए निदेशालय के निदेशक को एक आवेदन-पत्र लिखिए । (5)
अथवा
विद्यालयी शिक्षा में सुधार हेतु केन्द्रीय शिक्षामंत्री, भारत सरकार को प्रार्थना पत्र लिखिए । (5)
उत्तर:
प्रति निदेशक
संगीत निदेशालय
27 मीराबाई रोड लखनऊ
विषय -‘शास्त्रीय संगीत कलाकार’ पद हेतु आवेदन पत्र
महोदय,
दिनांक 17 अप्रैल 20XX को दिल्ली से प्रकाशित ‘टाइम्स ऑफ इंडिया’ प्रात: संस्करण समाचार पत्र में प्रकाशित विज्ञापन से ज्ञात हुआ है कि ‘संगीत निदेशालय को प्रशिक्षित शास्त्रीय संगीत कलाकारों की आवश्यकता है। मैं इस पद हेतु वांछित योग्यताएँ रखता हूँ अत: मैं इस पद हेतु आवेदन पत्र प्रस्तुत कर रहा हूँ।

मेरा स्ववृत्त (बायोडेटा) इस आवेदन पत्र के साथ संलग्न है। विज्ञापन में वर्णित सभी योग्यताओं और अर्हताओं को मैं पूर्ण करता हूँ। मेरी योग्यताओं का संक्षिप्त विवरण इस प्रकार है-
(1) मैं प्रारंभ में ही संगीत का विद्यार्थी रहा हूँ मैने इस विषय में प्रवीण और प्रभाकर भी किया हुआ है, मैंने कई संगीत महोत्सवों में शास्त्रीय संगीत की प्रस्तुतियाँ भी दी हैं।
(2) मैंने शास्त्रीय संगीत में एक कलाकार की हैसियत से अनेक पुरस्कार प्राप्त किए हैं तथा अनेक अखिल भारतीय शास्त्रीय संगीत प्रतियोगिताओं में भी विजयी रहा हूँ।
(3) मैंने साथी कलाकारों के साथ भी पूरे समूह का नेतृत्व करते हुए अनेक औपचारिक, अनौपचारिक संगीत कार्यक्रम किए हैं।
आपसे अनुरोध है कि उपर्युक्त तथ्यों को ध्यान में रखते हुए सकारात्मक दृष्टि से विचार करें और मुझे ‘संगीत निदेशालय’ में सेवाएँ प्रदान करने का अवसर देने की कृपा करें।
धन्यवाद
भवदीय
सुरेन्द्र गौतम

अथवा

सेवा में,
केन्द्रीय शिक्षा मंत्री,
भारत सरकार ।
विषय- विद्यालयी शिक्षा में सुधार हेतु।
महोदय,
किसी भी प्रकार के विकास एवं उन्नति के लिए शिक्षा एक महत्त्वपूर्ण साधन है। शिक्षा प्राप्त करने के लिए विद्यालय जाना आवश्यक होता है।

आजकल विद्यालयों में केवल ‘किताबी शिक्षा पर बल दिया जाता है जो केवल नौकरी दिलवाने तक ही सीमित रहती है। विद्यालयी शिक्षा को बेहतर बनाने के लिए विद्यालय को एक ‘सामाजिक शिक्षण केन्द्र’ के रूप में प्रस्तुत करना होगा जिससे बच्चे का सर्वांगीण विकास संभव हो सके शिक्षा में सुधार शिक्षकों की योग्यता, सक्रियता और पढ़ाने के कौशल पर भी निर्भर है। इस ओर विचार करने की महती आवश्यकता है।

एक कक्षा में 20 से ज़्यादा बच्चे न हों तो शिक्षक उन्हें भली-भाँति पढ़ा सकता है। इसके अलावा शिक्षण विधियों, प्रशिक्षण और परीक्षण की विधियों में सुधार किया जाना चाहिए, जिससे शिक्षा मैं गुणात्मक विकास संभव हो सके। कई बार पाठ्यपुस्तकें और पाठ्यक्रम बदले गए, लेकिन अनुकूल परिणाम प्राप्त नहीं हो सका। यथार्थ में पाठ्यपुस्तकें पढ़ाई का एक तुच्छ साधन मात्र होती हैं, साध्य नहीं।

मान्यवर, हमें वर्तमान शैक्षिक उद्देश्यों को भी पुनरीक्षित करना चाहिए। शिक्षा विद्यार्थियों के व्यक्तित्व विकास, अन्तर्निहित क्षमताओं के विकास करने और स्वस्थ जीवन निर्माण के लिए होनी चाहिए अतः पाठ्यक्रम लचीला और गतिविधि पर आधारित हो, साथ ही वह बच्चों की ग्रहण क्षमता के अनुरूप होना चाहिए। मुझे आशा ही नहीं पूर्ण विश्वास है कि आप व आपका मंत्रिमंडल मेरे द्वारा सुझाए गए सुझावों पर अवश्य विचार करेंगे तथा इस दिशा में मनन कर शीघ्रातिशीघ्र ठोस
कदम उठाएँगे।
धन्यवाद
भवदीय
डॉ. रितु शर्मा

CBSE Sample Papers for Class 11 Hindi Set 4 with Solutions

3. निम्नलिखित में से तीन प्रश्नों किन्हीं दो प्रश्नों के उत्तर लगभग 60 शब्दों में दीजिए- (3 × 2= 6)

(क) ‘आज आपको पार्क जाते वक्त पाँच वर्षीय रोता हुआ बच्चा मिला’ इस अवसर पर डायरी लेखन कीजिए । (3)
उत्तर:
10 अक्टूबर, 2022 आज जब में पार्क की तरफ जा रहा था तभी मेरी नज़र सड़क के किनारे पांच वर्षीय बच्चे पर पड़ी। वह रो रहा था और जोर-जोर से अम्मी अम्मी पुकार रहा था। मैं उसके पास गया और चुप करते हुए उससे रोने की वजह पूछी। तब उसने बताया कि बाज़ार से आते वक्त अपने माता-पिता से बिछुड़ गया हूं। मैंने उसको समझाया, तसल्ली ही कि जल्द आपके अम्मी-पापा मिल जाएँगे और फिर उसे अपने घर ले आया।

उसे खाना खिलाया पिलाया तथा उससे उसका पता पूछकर उसके घर छोड़ आया। अब वह बहुत खुश था। मुझे भी उसे उसके अम्मी-पापा से मिलाकर और फिर उसे खुश देखकर बहुत अच्छा महसूस हुआ।

(ख) पटकथा लेखन में कम्प्यूटर के लाभ बताइए । (3)
उत्तर:
आज जीवन के हर क्षेत्र में कम्प्यूटर की भूमिका बढ़ती जा रही है। अधिकतर सभी कार्य व्यापार कम्प्यूटर पर आधारित दिखाई पड़ते हैं। पटकथा लेखन में कम्प्यूटर की सहायता ली जा सकती है। आजकल कम्प्यूटर पर ऐसे सोफ्टवेयर आ गए हैं, जिनमें पटकथा लेखन का प्रारूप बना बनाया भी मिल जाता है। पूर्व समय में जहाँ लेखन कार्य दुष्कर हुआ करता था, वही कम्प्यूटर के माध्यम से सरल हो गया है।

यदि पटकथा-लेखन में कुछ गड़बड़ी हो तो कम्प्यूटर स्वतः संकेत कर देता है। पटकथा में सुधार लाने के सुझाव भी पटकथा लेखक को देता है इस प्रकार कमियों को पहचान कर तत्काल दूर किया जा सकता है। कम्प्यूटर के द्वारा पटकथा का संग्रह लम्बे समय तक किया जा सकता है। इतना ही नहीं, एक क्षण में दूर बैठे व्यक्ति के साथ साझा भी किया जा सकता है।

(ग) पटकथा की संरचना कैसे होती है ? (3)
उत्तर:
फ़िल्म तथा दूरदर्शन की पटकथा में पात्र – चरित्र, नायक प्रतिनायक, घटनास्थल दृश्य कहानी का क्रमिक विकास, समाधान आदि सभी कुछ होता है। इसमें छोटे-छोटे दृश्य, असीमित घटनास्थल होते हैं इसकी कथा फ्लैशबैक अथवा फ्लैश फॉरवर्ड तकनीक से किसी भी प्रकार से प्रस्तुत की जा सकती है। फ्लैश बैक से अतीत में हो चुकी और फ्लैश फॉरवर्ड से भविष्य में होने वाली घटनाओं को प्रस्तुत किया जाता है। इसमें एक ही समय में अलग-अलग स्थानों पर घटित घटनाओं को भी दिखाया जा सकता है।

CBSE Sample Papers for Class 11 Hindi Set 4 with Solutions

4. निम्नलिखित तीन प्रश्नों में से किन्हीं दो प्रश्नों के उत्तर (लगभग 40 शब्दों में) दीजिए- (2 × 2 = 4)

(क) शब्द कोश में दिए गए निम्न संकेतों से पूर्ण शब्द लिखिए । (2)
(ज्यो.), (त.), (ति), (ति.), (तु.), (दे.), (ना.), (न्या.), (प.), (पह.), (पा.), (पाराशरसं.), (पु.), (पु.), (पुर्त.), (प्र.), (प्रा.), (फा.), (फ्रे.), स्त्री. (बं.), (बं.), (बहु.), (बि.), (बी.), बुंदेल., (वृ. सं.), (बो.), (बोल.), (बौ., बौद्ध.) (भाग.), (भू.); (भू. क्रि.), (मनु.),
उत्तर:
(ज्यो.) – ज्योतिष
(त.) – तंत्रशास्त्र
(ति.) – तिब्बती
(ति.) – तिरस्कार- सूचक
(तु.) – तुर्की
(दे.) – देखिये
(ना.) – नाटक
(न्या.) – न्याय
(प.) – पद्मावत, जायसीकृत
(पह.) – पहलवी
(पा.) – पाली
(पाराशरसं.) – पाराशरसंहिता
(पु.) – पुल्लिंग
(पु.) – पुराण
(पुर्त.) – पुर्तगाली
(प्र.) – प्रत्यय
(प्रा.) – प्राचीन
(फा.) – फारसी
(फ्रें.) – फ्रेंच
(बं.) – बंगाली
(ब.) – वर्मी
(बहु.) – बहुवचन
(बि.) – बिहारी रत्नाकर
(बी.) – बीसलदेव रासो
(बुंदेल.) – बुंदेलखण्डी बोली
(वृ. सं.) – वृहत्संहिता
(बो.) – बोल-चाल
(बोल.) – बोल-चाल
(बी., बौद्ध) – बौद्ध साहित्य
(भाग.) – भागवत
(भू.) – भूषणग्रंथावली
(भू.क्रि.) – भूतकालिक क्रिया
(मनु.) – मनुस्मृति
(स्त्री.) – स्त्रीलिंग।

CBSE Sample Papers for Class 11 Hindi Set 4 with Solutions

(ख) हिन्दी में प्रयोग किए जाने वाले विदेशी भाषाओं से लिए गए शब्द लिखिए । (2)
उत्तर:
1. तुर्की से कालीन, तोप, बारूद, सराय आदि ।
2. अरबी से अजीब, अदालत, आखिर, आदमी ।
3. फारसी से- आबरू, आफत, कारीगर, किशमिश ।
4. चीनी से चाय, लीची ।
5. जापानी से -रिक्शा ।
6. पुर्तगाली से – अनन्नास, आलपीन, गमला ।
7. फ्रेंच से कारतूस, कूपन ।
8. अंग्रेजी से कोर्ट, पुलिस स्टेशन, ऑफिसर, स्कूटर।

(ग) स्ववृत्त लेखन में कौन-कौन से तत्त्व होते हैं ? (2)
उत्तर:
व्यक्ति परिचय – सबसे पहले अपने बारे में कुछ प्राथमिक जानकारियाँ दी जाती हैं जैसे आवेदक का नाम, जन्मतिथि, पिता का नाम, माता का नाम, पत्र व्यवहार का स्थायी पता, दूरभाष नम्बर, मोबाइल नम्बर, ई-मेल, आधार संख्या ।
शैक्षणिक योग्यता – परीक्षा का नाम, प्रशिक्षण, अनुभव एवं उपलब्धियाँ ।
कार्येत्तर गतिविधियाँ – किसी भी क्षेत्र एवं अभिरुचियों से सम्बन्धित एवार्ड एवं प्रमाण पत्र ।
प्रतिष्ठित व्यक्तियों के सन्दर्भ, जो उम्मीदवार के रिश्तेदार या नज़दीकी न हों किन्तु उम्मीदवार तथा उसकी योग्यता, क्षमता आदि से परिचित हों।
शैक्षणिक योग्यता को एक सारिणी के रूप में प्रस्तुत जाना चाहिए।

CBSE Sample Papers for Class 11 Hindi Set 4 with Solutions

VI. पाठ्य-पुस्तक आरोह भाग-1 (20 अंक)

1. निम्नलिखित तीन प्रश्नों में से किन्हीं दो प्रश्नों के उत्तर (लगभग 60 शब्दों में) दीजिए- (3 × 2 = 6)

(क) त्रिलोचन द्वारा रचित ‘चम्पा काले-काले अक्षर नहीं चीन्हती’ नामक कविता का सार लिखिए । (3)
उत्तर:
त्रिलोचन द्वारा रचित ‘चम्पा काले काले अक्षर नहीं ‘चीन्हती’ कविता लोक भावना से जुड़ी हुई है सुन्दर नामक ग्वाल की लड़की चंपा ठेठ अनपढ़ है। पशु चराती है। उसे काले अक्षरों में छिपे स्वरों पर आश्चर्य होता है वह लेखक के साथ शरारत करते हुए कभी उसकी कलम चुरा लेती है, कभी कागज गायब कर देती है। वह लेखक को दिन भर कागज़ गोदते रहने पर शिकायत भी करती है। लेखक उसे समझाता है कि वह भी पढ़ना-लिखना सीख ले। गाँधी बाबा भी यही चाहते हैं परन्तु चंपा पढ़ने-लिखने से मना करती है। लेखक उसे समझाता है कि जब उसकी शादी हो जाएगी और उसका बालम कलकत्ता चला जाएगा तब उसे पत्र कैसे लिखेगी? इस पर चंपा कहती है-आग लगे कलकत्ता को; वह तो अपने बालम को कलकत्ता नहीं जाने देगी। उसे अपने पास ही रखेगी।

(ख) दुष्यन्त कुमार द्वारा रचित ‘साये में धूप’ नामक कविता का सार लिखिए । (3)
उत्तर:
दुष्यन्त कुमार की यह गज़ल जन-जागरण को सन्देश देती है । कवि देश की दुर्व्यवस्था देखकर निराश है। आज़ाद भारत में सब लोगों का सपना था कि अब हर घर में रोशनी होगी, परन्तु हुआ यह कि पूरा शहर अंधेरे में डूबा हुआ है। देश की व्यवस्था ऐसी बन गई है कि यहाँ कष्ट ही कष्ट हैं। राजनीतिक व्यवस्था खुद कष्टों का कारण है इसलिए कवि का दिल करता है, यहाँ से कहीं और चले जाएँ।

यहाँ के लोग ऐसे आलसी, निकम्मे और शोषित हैं कि वे हर अन्याय को चुपचाप सह लेते हैं ये कमीज़ के न होने पर उसके लिए शोर नहीं मचाते, बल्कि अपने पाँवों से पेट ढकने की कोशिश करते हैं। ये लोग खुदा को एक हसीन नज़ारे के समान मन में पाले रहते हैं। इसी के सहारे इनकी ज़िन्दगी कट रही है। कवि की कोशिश है कि इन शोषितों की आवाज में विद्रोह हो और प्रभाव हो, जबकि ये शोषित खुद यह मानते हैं कि सत्ताधारी पत्थर कभी पिघल नहीं सकता। राजनीतिक नेता कवि की विद्रोही आवाज़ को कुचल देना चाहते हैं। अब कवि का भी निश्चय यही है कि वह क्रान्ति की आवाज़ बुलन्द करके रहेगा।

(ग) कवयित्री शिव का क्या संदेश लेकर आई हैं ? (3)
उत्तर:
अक्क महादेवी शिव जी की अनन्य भक्त हैं। उन्होंने सं. सार में शिव का संदेश प्रचारित करना चाहा जिससे प्रत्येक प्राणी को मुक्ति मिल सके। शिव करुणामयी हैं तथा संसार का कल्याण करने वाले हैं। जो भी प्राणी पवित्र मन से शिव भक्ति करता है, वे उसे मुक्ति अवश्य देते हैं। प्रत्येक प्राणी को जीवन में ऐसा अवसर नित्य प्राप्त नहीं होता है, अतः उसे इस अवसर को छोड़ना नहीं चाहिए।

CBSE Sample Papers for Class 11 Hindi Set 4 with Solutions

2. निम्नलिखित तीन प्रश्नों में से किन्हीं दो प्रश्नों के उत्तर (लगभग 40 शब्दों में) दीजिए- (2 × 2= 4)

(क) “यहाँ दरख्तों के साये में धूप लगती है” से दुष्यन्त कुमार क्या कहना चाहते हैं ? (2)
उत्तर:
दुष्यंतकुमार यह कहना चाहते हैं कि यहाँ तो दरख्त ही ऐसे हैं कि जिनकी छाया में छाँव मिलने की बजाय धूप लगती है। अर्थात् यहाँ की शासन व्यवस्था और शासक ऐसे हैं कि उनके आश्रय में रहकर कोई सुख-चैन नहीं, बल्कि कष्ट ही कष्ट हैं। आजादी के बाद नेताओं ने देश के सुख-चैन के लिए कुछ नहीं किया।

(ख) कल्पना करें, प्रेम प्राप्ति के लिए मीरा को किन-किन कठिनाइयों का सामना करना पड़ा होगा ? (2)
उत्तर:
मीरा कृष्ण की अनन्य उपासिका थी और उन्हे अपना पति व आराध्य मानती थीं इस कारण उन्हें कई प्रकार की कठिनाइयों का सामना करना पड़ा होगा। उन पर कई प्रकार की बंदिशें व पहरे लगाए गए होंगे। अपने आराध्य कृष्ण की भक्ति करने, भजन गाने व नाचने पर रोक लगायी गई होगी। ससुराल में परिवार वालों द्वारा अपमानित होना पड़ा होगा। महल से निकलने के बाद भूखे-प्यासे रह कर भटकना पड़ा होगा।

(ग) आदिवासी समाज की वर्तमान स्थिति पर टिप्पणी करें। (2)
उत्तर:
आदिवासी समाज की स्थिति में वर्तमान समय में काफी सुधार आने लगा है। इनके समाज में भी अब शिक्षा का प्रसार होने लगा है। आरक्षण की सुविधा के द्वारा सरकार इनके लिए नौकरियों में स्थान सुरक्षित कर उन्हें समाज की मुख्य धारा में सम्मिलित करने हेतु प्रयासरत है। यातायात व संचार के साधनों के कारण ये आधुनिक परिवेश में स्वयं को ढालने में समर्थ हो रहे हैं। आदिवासी समाज की उन्नति के लिए अनेक स्वयं सेवी संस्थाएँ भी प्रयासरत हैं जिनके परिणामस्वरूप ये नशे आदि की लत से बाहर आकर आगे बढ़ रहे हैं।

3. निम्नलिखित तीन प्रश्नों में से किन्हीं दो प्रश्नों के उत्तर (लगभग 60 शब्दों में) दीजिए- (3 × 2= 6)

(क) पाठ के आधार पर मियाँ नसीरुद्दीन के व्यक्तित्व का वर्णन कीजिए । (3)
उत्तर:
मियाँ नसीरुद्दीन पुश्तैनी नानवाई हैं। अपने काम या पेशे के प्रति गहरी निष्ठा और गर्व का अहसास उनके व्यक्तित्व का सबसे उभरा हुआ सबल पक्ष है। अपने बुजुर्गों बालिद और दादा साहिब के लिए उनके मन में असीम श्रद्धा है। श्रम और अभ्यास उनके हुनर का और जीवन का मूल मंत्र है वे छप्पन किस्म की रोटी बनाने के लिए मशहूर हैं। अपने पुराने ज़माने की यादें उन्हें व्याकुल कर देती हैं। नए जमाने में खाने पकाने की कद्र नहीं है- यही उनकी बूढ़ी उम्र का सबसे बड़ा दर्द है।

(ख) स्त्री के चरित्र की बनी बनाई धारणा से रजनी का चेहरा किन मायनों में अलग है ? (3)
उत्तर:
रजनी का चेहरा स्त्री के चरित्र की धारणा से बिल्कुल भिन्न है। भारतीय स्त्री शक्तिहीन, सहनशील और कोमल मानी जाती है, जबकि इस नाटक की नायिका रजनी संघर्षशील, जुझारू और शक्तिशाली है। वह स्वयं अन्याय नहीं देख सकती। अन्याय के खिलाफ आवाज़ उठाना तथा जन-आंदोलन खड़ा कर देना उसकी फितरत है।

CBSE Sample Papers for Class 11 Hindi Set 4 with Solutions

(ग) ‘ज़बान की चाबुक’ से क्या आशय है ? मास्टर त्रिलोक सिंह के किस कथन को लेखक ने ज़बान की चाबुक कहा है और क्यों ? (3)
उत्तर:
‘जुबान की चाबुक’ का तात्पर्य ऐसी कठोर वाणी से है, जो अपने प्रभाव से लोगों को तिलमिला कर रख दे। मास्टर त्रिलोक सिंह के इस कथन को कि तेरे दिमाग में तो लोहा भरा है। विद्या का ताप कहाँ लगेगा इसमें ? इसे लेखक ने ‘जुबान का चाबुक’ कहा है क्योंकि इसकी चोट शारीरिक चोट से भी अधिक पैनी थी। मास्टर जी के इस कथन ने धनराम के मन में यह भाव बैठा दिया कि वह लुहार है, विद्या उसके वश की नहीं।

4. निम्नलिखित तीन प्रश्नों में से किन्हीं दो प्रश्नों के उत्तर (लगभग 40 शब्दों में) दीजिए- (2 × 2= 4)

(क) शिवशंभू की दो गायों की कहानी के माध्यम से लेखक क्या कहना चाहता है ? (3)
उत्तर:
शिवशंभू की दो गायों की कहानी के माध्यम से लेखक वे यह कहना चाहता है कि भारत के पशु हों या मनुष्य, अपने संगी-साथियों के साथ गहरा लगाव रखते हैं। चाहे वे आपस में लड़ते-झगड़ते भी हों, तो भी उनका परस्पर प्रेम अटूट होता है। एक-दूसरे से विदा होते समय वे दु:ख अनुभव करते हैं। लेखक यह भी कहना चाहता है कि विदाई का समय करुणाजनक होता है।

CBSE Sample Papers for Class 11 Hindi Set 4 with Solutions

(ख) एक अध्यापक के रूप में त्रिलोक सिंह का व्यक्तित्व आपको कैसा लगता है? अपनी समझ में उनकी खूबियों और खामियों पर विचार करें। (3)
उत्तर:
अध्यापक त्रिलोक सिंह के व्यक्तित्व में जहाँ कुछ गुण हैं, वहीं कुछ अवगुण भी हैं। वे गाँव के स्कूल में पूरी लगन से पढ़ाते थे। वे किसी के सहयोग के बिना पाठशाला चलाते थे। उन्होंने सभी बच्चों को अनुशासन में बाँध रखा था। वे छात्रों के साथ मेहनत करना व प्रोत्साहित करना अपना कर्तव्य मानते थे। होशियार बच्चों के उज्ज्वल भविष्य की कामना करते थे। वहीं वे जातीय भेदभाव से ग्रस्त थे। पढ़ाई में कमज़ोर छात्रों की पिटाई लगवाना व उन पर व्यंग्य करना उचित न था । धनराम द्वारा तेरह का पहाड़ा न सुनाये जाने पर उसे पाँच छः दरौतियाँ धार लगाने को देना उनकी स्वार्थसिद्धि का परिचायक है।

(ग) ‘रजनी’ धारावाहिक की इस कड़ी की मुख्य समस्या क्या है ? क्या होता अगर- (3)
(i) अमित का पर्चा सचमुच खराब होता ।
(ii) सम्पादक रजनी का साथ न देता ।
उत्तर:
इस कड़ी की मुख्य समस्या स्कूली अध्यापकों द्वारा बच्चों को जबरदस्ती ट्यूशन पढ़ाने के लिए विवश करने के विरुद्ध जन-जागरण करना।
(i) यदि अमित का पर्चा सचमुच खराब होता तो रजनी को अपनी भूल का ज्ञान हो जाता। वह आन्दोलन न करती।
(ii) यदि संपादक रजनी का साथ न देता तो यह आन्दोलन इतना सफल न हो पाता। न ही इतनी जल्दी बोर्ड के अधिकारी ट्यूशन सम्बन्धी नियम बनाते।

CBSE Sample Papers for Class 10 Hindi A Set 11 with Solutions

CBSE Sample Papers for Class 10 Hindi A Set 11 with Solutions

Students must start practicing the questions from CBSE Sample Papers for Class 10 Hindi with Solutions Set 11 are designed as per the revised syllabus.

CBSE Sample Papers for Class 10 Hindi A Set 11 with Solutions

निर्धारित समय : 3 घंटे
अधिकतम अंक : 80

सामान्य निर्देश :

  • इस प्रश्नपत्र में दो खंड हैं-खंड ‘क’ और ‘ख’। खंड-क में वस्तुपरक/बहुविकल्पीय और खंड-ख में वस्तुनिष्ठ/ वर्णनात्मक प्रश्न दिए गए हैं।
  • प्रश्नपत्र के दोनों खंडों में प्रश्नों की संख्या 17 है और सभी प्रश्न अनिवार्य हैं।
  • यथासंभव सभी प्रश्नों के उत्तर क्रमानुसार लिखिए।
  • खंड ‘क’ में कुल 10 प्रश्न हैं, जिनमें उपप्रश्नों की संख्या 49 है। दिए गए निर्देशों का पालन करते हुए 40 उपप्रश्नों के उत्तर देना अनिवार्य है।
  • खंड ‘ख’ में कुल 7 प्रश्न हैं, सभी प्रश्नों के साथ उनके विकल्प भी दिए गए हैं। निर्देशानुसार विकल्प का ध्यान रखते हुए सभी प्रश्नों के उत्तर दीजिए।

खंड ‘क’
वस्तुपरक/बहुविकल्पीय प्रश्न (अंक : 40)

प्रश्न 1.
निम्नलिखित में से किसी एक गद्यांश को पढ़कर दिए गए प्रश्नों के लिए उचित विकल्प चुनिए। (1 × 5 = 5)
आत्मविश्वास मनुष्य की सबसे बड़ी पूँजी है। यह हमारे सभी आन्तरिक गुणों का शिरोमणि है, जिसके अभाव में हम लक्ष्य सिद्धि के प्रयत्नों से दूर हो जाते हैं। विद्यार्थी जीवन से ही हमें इस गुण का विकास स्वयं अपने अन्दर विकसित करना चाहिए, ताकि आने वाला भविष्य हमारे तात्कालिक लक्ष्यों से ओत-प्रोत और परिपूर्ण हो। आत्मविश्वास वस्तुतः अपनी शक्तियों की सच्ची पहचान का नाम है, अर्थात् अपनी कार्यक्षमता, कार्यकुशलता और योग्यता का ज्ञान और परख से रू-ब-रू होना और उस दिशा में अपने आपको झोंक देना तथा अपनी शक्तियों को कार्यरूप में परिणत करने की वास्तविकता ही आत्मज्ञान और आत्मविश्वास है। हमें जिस पदार्थ की कामना है, लिप्सा है उसकी सिद्धि के लिए हमें आत्मविश्वासी बनकर फल की प्राप्ति की साधना करनी होगी। दृढ़ निश्चय कीजिए, अटूट, अक्षय संकल्प कीजिए और अपने मार्ग की रुकावटों को प्रबल आत्मविश्वास से पराजित कीजिए। जिस प्रकार का व्यक्ति का आत्मविश्वास होगा, वैसी ही उसकी योग्यता होगी और उसी अनुपात में उसे फल की प्राप्ति होगी। यदि हमारी इच्छा अदम्य है तो प्रयत्न भी अदम्य होना चाहिए और उसका मूलाधार आत्मविश्वास ही है।

जिसमें आत्मविश्वास प्रबल होगा, दृढ़ इच्छा शक्ति होगी, वह व्यक्ति विपुल ज्ञान, अदम्य साहस और धन-सम्पत्ति का स्वामी होगा, हर मनोनुकूल क्षेत्र में विजेता होगा। आत्मविश्वास एक भावना है, एक अद्भुत गुण है, एक सर्वश्रेष्ठ आन्तरिक शक्ति है, एक आन्तरिक भावातिरेक है, जहाँ और जिस व्यक्ति के अन्दर इन गुणों को कार्यरूप में परिणत करने का साहस है, उत्साह है और शक्ति है, उसे सफलता और कार्यसिद्धि स्वतः ही प्राप्त हो जाती है। आत्मविश्वास ही साहस और उत्साह का जनक है, जीवन में कठोर परीक्षाएँ सम्मुख आएँ या कठिनाइयाँ बाधा डालें, आत्मविश्वासी को कोई भी परिस्थिति हतोत्साहित नहीं कर सकती।
(क) मनुष्य की सबसे बड़ी पूँजी क्या है?
(i) आत्मविश्वास
(ii) आत्ममंथन
(iii) तात्कालिक लक्ष्य
(iv) वह स्वयं
उत्तर:
(i) आत्मविश्वास।

(ख) आत्मविश्वास के अभाव में मनुष्य के साथ क्या होता है?
(i) उसमें आत्मनिर्भरता आ जाती है
(ii) वह लक्ष्य सिद्धि के प्रयत्नों से दूर हो जाता
(iii) उसका भविष्य उजज्वल हो जाता है
(iv) उसमें विपुल उत्साह आ जाता है
उत्तर:
(ii) वह लक्ष्य सिद्धि के प्रयत्नों से दूर हो जाता है।

(ग) किस प्रकार का व्यक्ति मन की इच्छा स्वरूप विजेता बन जाता है?
(i) उजज्वल भविष्य वाला
(ii) उत्साहित व्यक्ति
(iii) प्रबल आत्मविश्वासी व्यक्ति
(iv) हारा हुआ
उत्तर:
(iii) प्रबल आत्मविश्वासी व्यक्ति।

(घ) आत्मविश्वास वास्तव में किसका नाम है?
(i) आत्मज्ञानी का
(ii) अपनी शक्तियों की सच्ची पहचान का
(iii) अपने लक्ष्य का
(iv) लेखक का
उत्तर:
(ii) अपनी शक्तियों की सच्ची पहचान का।

(ङ) कथन (A) और कारण (R) को पढ़कर उपयुक्त विकल्प चुनिएकथन (A): आत्मविश्वासी मनुष्य के कार्य स्वयं सिद्ध हो जाते हैं।
कारण (R): किसी भी परिस्थिति में आत्मविश्वासी मनुष्य हतोत्साहित नहीं होता।
(i) कथन (A) गलत है किन्तु कारण (R) सही
(ii) कथन (A) और कारण (R) दोनों ही गलत
(iii) कथन (A) सही है और कारण (R) कथन (A) की सही व्याख्या है
(iv) कथन (A) सही है किन्तु कारण (R) कथन (A) की सही व्याख्या नहीं है
उत्तर:
(iii) कथन (A) सही है और कारण (R) कथन (A) की सही व्याख्या है

CBSE Sample Papers for Class 10 Hindi A Set 11 with Solutions

प्रश्न 2.
निम्नलिखित में से किसी एक पद्यांश को पढ़कर दिए गए प्रश्नों के लिए उचित विकल्प चुनिए। (1 × 5 = 5)
यदि आप इस पद्यांश का चयन करते हैं तो कृपया उत्तर-पुस्तिका में लिखें कि आप प्रश्न संख्या 2 में दिए गए पद्यांश-I पर आधारित प्रश्नों के उत्तर लिख रहे हैं। मैंने धरती के गीत सुने, अम्बर की बातें क्या जानूँ ? धरती ने पहले भी बोल सुने, धरती पर पहला स्वर फूटा, धरती ने जीवन दान दिया, धरती पर जीवन सुख लूटा। धरती माता के आँचल में, ममतामय स्नेह-दुलार मिला, धरती ने आँसू झेले हैं, धरती पर पहला प्यार खिला। धरती ने स्वर्ण बिखेरा है, नभ की सौगातें क्या जानूँ? फूलों ने हँस मोहकता दी, कलियों ने मृदु मुस्कानें दीं, मंजरियों ने मोहकता दी कोकिल ने मधुमय तानें दीं। बल्लरियों ने गलबाँहें दे, प्राणों को नव संगीत दिया, काँटों ने कठिन परीक्षा ली, जीवन का प्ररेक गीत दिया।

सोने के दिन कब देख सका, चाँदी की रातें क्या जानूँ? सूरज धरती की छाती पर, सम्पूर्ण तेज अजमाता है, नभ अपने बज्र प्रहारों से, धरती के प्राण कपाता है। ज्वालामुखियों, भूकम्पों ने, धरती पर प्रलय मचाया है। मानव ने मानव के बाघ से, धरती पर खून बहाया है, लपटों-शोलों से खेला हूँ, शीतल बरसातें क्या जानूँ?
(क) कवि को धरती माँ के आँचल में क्या मिला?
(i) ममतामय स्नेह
(ii) सुख
(iii) अन्न
(iv) फल
उत्तर:
(i) ममतामय स्नेह।

(ख) फूलों और कलियों ने धरती को क्या-क्या सौगातें दी?
(i) फूल और पत्तियाँ
(ii) नवजीवन
(iii) मोहकता और मृदु मुस्कान
(iv) गलबाँहे
उत्तर:
(iii) मोहकता और मृदु मुस्कान।

(ग) धरती पर प्रलय होने का क्या परिणाम होता है?
(i) कुछ नहीं
(ii) धरती काँप जाती है
(iii) मनुष्य प्रसन्न हो जाता है
(iv) नई सौगातें आती हैं
उत्तर:
(ii) धरती काँप जाती है।

(घ) ‘लपटों शोलों से खेलने’ का क्या अर्थ है?
(i) धरती पर प्राकृतिक आपदाएँ आना निश्चित
(ii) धरती पर लपटें निकलती हैं।
(iii) धरती शोलों से जलने लगती है।
(iv) धरती पर आग लग जाती है।
उत्तर:
(i) धरती पर प्राकृतिक आपदाएं आना निश्चित है।

(ङ) पद्यांश के अनुसार कवि को धरती से क्या मिला है?
(I) जीवन दान
(II) स्नेह दुलार
(III) निराशा
(IV) जीवन सुख
विकल्प :
(i) काथन II सही है
(ii) कथन I, II व III सही हैं
(iii) कथन I, II व IV सही हैं
(iv) कथन I, II, III व IV सही हैं
उत्तर:
(iii) कथन I, II व IV सही हैं।

अथवा

यदि आप इस पद्यांश का चयन करते हैं तो कृपया उत्तर-पुस्तिका में लिखें कि आप प्रश्न संख्या 2 में दिए गए पद्यांश-II पर आधारित प्रश्नों के उत्तर लिख रहे हैं।
जंगली फूलों ने लॉन के फूलों से
पूछा, बताओ क्या दुःख है
क्यों सूखे जा रहे हो
दिन प्रतिदिन मरे जा रहे हो !!
पीले, लाल, जामुनी, सफेद, नीले
पंचरंगे फूल बड़ी शान से
बिना पानी सड़क के किनारे
सूखे में खिल रहे थे
हर आते-जाते से बतिया रहे थे
डरते नहीं थे सर से निकलते
साँप से
ड्रोंगों के पतंगे को झपटने से
किंगफिशर की घोंसला बनाने की
तैयारी की चीख से
जंगली फूल खुश थे
कि गुलदस्ते में नहीं लगाए जाएंगे
धूप के तमतमाने पर भी इतराएँगे
जबकि सुंदर से सुंदर भी पिघलने, कुम्हलाने से नहीं बचेगा
वे जीने का उत्सव मनाते रहेंगे।
(क) धूप के तमतमाने पर भी कौन इतराएँगे?
(i) जंगली फूल
(ii) लॉन के फूल
(iii) गुलदस्ते
(iv) किंगफिशर
उत्तर:
(i) जंगली फूल।

(ख) काव्यांश में लॉन के फूल किसके प्रतीक हैं?
(i) निर्धन वर्ग के
(ii) सुविधा सम्पन्न वर्ग के
(iii) श्रमिक वर्ग के
(iv) सुन्दरता के
उत्तर:
(ii) सुविधा सम्पन्न वर्ग के।

(ग) जंगली फूल गुलदस्ते में नहीं लगाए जाने से क्यों खुश थे?
(i) क्योंकि वे वहाँ जाना नहीं चाहते थे
(ii) क्योंकि उन्हें सड़क पर उगना पसंद था
(iii) क्योंकि वहाँ जाने पर वे उनकी स्वच्छंदता उनसे छिन जाती
(iv) क्योंकि वह टूट जाते
उत्तर:
(iii) क्योंकि वहाँ जाने पर वे उनकी स्वच्छंदता उनसे छिन जाती

(घ) जीने का उत्सव कौन मनाएगा?
(i) जंगली फूल
(ii) लॉन के फूल
(iii) गुलदस्ते के फूल
(iv) संसार
उत्तर:
(i) जंगली फूल।

(ङ) कथन (A) और कारण (R) को पढ़कर उपयुक्त विकल्प चुनिएकथन (A) : किंगफिशर घोंसला बनाने की तैयारी कर रही थी।
कथन (R) : प्रत्येक जीव-जन्तु को स्वतंत्र रहकर जीना पसंद है।
(i) कथन (A) गलत है किन्तु कारण (R) सही
(ii) कथन (A) और कारण (R) दोनों ही गलत
(iii) कथन (A) सही है और कारण (R) कथन (A) की सही व्याख्या है
(iv) कथन (A) सही है किन्तु कारण (R) कथन (A) की सही व्याख्या नहीं है
उत्तर:
(iii) कथन (A) सही है और कारण (R) कथन (A) की सही व्याख्या है।

CBSE Sample Papers for Class 10 Hindi A Set 11 with Solutions

प्रश्न 3.
निम्नलिखित में से किन्हीं चार प्रश्नों के लिए उचित विकल्प चुनिए। (1 × 4 = 4)
(क) पैसे देखते ही गरीब की आँखों में चमक आ गई। वाक्य का संयुक्त रूप होगा
(i) गरीब ने पैसा देखा और उसकी आँखों में चमक आ गई।
(ii) चमक आ गई गरीब की आँखों में पैसा देखकर।
(iii) जैसे ही गरीब ने पैसा देखा वैसे ही उसकी आँखों में चमक आ गई।
(iv) पैसा देखा गरीब ने चमक आ गई आँखों में।
उत्तर:
(i) गरीब ने पैसा देखा और उसकी आँखों में चमक आ गई।

(ख) विद्यालय समय पर जाने के लिए जल्दी उठो। वाक्य का मिश्र वाक्य में रूप होगा
(i) विद्यालय समय पर जाओ तो जल्दी उठो।
(ii) यदि विद्यालय समय पर जाना है तो जल्दी उठो।
(iii) समय पर विद्यालय जाने के लिए जल्दी उठो।
(iv) जल्दी उठो विद्यालय समय पर जाने के लिए।
उत्तर:
(ii) यदि विद्यालय समय पर जाना है तो जल्दी उठो।

व्याख्यात्मक हल:
मिश्र वाक्य में एक वाक्य दूसरे पर आश्रित होता है प्रधान उपवाक्य और आश्रित उपवाक्य समुच्चयबोधक अव्यय द्वारा जुड़े हुए होते

(ग) मोहन पिताजी के पैरों में गिरा और क्षमा माँगने लगा। वाक्य का सरल रूप होगा
(i) पैरों में गिरा मोहन पिताजी के चरणों में और क्षमा माँगने लगा।
(ii) मोहन गिरा पिताजी के चरणों में और क्षमा माँगने लगा।
(iii) मोहन पिताजी के पैरों में गिरकर क्षमा माँगने लगा।
(iv) गिरा मोहन पिताजी के पैरों में और क्षमा माँगने लगा।
उत्तर:
मोहन पिताजी के पैरों में गिरकर क्षमा माँगने लगा। व्याख्यात्मक हल-सरल वाक्य में एक उद्देश्य और एक क्रिया होते हैं। अतः यही सही उत्तर है।

(घ) घंटी बजते ही छात्र कक्षा में चले गए। वाक्य का मिश्र वाक्य में रूप होगा
(i) जैसे ही घंटी बजी वैसे ही छात्र कक्षा में चले गए।
(ii) घंटी बजी और छात्र कक्षा में चले गए।
(iii) छात्र कक्षा में चले गए जैसे ही घंटी बजी।
(iv) कक्षा में गए छात्र, घंटी बजी।
उत्तर:
(i) जैसे ही घंटी बजी वैसे ही छात्र कक्षा में चले गए।

व्याख्यात्मक हल:
मिश्र वाक्य में एक वाक्य दूसरे पर आश्रित होता है प्रधान उपवाक्य और आश्रित उपवाक्य समुच्चयबोधक अव्यय द्वारा जुड़े हुए होते हैं।

(ङ) कॉलम 1 को कॉलम 2 के साथ सुमेलित कीजिए और सही विकल्प चुनकर लिखिए।

कॉलम 1 कॉलम 2
1. तुम चाय लोगे या कॉफी?  (I) मिश्र वाक्य |
2. उस लड़की को बुलाओ जिसने पीली फ्रॉक पहनी है।। (II) सरल उपवाक्य
3. पहली बार उसने इतना अच्छा अभिनय किया है। (III) संयुक्त वाक्य

विकल्प :
(i) 1 (III), 2 (I), 3 (II)
(ii) 1 (II), 2 (III), 3 (1)
(iii) 1 (I), 2 (II), 3 (III)
(iv) 1 (III), 2 (II), 3 (I)
उत्तर:
(i) 1 (III), 2 (I), 3 (II)

प्रश्न 4.
निम्नलिखित में से किन्हीं चार प्रश्नों के लिए उचित विकल्प चुनिए। (1 × 4 = 4)
(क) गरीबों का दर्द मुझसे देखा नहीं जाता। वाक्य का कर्तृवाच्य रूप होगा
(i) मैं गरीबों का दर्द नहीं देख सकता।
(ii) गरीबों का दर्द मेरे द्वारा नहीं देखा जा सकता।
(iii) नहीं देखा जा सकता मेरे द्वारा गरीबों का दर्द।
(iv) नहीं देख सकता दर्द गरीबों का।
उत्तर:
(i) मैं गरीबों का दर्द नहीं देख सकता।

(ख) भारतीय टीम खूब अच्छा खेली। वाक्य का भाववाच्य में रूप होगा
(i) खूब अच्छा खेली भारतीय टीम।
(ii) भारतीय टीम से अच्छा खेला गया।
(iii) अच्छा खेली भारतीय टीम खूब ।
(iv) खूब अच्छा भारतीय टीम खेली।
उत्तर:
(ii) भारतीय टीम से अच्छा खेला गया।

(ग) उस आदमी से कठिनाई से चला गया वाक्य का कर्मवाच्य में रूप होगा
(i) कठिनाई से चला गया उस आदमी से।
(ii) कठिनाई से चला गया उस आदमी से।
(iii) उस आदमी के द्वारा कठिनाई से चला जाता था।
(iv) कठिनाई से चला गया वह व्यक्ति।
उत्तर:
(iii) उस आदमी के द्वारा कठिनाई से चला जाता था।

(घ) कॉलम 1 को कॉलम 2 के साथ सुमेलित कीजिए और सही विकल्प चुनकर लिखिए

कॉलम 1 कॉलम 2
1. इस भवन का उद्घाटन मंत्री जी द्वारा किया गया। (I) कर्तृवाच्य
2. मोटापा घटाने के लिए रोज तेज टहले। (II) भाववाच्य
3. उससे रोया नहीं जा सकता। (III) कर्मवाच्य

विकल्प :
(i) 1 (III), 2 (I), 3 (II)
(ii) 1 (II), 2 (III), 3 (I)
(iii) 1 (1), 2 (II), 3 (III)
(iv) 1 (III), 2 (II), 3 (I)
उत्तर:
(i) 1 (III), 2 (I), 3 (II)

(ङ) मोटापा घटाने के लिए रोजाना तेज़ टहलें वाच्य का भेद बताइए
(i) कर्तृवाच्य
(ii) कर्मवाच्य
(iii) भाववाच्य
(iv) वाच्य
उत्तर:
(iii) कर्तृवाच्य।

CBSE Sample Papers for Class 10 Hindi A Set 11 with Solutions

प्रश्न 5.
निम्नलिखित में से किन्हीं चार प्रश्नों के लिए उचित विकल्प चुनिए। (1 × 4 = 4)
(क) विराज क्रिकेट खेल रहा था। रेखांकित पद का उचित परिचय होगा
(i) अकर्मक क्रिया, बहुवचन, भूतकाल।
(ii) सकर्मक क्रिया, एकवचन, भूतकाल।
(iii) द्विकर्मक क्रिया, बहुवचन, भूतकाल।
(iv) एककर्मक क्रिया, बहुवचन, भूतकाल।
उत्तर:
(ii) सकर्मक क्रिया, एकवचन, भूतकाल।

(ख) वह दरवाजे से बाहर आया। रेखांकित पद का उचित परिचय होगा
(i) जातिवाचक संज्ञा, पुल्लिंग, एकवचन।
(ii) भाववाचक संज्ञा, स्त्रीलिंग, एकवचन।
(iii) व्यक्तिवाचक संज्ञा, स्त्रीलिंग, एकवचन।
(iv) समूहवाचक संज्ञा, स्त्रीलिंग, एकवचन।
उत्तर:
(i) जातिवाचक संज्ञा, पुल्लिंग, एकवचन।

(ग) तुम्हें कोई बुला रहा है। रेखांकित पद का उचित परिचय होगा
(i) निश्चयवाचक सर्वनाम, पुल्लिंग, एकवचन।
(ii) पुरुषवाचक सर्वनाम, पुल्लिंग, एकवचन।
(iii) सम्बन्धवाचक सर्वनाम, पुल्लिंग, एकवचन।
(iv) अनिश्चयवाचक सर्वनाम, पुल्लिंग, एकवचन।
उत्तर:
(iv) अनिश्चयवाचक सर्वनाम, पुल्लिंग, एकवचन।

(घ) अभी-अभी मैंने दूरभाष पर बात की। रेखांकित पद का उचित परिचय होगा
(i) क्रियाविशेषण
(ii) संबंधबोधक अव्यय
(iii) समुच्चयबोधक अव्यय
(iv) विस्मयादिबोधक अव्यय
उत्तर:
(i) क्रियाविशेषण।

(ङ) टहलना एक अच्छा व्यायाम है। रेखांकित पद का उचित परिचय होगा
(i) भाववाचक संज्ञा, पुल्लिंग, एकवचन।
(ii) जातिवाचक संज्ञा, स्त्रीलिंग, एकवचन।
(iii) व्यक्तिवाचक संज्ञा, स्त्रीलिंग, एकवचन।
(iv) समूहवाचक संज्ञा, स्त्रीलिंग, एकवचन।
उत्तर:
(ii) भाववाचक संज्ञा, पुल्लिंग, एकवचन।

प्रश्न 6.
निम्नलिखित में से किन्हीं चार प्रश्नों के लिए उचित विकल्प चुनिए। (1 × 4 = 4)
(क) ‘चाहनहार सुवर्ण के, कविजन और सुनार’ – पंक्ति में प्रयुक्त अलंकार है
(i) श्लेष
(ii) उत्प्रेक्षा
(iii) मानवीकरण
(iv) अतिशयोक्ति
उत्तर:
(i) श्लेष

व्याख्यात्मक हल:
यहाँ ‘सुवर्ण’ के अर्थ है-सुंदर रंग और सोना। अतः यहाँ श्लेष अलंकार है।

(ख) मोर-मुकुट की चन्द्रिकनु, यौँ राजत नँद नन्द। मनु ससि सेखर की अकस, किये सेखर सत – चन्द्र ॥’ पंक्ति में प्रयुक्त अलंकार है
(i) श्लेष
(ii) उत्प्रेक्षा
(iii) मानवीकरण
(iv) अतिशयोक्ति
उत्तर:
(ii) उत्प्रेक्षा

व्याख्यात्मक हल:
यहाँ मोर पंख से बने मुकुट की चंद्रिकाओं (उपमेय) में शत-चन्द्र (उपमान) की संभावना व्यक्त की गई है। अतः यहाँ उत्प्रेक्षा अलंकार है।

(ग) ‘मैं तो मात्र मृत्तिका हूँ कुंभ और कलश बनकर जल लाती तुम्हारी अंतरंग प्रिया हो जाती हूँ॥’ -पंक्ति में प्रयुक्त अलंकार है
(i) श्लेष
(ii) उत्प्रेक्षा
(iii) मानवीकरण
(iv) अतिशयोक्ति
उत्तर:
(iii) मानवीकरण

व्याख्यात्मक हल:
यहाँ मृतिका अर्थात् मिट्टी कह रही है कि मैं कलश या कुंभ बनकर कार्य करती हूँ, जिसमें मदिरा या जल आदि को भरकर अंतरंग अर्थात् अकेलेपन की साथी या प्रिया बन जाती हूँ। अतः मानवीकरण अलंकार है।

(घ) ‘जो तनिक हवा से बाग हिली, लेकर सवार उड़ जाता था। राणा की पुतली फिरी नहीं, तब तक चेतक मुड़ जाता था – पंक्ति में प्रयुक्त अलंकार है
(i) श्लेष
(ii) उत्प्रेक्षा
(iii) मानवीकरण
(iv) अतिशयोक्ति
उत्तर:
(iv) अतिशयोक्ति

व्याख्यात्मक हल:
चेतक घोड़े के बारे में काफी बढ़ा-चढ़ाकर बताया गया है। अतः अतिशयोक्ति अलंकार है।

(ङ) ‘तुम भूल गए क्या मातृ प्रकृति को, तुम जिसके आँगन में खेले-कूदे, जिसके आँगन में सोएजागे-पंक्ति में प्रयुक्त अलंकार है।
(i) श्लेष
(ii) उत्प्रेक्षा
(iii) मानवीकरण
(iv) अतिशयोक्ति
उत्तर:
(iii) मानवीकरण

व्याख्यात्मक हल:
यहाँ प्रकृति को माता के रूप में वर्गीकृत किया गया है। अतः यहाँ मानवीकरण अलंकार है।

CBSE Sample Papers for Class 10 Hindi A Set 11 with Solutions

प्रश्न 7.
निम्नलिखित गद्यांश को पढ़कर पूछे गए प्रश्नों के लिए उचित विकल्प चुनिए। (1 × 5 = 5)
बार-बार सोचते, क्या होगा उस कौम का जो अपने देश की खातिर घर-गृहस्थी-जवानी-जिंदगी सब कुछ होम कर देने वालों पर भी हँसती है और अपने लिए बिकने के मौके ढूँढती है। दुखी हो गए। पंद्रह दिन बाद फिर उसी कस्बे से गुज़रे। कस्बे में घुसने से पहले ही ख्याल आया कि कस्बे की हृदयस्थली में सुभाष की प्रतिमा अवश्य ही प्रतिष्ठापित होगी, लेकिन सुभाष की आँखों पर चश्मा नहीं होगा, क्योंकि मास्टर बनाना भूल गया और कैप्टन मर गया। सोचा, आज वहाँ रुकेंगे नहीं, पान भी नहीं खाएँगे। मूर्ति की तरफ देखेंगे भी नहीं, सीधे निकल जाएँगे। ड्राइवर से कह दिया, चौराहे पर रुकना नहीं, आज बहुत काम है, पान आगे कहीं खा लेंगे।
(क) बार बार सोचने वाला कौन है?
(i) हालदार साहब
(ii) चश्मे वाला
(iii) ड्राइवर
(iv) मास्टर
उत्तर:
(i) हालदार साहब।

(ख) वे किस कौम के बारे में सोच रहे हैं?
(i) जिन्होंने देश की रक्षा के लिए अपने प्राणों का त्याग कर दिया
(ii) जिनके मन में स्वतंत्रता सेनानियों के प्रति सम्मान के भाव समाप्त हो गए
(iii) जो मूर्ति पर चश्मा नहीं लगा रहे हैं
(iv) जो मूर्ति देखकर निकल रहे हैं
उत्तर:
(ii) जिनके मन में स्वतंत्रता सेनानियों के प्रति सम्मान के भाव समाप्त हो गए।

(ग) आज पान न खाने के पीछे क्या कारण था?
(i) पानवाला नहीं था
(ii) उनका मन नहीं था
(iii) कैप्टन मर गया था।
(iv) उन्हें पान पसन्द नहीं था
उत्तर:
(iii) कैप्टन मर गया था।

(घ) हालदार साहब ने ड्राइवर को क्या आदेश दिया?
(i) जीप चलाने का
(ii) जीप से उतरने का
(iii) चौराहे पर नहीं रुकने का
(iv) पान खाने का
उत्तर:
(iii) चौराहे पर नहीं रुकने का।

(ङ) गद्यांश किस पाठ से लिया गया है?
(i) मानवीय करुणा की दिव्य चमक
(ii) लखनवी अंदाज़
(iii) बालगोबिन भगत
(iv) नेताजी का चश्मा
उत्तर:
(iv) नेताजी का चश्मा।

प्रश्न 8.
निम्नलिखित प्रश्नों के लिए उचित विकल्प चुनिए। (1 × 2 = 2)
(क) लखनवी अंदाज़’ पाठ में किस पर व्यंग्य किया गया है?
(i) लखनऊ पर
(ii) नवाबी परम्परा पर
(iii) लेखक पर
(iv) नए लेखकों पर
उत्तर:
(ii) नवाबी परम्परा पर।

(ख) वर्तमान समय एवं परिस्थितियों के अनुकूल काशी का सबसे उल्लेखनीय योगदान है
(i) बाबा विश्वनाथ के प्रति गहन भक्ति-भावना
(ii) परस्पर पूरकता का भाव
(iii) सुर और लय के सम्राट बिस्मिलला खाँ
(iv) मोक्षदायिनी गंगा की पावन धारा
उत्तर:
(iii) सुर और लय के सम्राट बिस्मिल्ला खां

प्रश्न 9.
निम्नलिखित पद्यांश को पढ़कर पूछे गए प्रश्नों के लिए उचित विकल्प चुनिए। (1 × 5 = 5)
कहीं साँस लेते हो,
घर घर भर देते हो,
उड़ने को नभ में तुम
पर पर कर देते हो,
आँख हटाता हूँ तो
हट नहीं रही है।
पत्तों से लदी डाल
कहीं हरी, कहीं लाल,
कहीं पड़ी है उर में
मंद गंध पुष्प माल
पाट पाट शोभा श्री
पट नहीं रही है।
(क) काव्यांश में किसके सौन्दर्य का वर्णन है?
(i) फागुन माह के सौन्दर्य का
(ii) कार्तिक माह के सौन्दर्य का
(iii) चैत्र माह के सौन्दर्य का
(iv) बैसाख माह के सौन्दर्य का
उत्तर:
(i) फागुन माह के सौन्दर्य का

(ख) कवि किससे आँख नहीं हटा पा रहा है?
(i) अपने घर से
(ii) आकाश से
(iii) प्राकृतिक सुन्दरता से
(iv) अपने परिवार से
उत्तर:
(iii) प्राकृतिक सुन्दरता से।

(ग) ‘पाट-पाट’ में कौनसा अलंकार है?
(i) रूपक
(ii) पुनरुक्तिप्रकाश
(ii) उपमा
(iv) उत्प्रेक्षा
उत्तर:
(ii) पुनरुक्तिप्रकाश।

(घ) काव्यांश के लेखक हैं
(i) सूरदास
(ii) तुलसीदास
(iii) मंगलेश डबराल
(iv) सूर्यकान्त त्रिपाठी ‘निराला’
उत्तर:
(iv) सूर्यकान्त त्रिपाठी “निराला’

(ङ) पेड़ों की डालियाँ कैसी हैं?
(i) लाल
(ii) पुष्पहीन
(iii) पत्तों से लदी
(iv) काली
उत्तर:
(iii) पत्तों से लदी।

CBSE Sample Papers for Class 10 Hindi A Set 11 with Solutions

प्रश्न 10.
निम्नलिखित प्रश्नों के लिए उचित विकल्प चुनिए। (1 × 2 = 2)
(क) गोपियों के अनुसार उद्धव किससे दूर रहे?
(i) प्रेम रूपी धागे से
(ii) गोपियों से
(iii) कमल के पत्ते से
(iv) तेल की गगरी से
उत्तर:
(i) प्रेम रूपी धागे से।

(ख) ‘आत्मकथ्य’ कविता में ‘मधुप’ शब्द का प्रयोग कवि ने किसके लिए किया है?
(i) मन रूपी भंवरे के लिए
(ii) भंवरे के लिए
(iii) तितली के लिए
(iv) फूलों के लिए :
उत्तर:
(i) मन रूपी भँवरें के लिए।

खंड ‘ब’
वर्णनात्मक प्रश्न (अंक : 40)

प्रश्न 11.
निम्नलिखित प्रश्नों में से किन्हीं तीन प्रश्नों के उत्तर लगभग 25-30 शब्दों में लिखिए। (2 × 3 = 6)
(क) नेताजी का चश्मा’ पाठ में नगरपालिका पर क्या – व्यंग्य है? स्पष्ट करें।
उत्तर:
‘नेताजी का चश्मा’ पाठ में नगरपालिका पर व्यंग्य किया गया है। कस्बे में नगरपालिका छोटे-मोटे काम करवाती रहती थी; जैसे-कभी सड़कों को पक्का करवा दिया, कभी कबूतरों के लिए छतरी बनवा दी आदि। इसी नगरपालिका के किसी उत्साही अधिकारी ने कस्बे के मुख्य बाज़ार में नेताजी की मूर्ति लगवा दी जो चश्माविहीन थी। लेखक ने इसके माध्यम से व्यंग्य किया है कि नगरपालिकाएँ दिखावे के लिए इस प्रकार की औपचारिकताएँ निभाकर अपने कर्तव्यों की इतिश्री कर लेती हैं।

(ख) पाठ ‘बालगोबिन भगत’ के आधार पर भगत के गायन की विशेषताएँ लिखिए।
उत्तर:
बालगोबिन भगत कबीर के सीधे-सादे पदों को बहुत ही मधुरता के साथ गाते थे। उनके मधुर गायन पर बच्चे, बड़े, बूढ़े, स्त्रियाँ सभी मुग्ध थे। उनके संगीत की स्वर-लहरी झंकृत कर देती थी। हलवाहे उनकी संगीत की ताल पर हल चलाते, स्त्रियों की उँगलियाँ संगीत की ताल पर रोपाई करतीं और बच्चे उनके मधुर संगीत पर झूम उठते।

(ग) बिस्मिल्ला खाँ सच्चे ईश्वर भक्त हैं? स्पष्ट कीजिए।
उत्तर:
नौबतखाने में इबादत नामक पाठ में कहते हैं कि बिस्मिल्ला खुदा में पूरा विश्वास रखते हैं। इसलिए वे सच्ची भावना और श्रद्धा से उसके द्वार पर माथा झुकाते हैं। उसके दरबार में सजदे करते हैं। उससे प्रार्थना करते हैं कि वह उसे सच्चा सुर बख्शे। उसे विश्वास है कि एक-न-एक दिन खुदा उसकी झोली में सच्चा सुर अवश्य देगा।

(घ) खीरा खाने की इच्छा होते हुए भी लेखक ने उसे खाने से इंकार क्यों किया?
उत्तर:
करीने से कटे हुए खीरे की फांकों को देखकर लेखक के मुँह में पानी आ रहा था। वे पहली बार खीरा खाने को मना कर चुके थे। अब यदि वे खीरा खा लेते तो उनका स्वाभिमान नष्ट हो जाता और नवाब साहब समझ जाते कि लेखक भी खीरा खाने को लालायित हैं।

प्रश्न 12.
निम्नलिखित प्रश्नों में से किन्हीं तीन प्रश्नों के उत्तर लगभग 25-30 शब्दों में लिखिए। (2 × 3 = 6)
(क) ‘आत्मकथ्य’ या अपनी बात कहने से प्रायः विद्वान लोग किस कारण बचना चाहते हैं? क्या प्रसाद कवि भी इसी कारण टाल देते हैं?
उत्तर:
प्रायः विद्वान लोग अपनी निजी अनुभूतियों को सार्वजनिक नहीं करना चाहते हैं। निजी अनुभूतियों की गोपनीयता को बनाए रखने के लिए वे ‘आत्मकथा लिखने से बचना चाहते हैं। कवि प्रसाद जी भी इसी कारण आत्मकथा लिखने के प्रश्ना को टाल देते हैं, क्योंकि अगर आत्मकथा लेखन में ईमानदारी बरती गई तो कवि या लेखक की अनेक निजी बातें सार्वजनिक हो जाएंगी और यदि आत्मकथा ईमानदारी से नहीं लिखी गई तो यह लेखन के साथ अन्याय होगा।

(ख) ‘उत्साह’ कविता में बादल किन-किन अर्थों की ओर संकेत करता है?
उत्तर:
‘उत्साह’ कविता में बादल निम्नलिखित अर्थों की ओर संकेत करता है
(i) नवीन कल्पना लाने वाला।
(ii) क्रांति और परिवर्तन लाने वाला।
(iii) बिजली की चमक के समान ओजस्विता वाला।
(iv) प्यासे जन की प्यास बुझाने वाला।

(ग) लक्ष्मण ने परशुराम को वीरों की क्या विशेषताएँ बतायीं?
उत्तर:
लक्ष्मण ने परशुराम को कहा जो शूरवीर होते हैं, वे शत्रु को युद्धक्षेत्र में सामने देखकर युद्ध करते हैं, अपने मुँह से अपनी प्रशंसा नहीं करते। जो अपने मुँह से प्रशंसा करते हैं, वे कायर होते हैं, वीर नहीं।

(घ) मुख्य गायक की सफलता का श्रेय संगतकार को न दिया जाना समाज की किस प्रवृत्ति का परिचायक है? इस प्रवृत्ति से क्या हानियाँ है?
उत्तर:
मुख्य गायक के द्वारा अंतरे की जटिल तानों में खोने पर संगतकार उसकी आवाज में अपनी आवाज मिलाकर उसे विश्वास दिलाता है कि वह अकेला नहीं है और छूटे हुए राग को फिर से गाया जा सकता है। ऐसे समय में संगतकार की भूमिका का महत्त्व समझ आता है।

CBSE Sample Papers for Class 10 Hindi A Set 11 with Solutions

प्रश्न 13.
निम्नलिखित प्रश्नों में से किन्हीं दो प्रश्नों के उत्तर लगभग 50-60 शब्दों में लिखिए। (4 × 2 = 8)
(क) लेखक ने प्रत्यक्ष अनुभव और अनुभूति में क्या अंतर बताया है?
उत्तर:
लेखक ने प्रत्यक्ष अनुभव और अनुभूति में अंतर बताते हुए कहा है कि जो घटना घटित होती है, उसका अनुभव होता है अर्थात् अनुभव तो घटित का होता है। इसके विपरीत अनुभूति संवेदना और कल्पना के सहारे उस सत्य को आत्मसात कर लेता है जो कृतिकार के साथ घटित नहीं हुआ है। जो आँखों के सामने नहीं आया अर्थात् जो घटित के अनुभव में नहीं आया, वही आत्मा के सामने ज्वलंत प्रकाश में आ जाता है, तब वह अनुभूति प्रत्यक्ष हो जाती है।

(ख) ग्रामीण बच्चों का बचपन शहरी बच्चों से अधिक प्रफुल्लित और आनन्द देने वाला होता है। क्यों? पाठ ‘माता का अंचल’ के आधार पर लिखिए।
उत्तर:
ग्रामीण बच्चों के बचपन में और शहरी जीवन के बच्चों के बचपन में बहुत अन्तर होता है। ग्रामीण अंचल में शहरों की तरह आधुनिक तकनीक के खिलौने नहीं होते और न ही घर के अन्दर बच्चों के खेलने के लिए किसी भी प्रकार के खेल-खिलौने होते हैं। बच्चे सारा दिन घर से बाहर मित्रों के साथ धूल, मिट्टी, खेत-खलिहानों में ही अपने खेल खेलते हैं। घर से बाहर मित्रों के साथ उनका बचपन ज्यादा आनन्द देने वाला और प्रफुल्लित होता है, क्योंकि उनको माता-पिता व किसी ओर की कोई रोक-टोक नहीं होती और न ही किसी अनुशासन में बँधे रहने की आवश्यकता होती है। स्वच्छन्द होकर बच्चे अपने क्रियाकलाप करते हैं, स्वयं अपने लिए खिलौने मिट्टी से बनाते हैं और उनका जीवन शहरी बच्चों की तुलना में अधिक रचनात्मक होता है।

(ग) प्रकृति ने जल संचय की व्यवस्था किस प्रकार की है?
उत्तर:
प्रकृति ने हिम शिखरों के रूप में बड़े ही अद्भुत ढंग से जल संचय की व्यवस्था की है। प्रकृति सर्दियों में जल को बर्फ के रूप में इकट्ठा कर लेती है। जब गर्मियों में जल के लिए सर्वत्र त्राहि-त्राहि मच रही होती है तो यही हिम शिखर पिघल कर जलधारा बन कर नदियों में जाकर मिलती है और प्यासे कंठों की प्यास बुझाती है तथा यही पानी नदियों के माध्यम से कृषि के काम आता है। इस प्रकार प्रकृति ने जल संचय की बड़ी ही अनुपम व्यवस्था की हुई है।

प्रश्न 14.
निम्नलिखित में से किसी एक विषय पर दिए गए संकेत बिन्दुओं के आधार पर लगभग 120 शब्दों में एक अनुच्छेद लिखिए। (6)
(क) विज्ञापनों का बढ़ता प्रभाव-संस्कृति पर प्रहार

  • भूमिका-विज्ञापन क्या है?
  • विज्ञापनों की आवश्यकता
  • विज्ञापनों का प्रभाव
  • बढ़ती व्यावसायिकता घटते मूल्य. विज्ञापन का दुष्प्रभाव
  • रोकने के उपाय
  • उपसंहार।

अथवा

(ख) आतंकवाद-एक नासूर

  • प्रस्तावना
  • आतंकवाद के कारण
  • आतंकवाद के दुष्परिणाम
  • भारत में आतंकवाद की स्थिति
  • आतंकवाद को रोकने के उपाय
  • उपसंहार।

अथवा

(ग) जीवन में व्यायाम का महत्त्व

  • भूमिका
  • शारीरिक स्वास्थ्य आवश्यक
  • व्यायाम के लाभ
  • सुन्दर-स्वस्थ शरीर का स्वरूप
  • मन पर प्रभाव
  • व्यायाम न करने से हानियाँ
  • उपसंहार।

उत्तर:
(क) विज्ञापनों का बढ़ता प्रभाव-
संस्कृति पर प्रहार विज्ञापन शब्द दो शब्दों के मेल से बना है-वि + ज्ञापन अर्थात् विशेष जानकारी। किसी भी वस्तु, उत्पाद या किसी भी तथ्य की विशेष रूप से आकर्षक तरीके से जानकारी को हम विज्ञापन कहते हैं। आज का युग विज्ञापन का युग है। जब से उपभोक्ता संस्कृति का प्रचार और प्रसार हुआ है तब से विज्ञापनों की भरमार और बाढ़-सी आ गयी है। उपभोक्ताओं को आकर्षित करने और वस्तु के विक्रय के लिए विज्ञापनों का प्रचार-प्रसार किया जाता है। आज प्रतियोगिता का समय है। इसलिए प्रत्येक उत्पादक कम-से-कम दाम लगाकर अधिक-से-अधिक लाभ अर्जित करना चाहता है। अपनी वस्तु की बिक्री को बढ़ाने और अधिक लाभ कमाने के लिए व्यापारी विज्ञापनों का सहारा लेते हैं। विज्ञापन समाचार-पत्रों के द्वारा, पत्रिकाओं के माध्यम से, रेडियो और दूरदर्शन के माध्यम से, हैण्ड बिल छपवाकर, दीवारों पर लिखवाकर और उद्घोषक के द्वारा घोषणा कराकर दिए जाते हैं। कई बार एक कम्पनी अपने प्रचार के लिए कई-कई माध्यमों का सहारा लेती है। कवि सम्मेलनों, दिवाली मेलों और कई प्रकार के कार्यक्रमों के विज्ञापन भी देखे जा सकते हैं। विज्ञापनों की भाषा बड़ी आकर्षक और लच्छेदार होती है। वस्त्र आदि का विज्ञापन करते समय नाना प्रकार की सुन्दरियों को उपयोग में लाया जाता है। परिधान व्यवसाय में तो विज्ञापन सुन्दरियों के सहारे ही आगे बढ़ते हैं। विज्ञापनदाता विश्व सुन्दरियों, अभिनेत्रियों, अभिनेताओं और खिलाड़ियों का उपयोग करते हैं। वे उन्हें इसके लिए अच्छी-खासी राशि देते हैं। कभी-कभी विज्ञापनदाता व्यवसायों की धार्मिक भावनाओं का भी पूरा-पूरा लाभ उठाते हैं। कई बार हम विज्ञापनों की चकाचौंध में इतने खो जाते हैं कि सही निर्णय नहीं ले पाते। इस प्रकार विज्ञापन हमें पथभ्रष्ट भी करते हैं और हमारा ज्ञानवर्द्धन भी करते हैं। यदि हम इनका सदुपयोग उचित दिशा में करते हैं तो यह हमारे लिए सहायक होते हैं और यदि इनके पीछे अनावश्यक रूप से भागते हैं तो यह हमें दिशा-भ्रमित भी करते हैं।

अथवा

(ख) आतंकवाद-एक नासूर
आतंकवाद वर्तमान समय की एक गम्भीर समस्या है। आतंकवादी वह व्यक्ति होता है जो अपना स्वार्थ पाने के लिए लोगों में भय फैलाता है। सामान्यतः दो प्रकार के आतंकवाद होते हैं। एक तो राजनीतिक आतंकवाद, जो अपने राजनीतिक स्वार्थ की पूर्ति के लिए भय फैलाते हैं और दूसरा आपराधिक आतंकवाद, जो अपहरण करके रुपये माँगते हैं। आतंकवाद हिंसा का एक गैर-कानूनी तरीका है जो लोगों को डराने के लिए आतंकवादियों के द्वारा प्रयोग किया जाता है। आज आतंकवाद एक सामाजिक मुद्दा बन चुका है। इसका इस्तेमाल आम लोगों और सरकार को डराने- धमकाने के लिए हो रहा है। बहुत आसानी से अपने लक्ष्य को प्राप्त करने के लिए विभिन्न सामाजिक संगठन, राजनीतिज्ञ और व्यापारिक उद्योगों के द्वारा आतंकवाद का इस्तेमाल किया जा रहा है। लोगों का समूह जो आतंकवाद का समर्थन करते हैं, उन्हें आतंकवादी कहते हैं। आतंकवाद को परिभाषित करना बहुत आसान नहीं है, क्योंकि इसने अपनी जड़ें बहुत गहराई तक जमायी हुई हैं। आतंकवादियों के पास कोई नियम और कानून नहीं है; वे समाज और देश में आतंक के स्तर को बढ़ाने और उत्पन्न करने के लिए केवल हिंसात्मक गतिविधियों का सहारा लेते हैं। पूरे विश्व के लिए आतंकवाद एक राष्ट्रीय और अन्तर्राष्ट्रीय समस्या बन चुका है। यह एक वैश्विक समस्या है जिसने लगभग सभी राष्ट्रों को प्रत्यक्ष और अप्रत्यक्ष रूप से प्रभावित किया हुआ है। हालांकि बहुत सारे देशों के द्वारा आतंकवाद का सामना करने की कोशिश की जा रही है, लेकिन कुछ लोगों के द्वारा इसे आज भी समर्थन दिया जा रहा है। आम लोगों को किसी भी समय खौफनाक तरीके से डराने का एक हिंसात्मक कुकृत्य है, आतंकवाद। आतंकवादियों के बहुत सारे उद्देश्य होते हैं जैसे-कि समाज में हिंसा के डर को फैलाना, राजनीतिक उद्देश्यों की प्राप्ति। इनके निशाने पर हर वक्त देश का आम नागरिक होता है। आतंकवाद का जिस ढंग से विस्तार हो रहा है, यदि उसको समय रहते नहीं रोका गया तो भारत सहित सभी देशों के लिए यह एक विकट समस्या बन जाएगा। दुनिया के सभी देशों को मिलकर ऐसी आपराधिक प्रवृत्ति पर रोक लगाने के प्रयास करने चाहिए।

अथवा

(ग) जीवन में व्यायाम का महत्त्व
कहावत है कि स्वस्थ शरीर में ही स्वस्थ मन का विकास होता है। स्वस्थ मन में अच्छे विचार आते हैं, इन्हें अपनाकर हम स्वस्थ रह सकते हैं। जैसे सुबह टहलने से शरीर में चुस्ती-फुर्ती आती है। इसी तरह थोड़ा व्यायाम करना चाहिए, खाना खाने के बाद व्यायाम नहीं करना चाहिए। छोटे बच्चों को थोड़ा टहलना चाहिए, कबड्डी तथा इस तरह के विभिन्न खेल खेलने चाहिए। स्वस्थ रहने के लिए नियमानुसार चलना चाहिए। समय पर खाना-पीना करना चाहिए। समय पर सभी काम करके व्यक्ति स्वस्थ रह सकता है। व्यायाम करने से धमनियाँ सही काम करती हैं और शरीर के सभी अंग ठीक रहते हैं तथा सही काम करते हैं। सुबह पार्क में टहलने से आँखों की ज्योति बढ़ती है। इस प्रकार हमें स्वस्थ और खुश रहना चाहिए। व्यायाम के अनेक प्रकार हैं। अपने शरीर की आवश्यकता के अनुसार हमें ऐसा व्यायाम चुनना चाहिए जो हमारी शारीरिक आवश्यकता पूर्ण करने के अलावा हमारी पहुँच में हो और साथ ही यदि यह हमारे लिए रुचिकर भी हो तो सोने पै सुहागा होगा। व्यायाम का अर्थ है-शरीर को इस तरह तानना-सिकोड़ना कि वह सही स्थिति में कार्य कर सके। जिस प्रकार अच्छे भोजन से शरीर को पोषण मिलता है उसी प्रकार से व्यायाम से शरीर लम्बे समय तक उचित दशा में बना रहता है। व्यायाम से शरीर को सुगठित, तन्दुरुस्त और फुर्तीला बनाया जा सकता है। व्यायाम से मानसिक तनाव और थकान मिटती है। शरीर का रक्त शुद्ध हो जाता है तथा पूरे शरीर में ताजगी आती है। व्यायाम यदि खले स्थान में या किसी पार्क में किया जाए तो अधिक लाभ मिलता है। फेफड़ों को शुद्ध वायु पर्याप्त मात्रा में मिलती है। पेशियों और अस्थियों को ताकत मिलती है। शरीर का दर्द और ऐंठन मिट जाती है। ज्ञानेन्द्रियों में सजगता आती है। सम्पूर्ण शरीर प्राकृतिक दशा में कार्य करने लगता है। शारीरिक दुर्बलता और सुस्ती समाप्त हो जाती है। इस तरह व्यायाम के प्रभाव से शरीर में नवजीवन का संचार होता है। बूढ़े और बीमार लोग भी अपने भीतर स्फूर्ति का अनुभव करने लगते हैं।

CBSE Sample Papers for Class 10 Hindi A Set 11 with Solutions

प्रश्न 15.
अपनी छोटी बहन को जल का महत्त्व समझाते हुए जल बचाओ पर एक पत्र लिखिए। (5)
अथवा
आपके क्षेत्र में वन विभाग के द्वारा लगाए गए पेड़-पौधे सूखते जा रहे हैं। इन पौधों के रख-रखाव और जीवित रखने के लिए वन अधिकारी को एक सुझाव पत्र लिखिए।
उत्तर:
परीक्षा भवन,
क ख ग
राजा पार्क, आगरा।
दिनांक…………….
प्रिय अनुजा रिया,
शुभ चिरंजीव। मैं यहाँ पर कुशलतापूर्वक हूँ तथा आशा करता हूँ कि तुम भी वहाँ पर खुशहाल होंगी। आज मैं इस पत्र के द्वारा तुम्हें जल के महत्त्व पर कुछ महत्त्वपूर्ण बातें बताना चाहता हूँ। जल हमारे जीवन की परम आवश्यकता है। हवा के बाद जल हमारी दूसरी प्राथमिक आवश्यकता है। आज हमारी नई पीढ़ी के द्वारा जल का बहुतायत से दुरुपयोग किया जा रहा है। जल के बिना हम अपने जीवन के एक दिन की भी कल्पना नहीं कर सकते। सोचो, यदि जल न हो तो हम पीने के अलावा अपने दैनिक कार्यों को बिना जल के कैसे सम्पन्न कर सकते हैं?
जल का संरक्षण अत्यन्त आवश्यक है वरना शीघ्र ही जल धरती से समाप्त हो जाएगा और हमारा जीवन भी बिना जल के समाप्त हो जाएगा। मुझे उम्मीद है कि तुम मेरी बात पर अमल करोगी और अपने आस-पास सभी को जल संरक्षण के महत्त्व को बताने में, भविष्य के लिए जल बचाने में और आने वाली पीढ़ी के जीवन को सुखमय बनाने में अपना योगदान दोगी। घर में माता-पिताजी को मेरा सादर प्रणाम देना।
तुम्हारा अग्रज
त थ द

अथवा

परीक्षा भवन,
क ख ग
राजा पार्क, आगरा।
वन अधिकारी,
वन विभाग,
भारत नगर, आगरा।
विषय-सूखते जा रहे पौधों की जानकारी के लिए पत्र।
महोदय,
सविनय निवेदन इस प्रकार है कि आपके वन विभाग के कर्मचारियों के द्वारा वन महोत्सव पर वृक्षारोपण कार्यक्रम में हमारे क्षेत्र में कुछ पौधे लगाये गए थे, परन्तु वर्षा न होने के कारण वे सूखते जा रहे हैं। आपके विभाग के माली भी पौधों की सिंचाई नहीं करते। इस विषय पर हमने आपके क्षेत्रीय कार्यालय में शिकायत की, लेकिन हमारी बात पर कोई सुनवाई नहीं हुई। अतः आपसे निवेदन है कि आप अपने कर्मचारियों को शैक्षणिक योग्यता उचित निर्देश देकर हमारी समस्या का समाधान करने का कष्ट करें। मैं और मेरे क्षेत्र के निवासी आपके अत्यन्त आभारी रहेंगे।
सधन्यवाद।
भवदीय, प फ ब।
दिनांक…………..

प्रश्न 16.
टाइम्स मल्टीनेशनल कंपनी में लेखापाल (अकाउंटेंट) के पद पर नियुक्ति के लिए स्ववृत्त बनाइए।
अथवा
आप हॉवर्ड यूनिवर्सिटी में माइक्रोबायोलॉजी में प्रवेश लेना चाहते हैं। इसके लिए समस्त जानकारियों को ज्ञात करने के लिए वहाँ के डीन को ईमेल कीजिए। (5)
उत्तर:
सेवा में
महाप्रबंधक महोदय,
टाइम्स मल्टीनेशनल कंपनी,
बापू नगर, जयपुर
विषय-लेखापाल (अकाउंटेंट) पद के लिए आवेदन पत्र।
महोदय
दिनांक 14 सितंबर, 2022 के दैनिक भास्कर में प्रकाशित विज्ञापन से ज्ञात हुआ है कि आपकी कंपनी में लेखापाल (अकाउंटेंट) की आवश्यकता है। इस पद के लिए मैं अपना आवेदन-पत्र प्रस्तुत कर रहा हूँ। मेरा स्ववृत्त इस आवेदन-पत्र के साथ संलग्न है। आपके विज्ञापन में वर्णित सभी योग्यताओं को मैं पूरा करता हूँ। मैं आपको विश्वास दिलाता हूँ कि मैं अपना कार्य पूर्ण निष्ठा से करूँगा। मेरा संक्षिप्त विवरण इस प्रकार है

नाम : पल्लव शर्मा
पिता का नाम : पुलकित शर्मा
माता का नाम : अंजली शर्मा
जन्मतिथि : 20 जुलाई, 1992
वर्तमान पता : 13, महावीर नगर, जयपुर
मोबाइल : 24XXXX1234
ईमेल : [email protected]

अन्य योग्यताएँ व अनुभव

  • कंप्यूटर का उच्च ज्ञान
  • हिंदी, अंग्रेजी भाषा की जानकारी।
  • पॉवर पल्प कंपनी में लेखापाल के रूप में कार्य करने का तीन वर्ष का अनुभव
  • टैली, जीएसटी में कार्य करने का अनुभव

उपलब्धियाँ

  • राज्य स्तरीय वाद-विवाद प्रतियोगिता में प्रथम पुरस्कार ।
  • विद्यालयी स्तर निबंध प्रतियोगिता में प्रथम पुरस्कार

कार्येत्तर गतिविधियां और अभिरुचियाँ

  • इन्टरनेट पर इनकम टैक्स से संबंधित ब्लॉग लिखना
  • आर्थिक गतिविधि से जुड़ी पत्रिकाएँ पढ़ना

आपसे अनुरोध है कि उपर्युक्त तथ्यों को ध्यान में रखते हुए मेरे आवेदन-पत्र पर सकारात्मक विचार किया जाए। मैं आपको विश्वास दिलाता हूँ कि मैं ईमानदारी से कार्य करूँगा और अपने कार्य व्यवहार से सभी को संतुष्ट रलूँगा।
धन्यवाद
भवदीय
हस्ताक्षर पल्लव शर्मा
दिनाकः: X/XX/XXXX
अथवा

From : [email protected]
To : [email protected]
Cc: ……
Bcc : ….
विषय-माइक्रोबायोलॉजी में प्रवेश से संबंधित जानकारी हेतु
महोदय
निवेदन है कि मैंने इस वर्ष स्रातक की परीक्षा 95% अंकों में उत्तीर्ण की है। मेरी रुचि माइक्रोबायोलॉजी में होने के कारण मैं आपकी यूनिवर्सिटी में प्रवेश लेने का इच्छुक हूँ। मुझे प्रवेश से संबंधित सभी जानकारियों से अवगत करवाने की कृपा करें। साथ ही छात्रवृत्ति से संबंधित जानकारी भी देने का प्रयास करें। आशा है कि आप मुझे सभी जानकारियाँ उपलब्ध करवाने की कृपा करें।
धन्यवाद
भवदीय
क. ख. ग.

CBSE Sample Papers for Class 10 Hindi A Set 11 with Solutions

प्रश्न 17.
सोने के आभूषणों के विक्रेता ‘कल्याण ज्वेलर्स’ के लिए लगभग 60 शब्दों में एक आकर्षक विज्ञापन तैयार कीजिए।
अथवा
पिता के नाम पुत्र का सन्देश लगभग 80 शब्दों में लिखिए जिसमें किसी यात्रा के स्थगन का वर्णन है। (4)
उत्तर:
CBSE Sample Papers for Class 10 Hindi A Set 11 with Solutions 1

अथवा

संदेश

दिनांक :…………….
समय :……………
पिताजी,
आज शाम 6 बजे सीताराम अंकल आये थे। वे कह रहे थे कि आप लोगों का कल शाम जोधपुर जाने का जो कार्यक्रम था , वह स्थगित हो गया है। आप उनसे फोन पर बात कर लें। शेष बातें वे आपको फोन पर ही बता देंगे।
सूरज

CBSE Sample Papers for Class 12 Political Science Set 2 with Solutions

CBSE Sample Papers for Class 12 Political Science Set 2 with Solutions

Students must start practicing the questions from CBSE Sample Papers for Class 12 Political Science with Solutions Set 2 are designed as per the revised syllabus.

CBSE Sample Papers for Class 12 Political Science Set 2 with Solutions

Time Allowed: 3 Hours
Maximum Marks: 80

General Instructions :

All the questions are compulsory.

  1. Question numbers 1-12 are multiple choice questions of one mark each.
  2. Question numbers 13-18 are of 2 marks each. Answers to these questions should not exceed 50 words each.
  3. Question numbers 19-23 are of 4 marks each. Answers to these questions should not exceed 100 words each.
  4. Question numbers 24-26 are passage, cartoon, and map-based questions. Answer accordingly.
  5. Question numbers 27-30 are of 6 marks each. Answers to these questions should not exceed 170 words.

Section – A (12 Marks)

Question 1.
ARF was established in the year_______ [1]
(A) 1992
(B) 1993
(C) 1994
(D) 1995
Answer:
(C) 1994

Explanation: ASEAN Regional Forum was the body that was established for the negotiation on different issues in the year 1994.

Question 2.
_______influenced the establishment of the organisation for European Economic Cooperation in 1948. [1]
(A) Marshall Plan
(B) Molotov Plan
(C) Manhattan Plan
(D) Warsaw Plan
Answer:
(A) Marshall Plan

Explanation: The Marshall Plan was put into place for the redevelopment of the Europe after the end of the World War II and this plan inspired the establishment of EEC.

Question 3.
Which one of the following was NOT given primacy by the makers of the Soviet System? [1]
(A) Abolition of private property
(B) Society based on the principle of equality
(C) No opposition party to be allowed
(D) No state control over economy
Answer:
(D) No state control over economy

Explanation: The USSR supported the idea of establishment of a centrally planned economy where there was total control of state over the economic affairs.

Question 4.
Who was appointed as the first Chief Election Commissioner of India. [1]
(A) Sukumar Sen
(B) H.J. Kania
(C) VP Menon
(D) C. Rajagopalachari
Answer:
(A) Sukumar Sen.

Explanation: The first Chief Election Commissioner of India was Sukumar Sen who successfully completed the first two general elections.

Assertion Reason Questions: Directions for Q.Nos. 5 and 6
In the question given below, there are two statements marked as Assertion (A) and Reason (R). Read these statements and choose one correct answer from the given options
(A) Both A and R are true and R is the correct explanation of A
(B) Both A and R are true and R is not the correct explanation of A
(C) A is true but R is false
(D) A is false, but R is true

CBSE Sample Papers for Class 12 Political Science Set 2 with Solutions

Question 5.
Assertion (A): The Soviet Union and Communism collapsed in 1991.
Reason (R): The internal weaknesses of the Soviet political and economic institutions failed to meet the aspirations of the Soviet people. [1]
(A) Both A and R are true and R is the correct explanation of A
(B) Both A and R are true and R is not the correct explanation of A
(C) A is true but R is false
(D) A is false, but R is true
Answer:
(A) Both A and R are true and R is the correct explanation of A

Explanation: The disintegration of the USSR took place in the year 1991 due to the political and economic crisis which the bloc was facing in its influenced nations.

Question 6.
Assertion (A): India did not give legitimacy to the communist government in China as per its foreign policy. [1]
Reason (R): India and China agreed to follow the ideals of the Panchsheel
(A) Both A and R are true and R is the correct explanation of A
(B) Both A and R are true and R is not the correct explanation of A
(C) A is true but R is false
(D) A is false, but R is true
Answer:
(D) A is false, but R is true

Explanation: India was one of the earliest nations of Asia to give legitimacy to the communist government in China and the two nations agreed to follow the ideals of the Panchsheel.

Question 7.
Which of the following leader of East Pakistan was arrested by the Pakistani army which led to the war of 1971? [1]
(A) Sheikh Mujibur Rehman
(B) Zia-ur Rehman
(C) Sheikh Hasina
(D) Zulfiqar Ali Bhutto
Answer:
(A) Sheikh Mujibur Rehman.

Explanation: West Pakistan made the arrest the head of the Awami League, Sheikh Mujibur Rehman, in East Pakistan which led to the beginning of the resistance movement in East Pakistan.

Question 8.
In 1966_______contested Indira Gandhi for the post of Prime Minister from among Congress MPs. [1]
(A) Morarji Desai
(B) Lai Bahadur Shastri
(C) Gulzari Lai Nanda
(D) Jayaprakash Narayan
Answer:
(A) Morarji Desai

Explanation: After the death of Lai Bahadur Shastri there was a competition between Morarji Desai and Indira Gandhi for the post of Prime Minister.

Question 9.
Which among the following would give more weightage to India’s proposal for permanent membership in the Security Council? [1]
(A) Nuclear capability
(B) It has been a member of the UN since its inception
(C) It is located in Asia
(D) India’s growing economic power and stable political system
Answer:
(D) India’s growing economic power and stable political system

Explanation: The rising economic and political impact of the Indians in the world makes a strong contenders for the upliftment of their position in the Security Council.

Question 10.
The key to growing India-Israel ties are: [1]
(A) defense
(B) religion
(C) trade
(D) politics
Answer:
(A) defense

Explanation: The growing proximity between India and Israel is due to the defense cooperation between the two nations.

CBSE Sample Papers for Class 12 Political Science Set 2 with Solutions

Question 11.
The political party with C. Rajagopalachari as one of the prominent leaders and the one which was formed after the Nagpur Resolution of Congress was: [1]
(A) Communist Party
(B) Swatantra Party
(C) Congress Socialist Party
(D) Jana Sangh
Answer:
(B) Swatantra Party

Explanation: C. Rajagopalachari established the Swatantra Party to give strong opposition to the unilateral rule of Congress.

Question 12.
Dalit Panthers were formed and acted strongly in the state of_______ [1]
(A) Orissa
(B) Maharashtra
(C) Madhya Pradesh
(D) Uttar Pradesh
Answer:
(B) Maharashtra

Explanation: Dalit Panthers were formed in the state of Maharashtra and fought for several reforms.

Section – B (12 Marks)

Question 13.
What is meant by a multipolar world order as visualized by India and Russia? [2]
Answer:
By a multipolar world order, they meant for the co-existence of several powers in the international system, collective security, greater regionalism, negotiated settlements of international conflicts, an independent foreign policy for all countries and decision-making through bodies like the UN which should be strengthened, democratized and empowered.

Question 14.
Has there been resistance to globalization in India? [2]
Answer:
Yes, resistance to globalization in India has come from different quarters.
(1) There have been left-wing protests to economic liberalization voiced through political parties as well as through forums like the Indian Social Forum.
(2) Resistance to globalization has also come from the political fight taking the form of objecting particularly to various cultural influences ranging from the availability of foreign television channels provided by cable networks, a celebration of Valentine’s Day, and westernization of the dress tastes of girl students in schools and colleges.

Question 15.
How was the reorganization of North-East India completed and by when? [2]
Answer:
The reorganization of the Northeast was almost completed in 1972. Meghalaya was carved out of Assam in 1972. Manipur and Tripura too emerged as separate states. The states of Mizoram and Arunachal Pradesh came into being much later. Nagaland had become a state in 1963.

Question 16.
Why is Japan considered as a rich country? [2]
Answer:
Japan was considered rich because it imported luxury goods like silk from China and textiles from India. In 2017, it became the third largest economy in the world. Japanese have a reputation of making high technology products. Japan is doing extremely well in Robotics and Automobile sectors and has a reputation of making high technology products, e.g., ASIMO, world’s most advanced humanoid robot is developed in Japan, other examples of Japanese brands are Maruti Suzuki, Sony, etc.

Question 17.
Discuss the causes of the Kargil war of 1999. [2]
Answer:
The cause of the conflict was the infiltration of Pakistani soldiers and Kashmiri militants into the Kargil district of Jammu and Kashmir and along the Line of Control. The intrusion into the area, that divided the Indian territory of Ladakh from the northern areas of the state, surprised the Indian army and Operation Vijay was launched immediately to flush out the enemies from the Kargil sector.  Tiger Hill, one of the highest peaks in the DrassKargil area of the State, became the focal point during the War. Indian Air Force (IAF) joined this operation and finally, after the fierce War of more than 60 days, India recaptured the Tiger Hill and pushed the Pakistani forces back to their territory.

Question 18.
How terrorism is a non-traditional threat to security? [2]
Answer:
Terrorism is a non-traditional threat to security.
Non-traditional notions of security go beyond military threats to include a wide range of threats and dangers affecting the conditions of human existence. Proponents of non-traditional threats to security say that not just the state but also individuals or communities or indeed all of humankind come under security.

CBSE Sample Papers for Class 12 Political Science Set 2 with Solutions

Section – C (20 Marks)

Question 19.
What was the Green Revolution? Mention two positive and two negative consequences of the Green Revolution. [4]
Answer:
Green Revolution: It was introduced to bring about revolutionary changes in agriculture, especially in food grains like wheat and rice to increase food production through high-yielding varieties of seeds, synthetic fertilizers, and scientific irrigation.
Positive Consequences:

  • It increased food production, especially in wheat, followed by an enhancement in food security.
  • Due to the green revolution, states like UR Punjab and Haryana became agriculturally prosperous.
  • It also resulted in the rise of middle peasant sections.

Negative Consequences:

  • It increased polarisation between the classes and regions.
  • The rich peasants and the large landholders were the major beneficiaries of this process.

Question 20.
Explain the process of establishing democracy in Nepal. [4]
Answer:
Nepal was a Hindu Kingdom and then a constitutional monarchy.

  • In 1990 King accepted the demand for a new democratic Constitution, but the democratic government could not survive long and was in trouble.
  • There was a violent conflict between the Maoist and the armed forces of the king.
  • In 2002, the king abolished dismissed the government and limited democracy in Nepal.
  • In 2006, there was a massive pro-democracy protest led by SPA.
  • King was forced to restore the house of representatives dissolved in 2002.
  • The transition to democracy is still in progress.

Question 21.
Compromise and accommodation are the two essential policies required by states to save planet Earth. Substantiate the statement in light of the ongoing negotiations between the North and the Southern environmental issues. [4]
Answer:
Compromise and accommodation are the two essential policies to save the planet Earth by the states but the states from North and South have different notions towards environmental issues. The Northern states or the First World states are concerned with ozone depletion and global warming whereas Southern States or the Third World states want to address the relationship between economic development and environmental management.

The developed countries of the North want to discuss the environmental issues which stand equally responsible for ecological conservation. The developing countries of the South feel that much of the ecological degradation of the world is created by developed countries through their industrial projects and if developed countries cause more environmental degradation, they are supposed to take more responsibility for it.

Developing countries are in the process of industrialization and they should be exempted from restrictions imposed on developed countries through various conventions like Kyoto Protocol, etc. The special needs of developing countries must be taken into consideration in the process of development, application, and interpretation of International Environmental Law.

Question 22.
State the main issues in Indian politics in the period after 1989. What different configurations of political parties these differences led to? [4]
Answer:
The main issues in Indian politics in the period after 1989 were as follows:
(1) Rise of other Backward Classes as a political force that changed the dynamics of politics in India.
(2) Decision of the National Front government to implement the recommendations of the Mandal Commission and subsequent stir.
(3) Rise of politics based on religious identity and debate about secularism and democracy.
(4) Demolition of Babri Masjid led to rise in communal tensions between the Hindus and Muslims.
(5) Anti-Muslim riots in Gujarat.
(6) End of Congress dominance and beginning of the era of coalition politics.
These issues led to different configurations of political parties as mentioned below:

Political parties accepted the political and social claims of the backward castes. All political parties began to support the reservation of seats for the backward classes in education and employment. This gave rise to an era of coalition politics. Coalition governments were formed, i.e., National Front in 1989, the United Front in 1996 and 1997, NDA in 1997, 1998, and 1999, and the UPA in 2004 and 2009.

CBSE Sample Papers for Class 12 Political Science Set 2 with Solutions

Question 23.
Mention the four main objectives of the ASEAN Economic Community. [4]
Answer:
Objectives of ASEAN Economic Community:

  • To create a common market and production base with ASEAN status.
  • To aid social and economic development in the region.
  • To improve the ASEAN Dispute Settlement Mechanism to resolve economic disputes.
  • To create Free Trade Area (FTA) for investment, labor, and services.

Section – D (12 Marks)

Question 24.

Read the passage given below and answer the questions that follow: [4]

In the context of world politics, what are the common interests of approximately 30 crore indigenous peoples spread throughout the world including India? There are 20 lakh indigenous people of the Cordillera region of the Philippines, 10 lakh Mapuche people of Chile, six lakh tribal people of the Chittagong Hill Tracts in Bangladesh, 35 lakh North American natives, 50,000 Kuna living east of Panama Canal and 10 lakh Small Peoples of the Soviet North.

Like other social movements, indigenous people speak of their struggles, their agenda and their rights. In India, the description ‘indigenous people’ is usually applied to the Scheduled Tribes who constitute nearly 8% of the population of the country. With the exception of small communities of hunters and gatherers, most indigenous populations in India depend for their subsistence primarily on the cultivation of land.

For centuries, if not millennia, they had free access to as much land as they could cultivate. It was only after the establishment of British colonial rule that areas, which had previously been inhabited by the Scheduled Tribe communities, were subjected to outside forces. Although they enjoy constitutional protection in political representation, they have not got many of the benefits of development in the country.

In fact, they have paid a huge cost for development since they are the single largest group among the people displaced by various developmental projects since independence.
(i) In which region of the Philippines, do the indigenous people live?
(A) Java
(B) Sumatra
(C) In the capital city
(D) Cordillera
(ii) What is the description mentioned in the paragraph for indigenous people living in India?
(A) Scheduled Tribes
(B) Untouchables
(C) OBC
(D) None of the above
(iii) What portion of the total population do the indigenous people of India constitute?
(A) 8 per cent
(B) 9 per cent
(C) 10 per cent
(D) 12 per cent
(iv) What main occupation do the indigenous people depend on?
(A) Hunting
(B) Gathering
(C) Weaving
(D) Land cultivation
Answer:
(i) (D) Cordillera
(ii) (A) Scheduled Tribes
(iii) (A) 8 percent
(iv) (D) Land cultivation

Question 25.
In the given outline political map of India, five states have been marked as (A), (B), (C) and (D). All these states were created after 1956. Identify them with the help of the information given below and write their correct names in your answer book along with the serial number of the information used and the concerned alphabet shown in the map as per the following format: [4]
(i) The state came into existence in the year 1966.
(ii) The state was created in the year 1987.
(iii) The state was created in the year 1960.
(iv) The state came into being in the year 2000.

Sr. Number for the
information used
Alphabet concerned Name of the state
(i)
(ii)
(iii)
(iv)

C:\Users\USER1\Desktop\CBSE Sample Papers for Class 12 Political Science Set 2 with Solutions.png

Answer:
(i) (D) Meghalaya
(ii) (A) Mizoram
(iii) (B) Gujarat
(iv) (C) Chhattisgarh

CBSE Sample Papers for Class 12 Political Science Set 2 with Solutions

Question 26.
Study the given Cartoon and answer the questions that follow: [4]

C:\Users\USER1\Desktop\CBSE Sample Papers for Class 12 Political Science Set 2 with Solutions 1.png

(i) When was NITI Aayog formed?
(A) 3rd January 2015
(B) 1st January 2015
(C) 1st January 2016
(D) 1st December 2016
(ii) The CEO and Vice Chairperson of NITI Aayog is appointed by_______
(A) President of India
(B) Members of Parliament
(C) Prime Minister of India
(D) Home minister of India
(iii) How did NITI Aayog serve for India?
(A) Back Bone
(B) Think Tank
(C) Planning Commission
(D) None of the above
(iv) Which councils of NITI Aayog will have to focus on developmental activities in specific areas and is patterned on the National Reforms Development Commission of China?
(A) National Councils
(B) International Councils
(C) Regional Councils
(D) All the above
Answer:
(i) (B) 1st January 2015
(ii) (C) Prime Minister of India
(iii) (B) Think Tank
(iv) (C) Regional Councils

Section – E (24 Marks)

Question 27.
Why was the end of the Second World War considered to be the beginning of the Cold War? Explain.
OR
How did the idea of a New International Economic Order (NIEO) originate? In 1972, which four reforms were proposed by UNCTAD to benefit the Least Developed Countries (LDCs)? [6]
Answer:
Factors responsible for the beginning of the Cold War:

  • The emergence of the USSR as the most powerful country.
  • Use of atom bombs by the USA against Japan.
  • The ideological rivalry between the two superpowers.
  • The two superpowers were keen on expanding their spheres.
  • Use of nuclear power raised many questions and comments like, why did USA drop bombs?
  • Some viewed that the US action was intended to stop the Soviet Union from making political and military gains.
  • It was necessary to end the war quickly.

Detailed Answer:
The Second World War was also the beginning of the Cold War. The World War ended when the United States dropped two atomic bombs on the Japanese cities of Hiroshima and Nagasaki. With the defeat of Germany and Japan there was the devastation of Europe and many other parts of the world.

The United States and the Soviet Union became the greatest powers in the world with the ability to influence events anywhere on earth.
Factors responsible for the beginning of the Cold War:

(1) The USSR emerged as the most powerful country after the Second World War. The Soviet economy at that time was more developed than the rest of the world except the US.

(2) The Second World War ended when the United States of America dropped two atomic bombs on the Japanese cities of Hiroshima and Nagasaki in August 1945, causing Japan to surrender. The US action was intended to stop the Soviet Union from making military and political gains in Asia and elsewhere and to show Moscow that the United States was supreme. There was ideological rivalry between the two superpowers.

The Western Alliance headed by the US represented the ideology of liberal democracy and capitalism while the Eastern Alliance headed by the Soviet Union was committed to the ideology of socialism and communism. The two superpowers were keen on expanding their spheres. This led to alliance systems in the world, especially in Europe. Most countries of Western Europe sided with US and those of eastern Europe joined the Soviet camp.

OR

Since the early 1970s, the issue of the New International Economic Order (NIEO) has been the frontal issue in international relations involving the developing countries on one side and the developed countries on the other side. Developing countries of the third world made persistent demand for the establishment of NIEO.

While the South repeatedly demanded NIEO, the North resisted it. The third world countries regard the restructuring of the existing international economic order as the only way to get out of the present problems related to poverty, scarcity, unemployment, and economic problems. NIEO stands for making the international system fair, just, and equitable by adopting a code of conduct for developed countries and by accepting the due rights of underdeveloped countries.

The United Nations Conference on Trade and Development brought out a report in 1972 entitled Towards a New Trade Policy for Development. The report proposed, a reform in the global trading system as follows:
(1) The developing countries should be given control over natural resources which are exploited.
(2) The least developing countries should be given access to western markets so that they can sell their products and therefore, make trade more useful for the poorer countries.
(3) Reduce the cost of technology of western countries.
(4) LDCs should be provided a role in international economic institutions.

CBSE Sample Papers for Class 12 Political Science Set 2 with Solutions

Question 28.
What is meant by the traditional notion of external security? Describe any two components of this type of security.
OR
Rapid environmental degradation is causing a serious threat to security. Do you agree with the statement? Substantiate your arguments. [6]
Answer:
The traditional notion of external security means a threat to security from outside. In this concept of security, the greatest danger to a country is from military threats. The source of this danger is another country that by threatening military action endangers the core values of sovereignty, independence, and territorial integrity. Following are the three components of the traditional notion of external security.
(A) Deterrence (B) Defence(C) Balance of power

Governments may choose to surrender when actually confronted by war, but they will not advertise this as the policy of the country. Therefore, security policy is concerned with preventing war, which is called deterrence, and with limiting or ending the war, which is called defense.

Prevent the other side from attacking by promising to raise the costs of war to an unacceptable level and to defend itself when war actually breaks out so as to deny the attacking country its objectives and to turn back or defeat the attacking forces altogether.

OR

The given notion of security comes under the non-traditional notions of security. Nontraditional notions of security go beyond military threats to include a wide range of threats and dangers affecting the conditions of human existence.
I agree with the statement that environmental degradation is causing a serious threat to security. Global warming is leading to environmental degradation.

Global warming can cause a rise in sea level of 1.5-2.0 meters. This rise in sea level can submerge 20 percent of Bangladesh, inundate most of the Maldives, and threaten nearly half the population of Thailand. This rise in sea level can pose a serious threat to human security. Environmental degradation is also responsible for aggravating diseases in people. Air pollution is causing a rise in cases of asthma.

Even young people are prone to many kinds of diseases which are related to environmental degradation. Water pollution is another example of environmental degradation. It also has an adverse effect on the life of the people. Many waterborne diseases are posing threats to people. So it is beyond any doubt that rapid environmental degradation is causing a serious threat to security.

Question 29.
Examine any six reasons for the imposition of an Emergency in India in 1975.
OR
Discuss the effects of Emergency on the following aspects of our polity. [6]
(A) Effect on civil liberties for citizens.
(B) Impact on the relationship between the Executive and Judiciary.
(C) Functioning of Mass Media.
(D) Working of Police and Bureaucracy
Answer:
Circumstances that led to imposition of Emergency:

  • Clash between the Executive (Government) and Judiciary.
  • Rising prices and lower growth rate.
  • Student movements in Bihar and Gujarat against price rise and corruption.
  • Railway strike led by George Fernandes.
  • A big rally at Ram Lila Maidan and call to the employees including police/army not to obey the undemocratic orders.
  • Judgement of Allaha bad High Court setting aside the election of Indira Gandhi. All these led to an atmosphere of distrust against Indira Gandhi who hurriedly tried to save her position by imposing emergency

OR

(A) There were mass arrests on the grounds of Preventive Detention. The arrests could not be challenged in any court of law. There were no reasons given for the arrests of the so-called suspects.
(B) There were many new changes incorporated through the 42nd Amendment of the Constitution. The role of the Supreme Court was curbed to some extent. The Court could not challenge the elections of the President of India, Vice President of India, and Prime Minister of India. It also gave the power to the government to a postponement of elections in situations of Emergency.
(C) The powers of media were curtailed by the imposition of press censorship. There was a ban on protests, strikes, agitations, etc. Fundamental Rights were curtailed and suspended. Many eminent Padma Bhushan and Padma Vibhushan awarded and returned their titles. Some prominent newspapers like Indian Express protested by leaving blank spaces at places where news was censored.
(D) It is a combination of restrictions on the civil liberties of the people along with the tension between the government and the bureaucracy.

CBSE Sample Papers for Class 12 Political Science Set 2 with Solutions

Question 30. Examine the main provisions of the Rajiv Gandhi- Longowal Accord of 1985. How far was the Accord successful in bringing back normalcy in Punjab? Explain.
OR
Which three lessons do we learn from regional aspirations and their accommodation as an integral part of democratic politics? Describe. [6]
Answer
After coining to power following the election in 1984, the new Prime Minister Rajiv Gandhi initiated a dialogue with moderate Akali leaders. In July 1985, he reached an agreement with Harchand Singh Longowal, then the President of the Akali Dal. This agreement, known as the Rajiv Gandhi Longowal Accord or the Punjab Accord, was a step towards bringing normalcy to Punjab.

It was agreed that Chandigarh would be transferred to Punjab, a separate commission would be appointed to resolve the border dispute between Punjab and Haryana, and a tribunal would be set up to decide the sharing of Ravi- Beas river water among Punjab, Haryana and Rajasthan. The agreement also provided for compensation to and better treatment of those affected by the militancy in Punjab and the withdrawal of the application of the Armed Forces Special Powers Act in Punjab.

However, peace did not come easily or immediately. The cycle of violence continued nearly for a decade. Militancy and counter-insurgency violence led to excesses by the police and violations of human rights. Politically, it led to the fragmentation of the Akali Dal.

The central government had to impose. President’s rule in the State and the normal electoral and political process was suspended. It was not easy to restore the political process in an atmosphere of suspicion and violence. When elections were held in Punjab in 1992, only 24 percent of the electors tuned out to vote. Militancy was eventually eradicated by the security forces. But the losses incurred by the people of Punjab – Sikhs and Hindus like were enormous.

Peace returned to Punjab by the middle of the 1990s. The alliance of Akali Dal (Badal) and the BJP scored a major victory ini997, in the first normal elections in the State in the post-militancy era. The State is once again preoccupied with questions of economic development and social change. Though religious identities continue to be important for the people, politics has gradually moved back along secular lines.

OR

  • Regional aspirations are very important part of democratic politics and the expression of regional issues is a normal phenomenon.
  • Democratic negotiations are the best way to resolve regional issues.
  • Regional matters can be resolved by power sharing within the constitutional framework.
  • Regional balance and economic development decrease the feeling of regional discrimination. Therefore, the problem of the backwardness of regions should be addressed at a priority.
  • Constitutional provisions are already incorporated in resolving regional issues. As the sixth schedule of the Constitution allows different tribes complete autonomy of preserving their practices and customary laws.
  • Federalism should be given respect in the true sense.
ICSE Physics Question Paper Solved for Class 10

ICSE 2015 Physics Question Paper Solved for Class 10

Solving ICSE Class 10 Physics Previous Year Question Papers ICSE Class 10 Physics Question Paper 2015 is the best way to boost your preparation for the board exams.

ICSE Class 10 Physics Question Paper 2015 Solved

Time : 2 hours
Maximum Marks: 80

General Instructions:

  • Answers to this paper must be written on the paper provided separately.
  • You will not be allowed to write during the first 15 minutes. This time is to be spent in reading the question paper.
  • The time given at the head of paper is the time allotted for writing the answers.
  • Attempt all questions from Section I and any four questions from Section II.
  • The intended marks of questions or parts of questions are given in brackets [ ].

Section – I
(40 Marks)
Attempt all question from this Section.

Question 1
(a) When a body is placed on a table top, it exerts a force equal to its weight downwards on the table top but does not move or fall. [2]
ICSE 2015 Physics Question Paper Solved for Class 10 1
Force due to weight of the body
(i) Name the force exerted by the table top.
(ii) What is the direction of the force?
Answer:
(i) Reaction force.
(ii) Upwards.

ICSE 2015 Physics Question Paper Solved for Class 10

(b) (i) Name one factor that affects the lateral displacement of light as it passes through a rectangular glass slab. [2]
(ii) On reversing the direction of the current in a wire, the magnetic field produced by it gets _________ .
Answer:
(i) Refractive index of glass.
(ii) Reversed.

(c) (i) On what factor does the position of the centre of gravity of a body depend? [2]
(ii) What is the SI unit of the moment of force?
Answer:
(i) Distribution of mass.
(ii) N m.

(d) Name the factors affecting the turning effect of a body. [2]
Answer:
The following are the factors which affect the turning effect of a body:

  1. Magnitude of force.
  2. Distance of point of application of force from the axis of rotation.
  3. Angle between force and distance from axis of rotation.

(e) (i) Define equilibrium. [2]
(ii) In a beam balance when the beam is balanced in a horizontal position, it is in equilibrium.
Answer:
(i) It is a state in which net force acting on a body or system is zero.
(ii) Stable.

Question 2
(a) How is work done by a force measured when the force: [2]
(i) is in the direction of displacement?
(ii) is at an angle to the direction of displacement?
Ans.
(i) W = FS
(ii) W = FS cos θ

(b) State the energy changes in the following while in use: [2]
(i) Burning of a candle.
(ii) A steam engine.
Answer:
(i) Chemical energy into light and heat energy.
(ii) Heat energy into mechanical energy.

ICSE 2015 Physics Question Paper Solved for Class 10

(c) (i) A scissor is a multiplier.
(ii) 1 kWh = _______ J. [2]
Answer:
(i) Velocity.
(ii) 3.6 × 106 J

(d) Explain the motion of a planet around the Sun in a circular path. [2]
Answer:
A planet revolves around the sun due to gravitational force between it and the sun. The gravitational force provides the necessary centripetal force for the circular motion.

(e) Rajan exerts a force of 150 N in pulling a cart at a constant speed of 10 m/s. Calculate the power exerted. [2]
Answer:
P = F × v = 150 × 10 = 1500 W

Question 3
(a) (i) Give the expression for mechanical advantage of an inclined plane in terms of the length of an inclined plane. [2]
(ii) Name a common device where a gear train is used.
Answer:
(i) M.A. = \(\frac{l}{h}\)
(ii) Watch.

(b) The speed of light in glass is 2 × 105 km/s. What is the refractive index of glass? [2]
Answer:
Refractive index µ = \(\frac{c}{v}=\frac{3 \times 10^8}{2 \times 10^8}\) = 1.5

ICSE 2015 Physics Question Paper Solved for Class 10

(c) (i) Draw a graph between displacement and the time for a body executing free vibrations. [2]
(ii) Where can a body execute free vibrations?
Answer:
(i) The graph is as shown :
ICSE 2015 Physics Question Paper Solved for Class 10 1
(ii) In vacuum.

(d) (i) What happens to the resistivity of semi-conductor with the increase of temperature? [2]
(ii) For a fuse, higher the current rating is the fuse wire.
Answer:
(i) It decreases.
(ii) Thicker.

(e) (i) Name the high energetic invisible electromagnetic waves which help in the study of the structure of crystals. [2]
(ii) State an additional use of the waves mentioned in part (i).
Answer:
(i) X-rays.
(ii) Diagnostic tool.

Question 4
(a) Rishi is surprised when he sees water boiling at 115.C in a container. Give reasons as to why water can boil at the above temperature. [2]
Answer:
The boiling point of water depends upon the pressure at which it is boiled. Higher the pressure, higher is the boiling point of water.

(b) (i) Why does a current carrying, freely suspended solenoid rest along a particular direction? [2]
(ii) State the direction in which it rests.
Answer:
(i) This is because when current is passed through the solenoid it begins to act as a magnet.
(ii) It rests in the north-south direction.

ICSE 2015 Physics Question Paper Solved for Class 10

(c) Find the equivalent resistance between points A and B. [2]
ICSE 2015 Physics Question Paper Solved for Class 10 3
Answer:
Resistors of 4 Ω,6 Ω and 12 Ω are in parallel, therefore, equivalent resistance of these three
\(\frac{1}{R}=\frac{1}{4}+\frac{1}{6}+\frac{1}{12}=\frac{12}{24}=\frac{1}{2}\) or R = 2 Ω
Now, R is connected in series with 2 Ω. and 5 Ω resistors, hence equivalent resistance of the combination is R Req = 2 + 5 + 2 = 9 Ω

(d) Give two similarities between an AC generator and a DC motor [2]
Answer:

  1. Both have a concave shaped field magnet.
  2. Both have an armature.

(e) (i) Why is a cathode ray tube evacuated to a low pressure? [2]
(ii) What happens if the negative potential is changed on a grid?
Answer:
(i) So that the electrons emitted do not collide with the
gas molecules.
(ii) The change in negative potential changes the number of electrons striking the screen which in turn changes the brightness of the screen.

Section – II (40 Marks)
Attempt any four questions from this Section.

Question 5
(a) Draw a simplified diagram of a lemon crusher, indicting direction of load and effort. [2|
Answer:
The diagram is as shown :
ICSE 2015 Physics Question Paper Solved for Class 10 2

(b) (i) Name the physical quantity measured in terms of horse power. [4]
(ii) A nut is opened by a wrench of length 20 cm. If the least force required is 2N, find the moment of force needed to loosen the nut.
(iii) Explain briefly why the work done by a fielder when he takes a catch in a cricket match is negative.
Answer:
(i) Power.
(ii) Moment of force = force × length of arm
= 2 × 0.2
= 0.4 N m.
(iii) When a cricketer catches the ball he applies a force opposite to the direction of motion of the ball. Since the displacement of the ball and the force applied are opposite, therefore, the work done is negative.

ICSE 2015 Physics Question Paper Solved for Class 10

(c) A block and tackle system has V.R. = 5. [4]
(i) Draw a neat labelled diagram of a system indicating the direction of its load and effort.
(ii) Rohan exerts a pull of 150 kgf. What is the maximum load he can raise with this pulley system if its efficiency = 75%?
Answer:
(i) The diagram is as shown :
ICSE 2015 Physics Question Paper Solved for Class 10 3
(ii) η = \(\frac{\text { M.A. }}{\text { V.R. }}=\frac{L}{E n}\)
where n is the number of pulleys.
or 0.75 = \(\frac{\mathrm{L}}{150 \times 5}\)
or L= 0.75 × 150 × 5
or L = 562.5 kgf

Question 6
(a) (i) Where should an object be placed so that a real and inverted image of the same size as the object is obtained using a convex lens?
(ii) Draw a ray diagram to show the formation of the image as specified in the part a(i).
Answer:
(i) At 2F.
(ii) The ray diagram is as shown :
ICSE 2015 Physics Question Paper Solved for Class 10 4

(b) (i) Why does the Sun appear red at sunrise? [3]
(ii) Name the subjective property of light related to its wavelength.
Answer:
(i) Light from the Sun, near the horizon, passes through thicker layers of air and covers a large distance in the earth’s atmosphere before reaching our eyes.
ICSE 2015 Physics Question Paper Solved for Class 10 5
However, light from the Sun overhead would travel relatively shorter distance. At noon, the Sun appears white as only a little of the blue and violet colours are scattered. Near the horizon, most of the blue light and shorter wavelengths are scattered away by the particles. Therefore, the light that reaches our eyes is of longer wavelengths. This gives rise to the reddish appearance to the Sun.

(ii) Colour.

ICSE 2015 Physics Question Paper Solved for Class 10

(c) Jatin puts a pencil into a glass container having water and is surprised o see the pencil in a different state. [4]
(i) What change is observed in the appearance of the pencil?
(ii) Name the phenomenon responsible for the change.
(iii) Draw a ray diagram showing how the eye sees the pencil.
Answer:
(i) The pencil seems bent at the water-air interface.
(ii) Refraction of light.
(iii) The ray diagram is as shown :
ICSE 2015 Physics Question Paper Solved for Class 10 6

Question 7
(a) (i) State the safe limit of sound level in terms of decibel for human hearing. [2]
(ii) Name the characteristic of sound in relation to its waveform.
Answer:
(i) 120 dB.
(ii) Timbre.

(b) A person standing between two vertical cliffs and 480 m from the nearest cliff shouts. He hears the First echo after 3s and the second echo 2s later. Calculate : [3]
(i) The speed of sound.
(ii) The distance of the other cliff from the person.
Answer:
(i) The first echo is heard from the nearest cliff.
Let d1 be the distance of the nearest cliff. Then distance travelled by sound in going and coming back from the nearest cliff is
S1 = 2d1= 480 × 2 = 960 m
Time interval t1 = 3 s
Therefore, Speed of sound is
v = \(\frac{\text { Total distance of travelled }}{\text { Time taken }}\)
= \(\frac{960}{3}\) = 320 m s-1
(ii) The second echo is heard from the farther cliff. If d2 is the distance from the person, then distance travelled by sound
S2 = 2d2 and time taken t2 = 3 +2 = 5s
Now, v = \(\frac{\mathrm{S}_2}{t_2}\)
⇒ 320 = \(\frac{S_2}{5}\) ms-1
Therefore, S2 = 320 × 5 = 1600 m

(c) In the diagram below, A, B, C, D are four pendulums suspended from the same elastic string PQ. The length of A and C are equal to each other while the length of pendulum B is smaller than that of D. Pendulum A is set in to a mode of vibrations. [5]
ICSE 2015 Physics Question Paper Solved for Class 10 8
(i) Name the type of vibrations taking place in pendulums B and D?
(ii) What is the state of pendulum C?
(iii) State the reason for the type of vibrations in pendulum B and C.
Answer:
(i) Force vibrations.
(ii) Pendulum C, after sometime, will start vibrating with the same amplitude as that of pendulum A.
(iii) The type of vibrations in pendulum B and C are called forced vibrations. In case of pendulum C it comes into a state of resonance and vibrates in phase with pendulum A while pendulum B does not.

ICSE 2015 Physics Question Paper Solved for Class 10

Question 8.
(a) (i) Name the device used to increase the voltage at a generating station.
(ii) At what frequency is AC supplied to residential houses?
(iii) Name the wire in a household electrical circuit to which the switch is connected. [3]
Answer:
(i) Transformer.
(ii) 50 Hz.
(iii) Live wire.

(b) The relationship between the potential difference and the current in a conductor is stated in the form of a law. [3]
(i) Name the law.
(ii) What does the slope of V-I graph for a conductor represent?
(iii) Name the material used for making the connecting wire.
Answer:
(i) Ohm’s law.
(ii) Resistance
(iii) Copper.

(c) A cell of Emf 2 V and internal resistance 1.2 U is connected with an ammeter of resistance 0.8 U and two resistors of 4.5 U and 9 U as shown in the diagram below: [4]
ICSE 2015 Physics Question Paper Solved for Class 10 9
(i) What would be the reading on the Ammeter?
(ii) What is the potential difference across the terminals of the cell?
Answer:
(i) Here, 4.5 Ω and 9 Ω resistors are in parallel, therefore, equivalent resistance is
\(\frac{1}{R}=\frac{1}{45}+\frac{1}{9}\)
\(\frac{10}{45}+\frac{1}{9}=\frac{10+5}{45}=\frac{15}{45}=\frac{1}{3}\)
or R = 3 ohm
Now, ammeter and the cell are connected in series with this resistance, hence
Total resistance of the circuit is
Rnet = 3 + 0.8 + 1.2 = 5 ohm
Hence, current through the ammeter and its reading is
I = \(\frac{\mathrm{V}}{\mathrm{R}_{\text {net }}}=\frac{2}{5}\) = 0.4 A

(ii) V = E – Ir
= 2 – 0.4 × 1.2
= 2 – 0.48
= 1.52 V

ICSE 2015 Physics Question Paper Solved for Class 10

Question 9
(a) (i) Name a gas caused by the Greenhouse effect. [2]
(ii) Which property of water makes it an effective coolant?
Answer:
(i) Carbon dioxide.
(ii) Unusally high specific heat capacity.

(b) (i) Water in lakes and ponds do not freeze at once in cold countries. Give a reason is support of your answer.
(ii) What is the principle of Calorimetry?
(iii) Name the law on which this principle is based.
(iv) State the effect of an increase of impurities on the melting point of ice.
Answer:
(i) In the beginning of winter when the atmospheric temperature drops to ft °C, the water in ponds and lakes do not freeze all at once due to the high latent heat of water. Every kilogram of water when frozen releases 333 kJ of heat energy to the atmosphere warming up the atmosphere. It takes more severe winter (large drop in temperature) and hence, more time for water to freeze. So it happens slowly and gradually.

(ii) “Heat lost by the hot body is equal to the heat gained by the cold body”.
(iii) Law of conservation of energy.
(iv) Impurities decrease the melting point of a solid.

(c) A refrigerator converts 100 g of water at 20°C to ice at -10°C in 35 minutes.
Calculate the average rate of heat extraction in terms of watts.
Given: Specific heat capacity of ice = 2.1 J g-1. C-1
Specific heat capacity of water = 4.2 J g-1.C-1
Specific latent heat of fusion of ice = 336 J g-1 [4]
Answer:
Given : mass of water = 100 g
Heat extracted to convert 100 g water at 20°C to water at 0°C
= Mass x Specific heat capacity of water × Change in temperature
= 100 × 4.2 × (20 – 0) = 8400 J
Heat extracted to convert 100 g water at 0°C to ice at 0°C
= Mass × Specific heat capacity of water × Rise in temperature
= 100 × 336 = 33600 J
Heat extracted to convert 100 g ice at 0°C to ice at-10°C
= Mass × Specific heat Capacity of ice × Change in temperature
= 100 × 2.1 × (0- (-10)) = 2100 J
Therefore,
Total heat extracted in 35 minutes = 8400 + 33600 + 2100 = 44100 J
Therefore, average rate of heat extraction is
R = \(\frac{44100}{35 \times 60}\)
= 21 J s-1 = 21 watt

Question 10
(a) (i) What is thermionic emission? [2]
(ii) Name the unit in which the work function of a metal is expressed.
ICSE 2015 Physics Question Paper Solved for Class 10 10
Answer:
(i) The emission of electrons from a metal surface on heating is called thermionic emission.
(ii) Electron volt (eV).
ICSE 2015 Physics Question Paper Solved for Class 10 7

ICSE 2015 Physics Question Paper Solved for Class 10

(b) (i) Complete the diagram as given below by drawing the deflection of radioactive radiations in an electric filed. [5]
(ii) State any two precautions to be taken while handling radioactive substances.
Answer:
(i) The completed diagram is as shown alongside.

(ii) A person handling radioactive sources should wear protective clothing and special badge (of photographic emulsion) to monitor the extent of exposure to harmful radiation.

The sources should be handled by the forceps provided. The user should wear rubber gloves. Hands should be washed and even a shower taken after each use.

(c) An atomic nucleus A is composed of 84 protons and 128 neutrons. [3]
(i) The nucleus A emits an alpha particle and is transformed into nucleus B. What is the composition of nucleus B?
(ii) The nucleus B emits a beta particle and is transformed into nucleus C. What is the composition of nucleus C?
(iii) Does the composition of nucleus C change if it emits gamma radiations?
Answer:
(i) The composition of nucleus B is 82 protons and 124 neutrons.
(ii) The composition of nucleus C is 83 protons and 124 neutrons.
(iii) No.

CBSE Sample Papers for Class 12 Informatics Practices with Solutions

CBSE Sample Papers for Class 12 Informatics Practices with Solutions 2022-2023

Solved CBSE Sample Paper Class 12 Informatics Practices 2022-2023 with Solutions: Solving Pre Board CBSE Sample Papers for Class 12 Informatics Practices with Solutions Answers 2022-2023 Pdf Download helps to understand the pattern of questions asked in the board exam. Know about the important concepts to be prepared for CBSE Class 12 Informatics Practices board exam and Score More marks. Here we have given CBSE Class 12 Informatics Practices Sample Papers 2023.

Board – Central Board of Secondary Education (CBSE)
Subject – CBSE Class 12 Informatics Practices
Year of Examination – 2022-2023

CBSE Sample Paper Class 12 Informatics Practices 2022-2023 with Solutions

According to the new CBSE Exam Pattern, MCQ Questions for Class 12 Informatics Practices Carry 20 Marks. Click on the link below to access the CBSE Class 12 Informatics Practices Sample Paper 2023 Solved.

CBSE Class 12 Informatics Practices Sample Papers 2023 with Solutions

Informatics Practices Sample Paper Class 12 Question Paper Design 2022-23

Divisions Typologies of Questions No. of Questions Marks Total
Section A MCQs 16 1 × 16 = 16 18 Marks
Assertion & Reasons 2 1 × 2 = 2
Section B Very Short Answer Questions 7 2 × 7 = 14 14 Marks
Section C Short Answer Questions 5 3 × 5 = 15 15 Marks
Section D Long Answer Questions 3 5 × 3 = 15 15 Marks
Section E Case-based Questions 2 4 × 2 = 8 8 Marks
Total 35 Questions 70 Marks

We hope these CBSE Sample Papers for Class 12 Informatics Practices with Solutions 2022-2023 will help in self-evaluation. Stay tuned for further updates on the CBSE Sample Paper of Informatics Practices Class 12 2023 for their exam preparation.

Recommended Reading On: Computer MCQ Questions for Class 3 with Answers

CBSE Sample Papers for Class 10 Sanskrit Set 9 with Solutions

CBSE Sample Papers for Class 10 Sanskrit Set 9 with Solutions

Students must start practicing the questions from CBSE Sample Papers for Class 10 Sanskrit with Solutions Set 9 are designed as per the revised syllabus.

CBSE Sample Papers for Class 10 Sanskrit Set 9 with Solutions

अवधि: : होरात्रयम्
पूर्णाङ्काः : 80

सामान्य निर्देशः
सैंपल पेपर 1 में दिये गये निर्देशानुसार।

रवण्ड: ‘क’ अपठित-अवबोधनम्
(अंक : 10)

1. अधोलिखितं गद्यांशं पठित्वा प्रदत्तप्रश्नानाम् उत्तराणि संस्कृतेन लिखत (10)
(निम्नलिखित गद्यांश को पढ़कर दिए गए प्रश्नों के उत्तर संस्कृत में लिखिए।)

रामायणं लौकिकसंस्कृतस्य प्रथमः ग्रन्थः मन्यते। अस्मिन् ग्रन्थे मर्यादा पुरुषोत्तमस्य रामचन्द्रस्य जीवनस्य वृत्तान्तं वर्तते। अस्य लेखक : महर्षि वाल्मीकिः आसीत्। सः आदिकविः अपि कथ्यते। तस्य रचना ‘आदिकाव्यम्’ इति विद्वांसः मन्यन्ते। इदं काव्यम् अनुष्टुप् छन्दसि निबद्धम् अस्ति। अस्मिन् (ग्रंथे) महाकाव्ये चतुर्विशतिसहस्र-श्लोकाः सन्ति। अनेन रामायणं चतुर्विंशति-साहस्री संहिता’ इति नाम्ना अपि प्रसिद्धम् अस्ति। अस्य भाषा अति सरला मधुरा च अस्ति। रामायणे कविना यद् वर्णनं कृतम् तत् सर्वत्रं अलौकिकम् अस्ति । महर्षि वाल्मीकिः श्री रामस्य वर्णनम् अवताररूपे न कृत्वा केवलं मर्यादा पुरुषोत्तम रूपे कृतवान् अस्ति।

(अ) एकपदेन उत्तरत-(केवलं प्रश्नद्वयम्) (1 × 2 = 2)
(एक शब्द में उत्तर लिखिए) (केवल दो प्रश्न)

(i) लौकिकसाहित्यस्य प्रथमः ग्रन्थः किम् अस्ति ?
(ii) आदिकविः कः आसीत् ?
(i) रामायणे कति श्लोकाः सन्ति?
उत्तर:
(i) रामायणम्
(ii) वाल्मीकि:
(iii) चतुर्विशति-सहस्रं

(आ) पूर्णवाक्येन उत्तरत-(केवलं प्रश्न-द्वयम्) (2 × 2 = 4)
(पूर्णवाक्य में उत्तर लिखिए) (केवल दो प्रश्न)

(i) रामायणे कस्य जीवनस्य -वृत्तान्तं वर्तते?
(ii) रामायणं केन नाम्ना प्रसिद्धम् अस्ति?
(iii) अयं ग्रन्थः कस्मिन् छन्दसि निबद्धः अस्ति?
उत्तर:
(i) रामायणे मर्यादा पुरुषोत्तमस्य श्रीरामस्य जीवनस्य वृतान्तं वर्तते।
(ii) रामायणं चतुर्विशति साहस्री साहिता नाम्ना प्रसिद्धम् अस्ति।
(iii) अयं ग्रन्थ: अनुष्टुप् छन्दसि निबद्धः अस्ति।

CBSE Sample Papers for Class 10 Sanskrit Set 9 with Solutions

(इ) अस्य अनुच्छेदस्य कृते उपयुक्तं शीर्षकं संस्कृतेन लिखत। (द्वि-त्रिशब्दात्मकं)1
(इस अनुच्छेद के लिए उपयुक्त शीर्षक संस्कृत में लिखिए।) (दो-तीन शब्द का वाक्य)
उत्तर:
शीर्षाकं आदिकाव्यं-रामायणम्। मर्यादापुरुषोत्तम रामस्य जीवनी।

(ई) यथानिर्देशम् उत्तरत-(केवलं प्रश्नत्रयम्) (1 × 3 = 3)
(निर्देशानुसार उत्तर दीजिए) (केवल तीन प्रश्न)

(i) मर्यादा पुरुषोत्तमस्य रामचन्द्रस्य जीवनस्य वृत्तान्तं कुत्र वर्तते?
(क) महाभारते
(ख) गीतायां
(ग) रामायणे
(घ) नीतिशतके
उत्तर:
(ग) रामायणे

(ii) आदिकविः कः कथ्यते?
(क) कालिंदासः
(ख) वाल्मीकिः
(ग) भवभूतिः
(घ) भारविः
उत्तर:
(ख) वाल्मीकिः

(iii) ‘आदिकाव्यम्’ इति विद्वान्सः मन्यन्ते इत्यत्र मन्यन्ते इति क्रियापदस्य किं कर्तृपदम्?
(क) इति
(ख) विद्वान्सः
(ग) तस्य
(घ) आदिकाव्यम्
उत्तर:
(ख) विद्वान्सः

(iv) ‘पुरुषोत्तमस्य’ इति पदस्य विशेष्यं गद्यांशात् चित्वा लिखत।
(क) रामचन्द्रस्य
(ख) रामायण:
(ग) वाल्मीके:
(घ) मर्यादा
उत्तर:
(क) रामचन्द्रस्य

रवण्ड: ‘ रव’ रचनात्मक-कार्यम्
(अंक : 15)

2. भवतः नाम मोहनः। भवतः विद्यालये वार्षिकोत्सवे ‘अभिज्ञान शाकुन्तलम्’ संस्कृत-नाटकस्य मञ्चनं भविष्यति। तदर्थं स्वमित्रं सोहनं प्रतिलिखितं निमन्त्रणपत्रं मञ्जूषा पद सहायतया पूरयित्वा पुनः लिखत (1/2 × 10 = 5)
(आपका नाम मोहन है। आपके विद्यालय में वार्षिकोत्सव पर अभिज्ञानशाकुन्तलम् संस्कृत नाटक का मञ्चन होगा। उसके लिए अपने मित्र सोहन को लिखे गये निमन्त्रण-पत्र को मञ्जूषा पदों की सहायता से पूरा करके पुनः लिखिए।) नवोदय विद्यालयः
(i)………..
दिनाङ्कः
(ii)………..
नमोनमः।
अत्र कुशलं (iii)………… अस्माकं विद्यालये आगामी सप्ताहे विद्यालयस्य वार्षिकोत्सवस्य आयोजनं भविष्यति। तत्र ‘अभिज्ञानशाकुन्तलम्’ इति नाटकस्य (iv)…….. भविष्यति। अहं तस्मिन् नाटके (v)……करिष्यामि। अस्य कार्यक्रमस्य मुख्यातिथि: (vi)…… भविष्यति। भवान् अवश्यमेव तत् (Vii)……………आगच्छतु। मम (viii)……….भविष्यति। सर्वेभ्यः अग्रजेभ्य मम (ix)….. निवेद्यताम् इति ।
भवदीय: वयस्यः
(x).
उत्तर:
(i) मुम्बईतः
(ii) सोहन
(iii) तत्रास्तु
(iv) मञ्चनम्
(v) अभिनयं
(vi) शिक्षानिदेशक:
(vii) द्रष्टुम्
(viii) उत्साहवर्धनम्
(ix) प्रणामाञ्जलि:
(x) मोहन:।

मञ्जूषा-प्रणामाञ्जलिः, मोहनः, उत्साहवर्धनम्, तत्रास्तु, शिक्षानिदेशकः, सोहन, मुम्बईतः, मञ्चनम्, अभिनयं, द्रष्टुम्।

CBSE Sample Papers for Class 10 Sanskrit Set 9 with Solutions

3. अधः प्रदत्तं चित्रं दृष्ट्वा मञ्जूषायां प्रदत्तशब्दानां साहाय्येन पञ्च वाक्यानि संस्कृतेन लिखत (1 × 5 = 5)
(नीचे दिए गए चित्र को देखकर मंजूषा में दिए गए शब्दों की सहायता से पाँच वाक्य संस्कृत में लिखिए।)
CBSE Sample Papers for Class 10 Sanskrit Set 9 with Solutions Img 1
उत्तर:
(i) अस्मिन् चित्रे विद्यालयस्य प्रार्थना सभा अस्ति।
(ii) प्रार्थनासभायां प्रधानाचार्य: अपि प्रार्थनां करोति।
(iii) अस्मिन् चित्रे दश बालका: बालिका: च तिष्ठन्ति।
(iv) ते सर्वे छात्राः प्रातःकाले ईश्वरस्य प्रार्थनां कुर्वन्ति।
(v) विद्यालयस्य भवनं अति सुन्दरम् अस्ति।

मञ्जूषा-विद्यालयः, प्रधानाचार्यः, छात्राः, प्रार्थनासभा, प्रात:काले, कुर्वन्ति, सुन्दर भवनम्।

अथवा
मञ्जूषायां प्रदत्तशब्दानां साहाय्येन निम्नलिखितं विषयम् अधिकृत्य पञ्चभिः संस्कृतवाक्यैः एकम् अनुच्छेदं लिखत। (मञ्जूषा में दिए गए शब्दों की सहायता से निम्नलिखित विषय पर पाँच संस्कृत वाक्यों में एक अनुच्छेद लिखिए।)
“मम मातृभूमिः”
उत्तर:
(i) अस्माकं मातृभूमि: नाम “भारत” अस्ति।
(ii) अस्माकं मातृभूमिः जनानां भरणं-पोषणं कुर्वन्ति।
(iii) “स्वर्गादपि गरीयसी” मातृभूमि: भवति।
(iv) वयं सर्वदा येन-केन प्रकारेण मातृभूमिं रक्षिष्यामः।
(v) “अर्हनिशं” सत्यं अस्माभिः इयं माता वन्दनीया।

मञ्जूषा-पोषणम्, स्वर्गादपि, येन-केन प्रकारेण, सत्यम्, मातृभूमिः, वन्दनीया।

4. अधोलिखितानि वाक्यानि संस्कृतभाषया अनूद्य, लिखत-(केवलं पञ्चवाक्यम्, रचनात्मक) (1 × 5 = 5)
(निम्नलिखित वाक्यों का संस्कृत भाषा में अनुवाद लिखिए) (केवल पाँच वाक्य)

(i) मैं पिताजी के साथ घर जाता हूँ।
I go home with my father.
(ii) मित्र पत्र लिखते हैं।
Friends write a letter
(iii) विद्यालय के चारों ओर फूल हैं।
There are flowers around the school
(iv) सीता गीत गाती है।
Seeta sings a song.
(v) कल तुम कहाँ जाओगे?
Where will you go tomorrow?
(vi) बच्चे गेंद से खेलते हैं।
Children play with ball.
(vii) मेरी माँ एक परिश्रमी महिला है।
My mother is a hard-working woman
उत्तर:
(i) अहं जनकेन सह गृहं गच्छामि।
(ii) मित्राणि पत्राणि लिखन्ति।
(iii) विद्यालयम् परितः पुष्पाणि सन्ति।
(iv) सीता गीतं गायति।
(v) श्व: त्वम् कुत्र गमिष्यसि ?
(vi) बालका: कन्दुकेन क्रीडन्ति।
(vii) मम जननी एका उद्यमिनी महिला अस्ति।

रवण्ड: – ‘ग’ (अनुप्रयुक्त-व्याकरणम्)
(अंक : 25)

5. अधोलिखितवाक्येषु रेखाङ्कितपदानां सन्धिं सन्धिच्छेदं वा कुरुत (केवलं प्रश्नचतुष्ट्यम्) (1 × 4 = 4)
(निम्नलिखित वाक्यों में रेखांकित पदों के संधि अथवा संधि-विच्छेद कौजिए। (केवल चार प्रश्न)

(i) षण्णाम् मासानां समूहः एव अयनम् ।
(ii) सं + गच्छ एव जीवनस्य आदर्शम्।
(iii) तपस्ते गीतम् अगायन्।
(iv) कुशः + वादने भविष्यति ।
(v) सर्वथा जागरुकः + अहम् छात्राणां कृते आदर्श: एव।
(क) जागरुकोम्
(ख) जागरूकोऽम्
(ग) जागरुकोऽम्
उत्तर:
(i) षट् + नाम्
(ii) सक्जच्छ
(iii) तप: $+$ ते
(iv) कुशो वादने
(v) जागरूकोsहम्

CBSE Sample Papers for Class 10 Sanskrit Set 9 with Solutions

6. अधोलिखितेषु वाक्येषु रेखाङ्कितपदानां समासं विग्रहं वा प्रदत्तविकल्पेभ्यः चित्वा लिखत-(केवलं प्रश्नचतुष्ट्य म्) (1 × 4 = 4)
(निम्नलिखित वाक्यों में रेखांकित पदों का समास अथवा विग्रह दिए गए विकल्पों में से चुनकर लिखिए।) (केवल चार प्रश्न)

(i) यथासमयम् कार्य कुरु।
(क) समयम् अनतिक्रम्य
(ख) समयम् अतिक्रम्य
(ग) समयस्य अतिक्रम्य
(घ) समयेन अतिक्रम्य
उत्तर:
(क) समयम् अनतिक्रम्य

(ii) अहम् जले स्थितप्रज्ञः इव तिष्ठामि।
(क) स्थितः प्रज्ञः यस्य सः
(ख) स्थिता प्रज्ञा यस्य सः
(ग) स्थिता प्रज्ञा यस्याः सा
(घ) स्थितप्रज्ञा भूत्वा
उत्तर:
(ख) स्थिता प्रज्ञा यस्य सः

(iii) स; मातरं पितरं च वन्दते।
(क) मातपितरौं
(ख) माता पितरौ
(ग) माता पितरम्
(घ) माता पितां च
उत्तर:
(क) मातपितरौं

(iv) छात्राः अहर्निशं कार्यं कुर्वन्ति ।
(क) अहः च निश: च
(ख) अहम च निशा च
(ग) अहःच निशा च
(घ) अह: निशा च
उत्तर:
(ग) अहःच निशा च

(v) नास्ति विद्यासमं चक्षुः।
(क) विद्यया समम्
(ख) विद्यया समः
(ग) विद्यायाः समम्
(घ) विद्या समः
उत्तर:
(क) विद्यया समम्

7. अधोलिखितेषु वाक्येषु रेखाङ्कितपदानां प्रकृति-प्रत्ययौ संयोज्य विभज्य वा उचितम् उत्तरं विकल्पेभ्यः चित्वा लिखत- (केवलं प्रश्नचतुष्ट्यम्) (1 × 4 = 4)
(निम्नलिखित वाक्यों में रेखांकित पदों के प्रकृति-प्रत्ययों को जोड़कर अथवा अलग करके उचित उत्तर विकल्पों में से चुनकर लिखिए।) (केवल चार प्रश्न)

(i) सर्वेषाम् एव महत्त्वं भवति।
(क) महत् + त्व
(ख) महान् + त्वम्
(ग) महत् + तु
(घ) महत् + तल्
उत्तर:
(क) महत् + त्व

(ii) अलोक + इक् उन्नतिः विद्यया एव भवति।
(क) अलौकिकी
(ख) अलौकिका
(ग) आलोकिकी
(घ) अलौकिक:
उत्तर:
(क) अलौकिकी

(iii) प्रकृतेः रमणीय + तल् दर्शनीया अस्ति।
(क) रमणीयता
(ख) रमणीयताम्
(ग) रमणीया
(घ) रमणीयम्
उत्तर:
(क) रमणीयता

(iv) अस्याः अनुज + टाप् दीपिका अस्ति।
(क) अनुजा
(ख) अनुज्ञता
(ग) अनुजः
(घ) अनुजे
उत्तर:
(क) अनुजा

(v) अद्य अस्माकं वर्ष + इक् + ङीप् परीक्षा आरभते।
(क) वार्षिकी
(ख) वर्षकी
(ग) वर्षिकी
(घ) वार्षिक
उत्तर:
(क) वार्षिकी

8. वाच्यानुसारम् उचितपदैः रिक्तस्थानानि पयित्वा अधोलिखितं पूरयित्वा अधोलिखितं संवादं पुनः लिखत (केवलं प्रश्नत्रयम्) (1 × 3 = 3)
(वाक्य के अनुसार उचित पदों से रिक्त स्थानों को पूरा करके नीचे लिखे हुए संवाद) (केवल तीन प्रश्न)

(i) सीता गीतां कथयति-किं त्वया इंद गीतं………..?
(क) गायति
(ख) गेयते
(ग) गीयते
(ख) पठ्यते
उत्तर:
(ग) गीयते

(ii) गीता-अहं गीतं न………………..।
(क) गायामि
(ख) गायति
(ग) गायसि
(घ) गीयते
उत्तर:
(क) गायामि

(iii) सीता पुनः पृच्छति – सरलतया कि…………..?
(क) करोति
(ख) क्रियते
(ग) गच्छति
(ग) गीयते
उत्तर:
(ख) क्रियते

(iv) गीता-सा पुस्तकं………………….।
(क) पठ्यते
(ख) लिखति
(ग) गच्छति
(घ) पठति
उत्तर:
(घ) पठति

CBSE Sample Papers for Class 10 Sanskrit Set 9 with Solutions

9. अधोलिखितं दिनचर्चायां रिक्तस्थानानि कालबोधकशब्दैः पूरयत लिखत। (केवलं प्रश्नचतुष्टयम्) (1 × 4 = 4)
(कालबोधक शब्दों से निम्नलिखित दिनचर्या को पूरा कीजिए।) (केवल चार प्रश्न)

(i) प्रात: 7:00-वादने प्रार्थना सभा भवति ।
(ii) प्रातः 10.00-वादने अविकाशः भवति।
(iii) मध्यान्हे 1.15-वादने विविधाः क्रीडाः भवन्ति ।
(iv) अहं 2:30-वादने गृहं गच्छामि।
(v) त्वं 5.00-वादने विद्यालयं गच्छंसि।
उत्तर:
(i) सप्त
(ii) दश
(iii) सपाद एक
(iv) सार्ध द्वि
(v) पर्च।

10. मञ्जूषायां प्रदत्तैः उचितैः अव्ययपदैः अधोलिखितेषु वाक्येषु रिक्तस्थानानि पूरयत- (केवलं प्रश्नत्रयम्) (1 × 3 = 3)
(मञ्जूषा में दिये गये उचित अव्ययपदों से वाक्यों में रिक्तस्थानों की पूर्ति कीजिए) (केवल तीन प्रश्न)

(i) जनाः ………………. धावन्ति।
(ii) नेता स्वभाषणे …………….. वदति।
(iii) वर्षायाः अनन्तरं ……………….. अन्धकारम् अभवत्।
(iv) सर्वे जनाः धावन्ति अत: त्वम् ………………. धाव।
उत्तर:
(i) (ख) इतस्तत:
(ii) (ख) उच्चै:
(iii) (क) सहसा
(iv) (ग) अपि।

11. अधोलिखितेषु वाक्येषु रेखाङ्कित-अशुद्धपदाय उचितं पदं चित्वा वाक्यानि पुनः लिखत- (केवलं प्रश्नत्रयम्) (1 × 3 = 3)
(निम्नलिखित वाक्यों में रेखाङ्कित अशुद्ध पद के लिए उचितं पद चुनकर वाक्यों को पुनः लिखिये) (केवल तीन प्रश्न)

(i) जनाः फलं खादतः
(क) खादसि
(ख) खादन्ति
(ग) खादतु
(घ) खादति
उत्तर:
(ख) खादन्ति

(ii) बालकाः पुस्तकं अपठत्।
(क) त्वम्
(ख) स:
(ग) अहम्
(घ) वयम्
उत्तर:
(ख) स:

(iii) अहं सत्यं वदसि।
(क) स:
(ख) त्वं
(ग) त्वम्
(घ) ते
उत्तर:
(ग) त्वम्

(iv) छात्राः कन्दुकेन क्रीडिस्यथ
(क) क्रीडिष्यन्ति
(ख) क्रीडथ
(ग) क्रीडामि
(घ) क्रिडन्ति
उत्तर:
(क) क्रीडिष्यन्ति

रवण्ड: – ‘घ’ पठित-अवबोधनम्
(अंक : 30)

12. अधोलिखितं गद्यांशं पठित्वा प्रदत्तप्रश्नानाम् उत्तराणि संस्कृतेन लिखत
(निम्नलिखित गद्यांश को पढ़कर दिए गए प्रश्नों के उत्तर संस्कृत में लिखिए।)

“बहून्यपत्यानि मे सन्तीति सत्यम्। तथाप्यहमेस्मिन् पुत्रे विशिष्य आत्मवेदनामनुभवामि। यतोहि अयमन्येभ्यो दुर्बलः। सर्वेष्वपत्येषु जननी तुल्यवत्सला एव। तथापि दुर्बले सुते मातुः अत्यधिका कृपा सहजैव इति। सुरभिवचनं श्रुत्वा भृशं विस्मितस्याखण्डलस्यापि हृदयमद्रवत्। सः च तामेवमसान्त्वयत्-“गच्छ वत्से। सर्व भद्रं जायेत।” अचिरादेव चण्डवातेन मेघरवैश्च सह प्रवर्षः समजायत्। पश्यतः एव सर्वत्र जलोपप्लवः सञ्जातः। कृषक: हर्षातिरेकेण कर्षणाविमुखः सन् वृषभौ नीत्वा गृहमगात्

(अ) एकपदेन उत्तरत-(केवलं प्रश्नद्वयम्) (1/2 × 2 = 1)
एक शब्द में उत्तर लिखिए (केवल दो प्रश्न)

(क) सर्वत्र कः सञ्जातः?
(ख) बहूनि अपत्यानि कस्याः सन्ति ?
(ग) को नीत्वा गृहम् आगतः?
उत्तर:
(क) जलोपप्लव:
(ख) मे (सुरभेः),
(ग) वृषभौ।

(आ) पूर्णवाक्येन उत्तरत-(केवलम् प्रश्नद्वयम्) (1 × 2 = 2)
पूर्ण वाक्य के उत्तर लिखिए (केवल दो प्रश्न)

(क) सुरभिः आखण्डलं किमुवाच?
(ख) कस्मिन् एव सुरभेः वात्सल्यम् अधिकम् आसीत्?
(ग) कृषक: को नीत्वा गृहं आगतः?
उत्तर:
(क) सुरभिः उवाच-बहून्यपत्यानि आखण्डलं सहजैव इति।
(ख) दुर्बले सुते एव सुरभे: वात्सल्यम् अधिकम् आसीत् ?
(ग) कृषक: वृषभौः नीत्वा सायं गृहं आगतः।

(इ) निर्देशानुसार उत्तरत। (केवलं प्रश्नद्वयम्) (1 × 2 = 2)
(निर्देशानुसार उत्तर लिखिए) (केवल दो प्रश्न)

(क) “बहून्यपत्यानि मे सन्ति इति सत्यम्” असत्यम् इति पदस्य किं विलोमपदं प्रयुक्तम? किं विलोमपदं प्रयुक्तम?
(ख) पश्यतः एव सर्वत्र जलोपप्लवः सञ्जात् इत्यत्र किं क्रियापद प्रयुक्तम्?
(ग) ‘सर्वेषु अपत्येषु जननी तुल्यवत्सला एवं इत्यत्र किं विशेष्यपदं प्रयुक्तम्?
उत्तर:
(क) सत्यम्
(ख) सञ्जात:
(ग) अपत्येषु

13. अधोलिखितं पद्यांशं पठित्वा प्रदत्तप्रश्नानाम् उत्तराणि संस्कृतेन लिखत (05)
(निम्नलिखित पद्यांश को पढ़कर दिए गए प्रश्नों के उत्तर संस्कृत में लिखिए।)

वायुमण्डलं भृशं दूषितं न हि निर्मलं जलम्।
कुत्सितवस्तुमिश्रितं भक्ष्यं समलं धरातलम्॥
करणीयं बहिरन्तर्जगति तु बहु शुद्धीकरणम्।
शुचि…….. ||

(अ) एकपदेन उत्तरत-(केवलं प्रश्नद्वयम्) (1/2 × 2 = 1)
(एक शब्द में उत्तर लिखिए) (केवल दो प्रश्न)

(क) कीदृशं भक्ष्यम् अस्ति?
(ख) कीदृशं जलं नास्ति?
(ग) समलं किमस्ति?
उत्तर:
(क) कुत्सितवस्तुमिश्रितं
(ख) निर्मलं जलं
(ग) धरातलम्

(आ) पूर्णवाक्येन उत्तरत-(केवलं प्रश्नद्वयम्) (1 × 2 = 2)
पूर्ण वाक्य में उत्तर लिखिए। (केवल दो प्रश्न)

(क) कीदृशं वायुमण्डलम्?
(ख) बहु शुद्धीकरणं कुत्र करणीयम्?
(ग) वायुमण्डलं सर्वदा किं करोति?
उत्तर:
(क) भृशं दूषितं वायुमण्डलम् अस्ति।
(ख) बहि: अन्तः जगति या (बहिरन्तर्जगति) बहु शुद्धिकरणं करणीयम्।
(ग) वायुमण्डलं सर्वदा भृशं दुषितं न हि निर्मलं जलम् करोति।

(इ) निर्देशानुसारम् उत्तरत-(केवलं प्रश्नद्वयम्) (1 × 2 = 2)
निर्देशानुसार उत्तर लिखिए (केवल दो प्रश्न)

(क) ‘अत्यधिकम’ इत्यर्थे अत्र किं पदं प्रयुक्तम्।
(ख) धरातलं समलम् अस्ति’ इत्यत्र किं कर्तृपदम् अस्ति ?
(ग) ‘अभक्ष्यम्’ इति पदस्य किं विलोमपदं अत्र प्रयुक्तम?
उत्तर:
(क) भृशम्
(ख) धरातलं
(ग) भक्ष्यम:

CBSE Sample Papers for Class 10 Sanskrit Set 9 with Solutions

14. अधोलिखितं नाट्यांशं पठित्वा प्रदत्तप्रश्नानाम् उत्तराणि संस्कृतेन लिखत
(निम्नलिखित नाट्यांश को पढ़कर दिए गए प्रश्नों के उत्तर संस्कृत में लिखिए।)

निद्राभङ्गदुःखेन वनराजः सन्नपि तुच्छजीवैः आत्मनः एतादृश्या दुरवस्थया श्रान्तः सर्वजन्तून् दृष्ट्वा पृच्छति

सिंहः – (क्रोधेन गर्जन्) भोः! अहं वनराजः किं भयं न जायते? किमर्थ मामेवं तुदन्ति सर्वे मिलित्वा?
एक: वानरः – यतः त्वं वनराजः भवितुं तु सर्वथाऽयोग्यः। राजा तु रक्षक: भवति परं भवान् तु भक्षकः। अपि च स्वरक्षायामपि समर्थः नासि तर्हि कथमस्मान् रक्षिष्यसि?
अन्यः वानरः – किं न श्रुतां त्वया पञ्चतन्त्रोक्तिः यो न रक्षति वित्रस्तान् पीड्यमानान्परैः सदा। जन्तून् पार्थिवरूपेण स कृतान्तो न संशयः॥
काकः – आम् सत्यं कथितं त्वया-वस्तुतः वनराजः भवितुं तु अहमेव योग्यः ।

(अ) एकपदेन उत्तरत। (केवलं प्रश्नद्वयम्) (1/2 × 2 = 1)
(एक शब्द के उत्तर लिखिए) (केवल दो प्रश्न)

(क) सर्वजन्तून् दृष्ट्वा कः पृच्छति?
(ख) क्रोधेन क; गर्जति?
(ग) राजा क: भवति?
उत्तर:
(क) वनराजः
(ख) सिंह:
(ग) रक्षक:

(आ) पूर्णवाक्येन उत्तरत। (केवलं प्रश्नद्वयम्) (1 × 2 = 2)
(पूर्णवाक्य में उत्तर लिखिए) केवल दो प्रश्न

(क) सर्वे, मिलित्वा कम् एव तुदान्ति?
(ख) एक: वानर: वनराजं किं कथयति?
(ग) सिंह: क: भवति?
उत्तर:
(क) सर्वे मिलित्वा सिंह तुदन्ति।
(ख) एकः वानरः वनराजं कथयति यत् त्वं वनराजः भवितुं तु सर्वथाऽयोग्यः।
(ग) सिंह: भक्षक: भवति।

(इ) निर्देशानुसार उत्तरत। (केवलं प्रश्नद्वयम्) (1 × 2 = 2)
(निर्देशानुसारम् उत्तर लिखिए) (केवल दो प्रश्न)

(क) राजा तु रक्षकः भवति इत्यस्मिन् वाक्ये किं कर्तृपदं प्रयुक्तम्
(ख) ‘एतादृश्या दुरवस्थया’ इत्यत्र किं विशेषणपदं प्रयुक्तम?
(ग) ‘अवासाद’ इत्यर्थे अत्र किं पर्यायपदं प्रयुक्तम्?
उत्तर:
(क) राजा
(ख) दुरवस्थया
(ग) तुदन्ति

15.रेखाडितपदानि आधृत्य प्रश्न निर्माण करुत। (केवलं प्रश्नचतुष्टयम्) (1 × 4 = 4)
(रेखांकित पदों के आधार पर प्रश्न निर्माण विकल्पों में से चुनकर लिखिए।) (केवल चार प्रश्न)

(क) मयूरस्य नृत्यं प्रकृतेः आराधना।
(ख) ग्रामस्य आरक्षी एव चौरः आसीत्।
(ग) राजा तु रक्षकः भवति।
(घ) पिककाकयोः भेद: वसन्तसमये प्राप्ते?
(ङ) उद्याने खगानाम् कलरवं चेतः प्रसादयति।
उत्तर:
(क) कस्या:
(ख) कस्य
(ग) क:
(घ) कदा
(ङ) केषाम्

16. मञ्जूषातः समुचित-पदानि चित्वा अधोलिखितश्लोकस्यः अन्वयं पूरयत (1 × 4 = 4)
(मञ्जूषा से उचित पदों को चुनकर निम्नलिखित श्लोकों के अन्वय को पूरा कीजिए।)

यः इच्छत्यात्मनः श्रेयः प्रभूतानि सुखानि च।
न कुर्यादहितं कर्म स; परेभ्यः कदापि च ॥

अन्वयः (i)….. आत्मनः श्रेयः प्रभूतानि सुखानि (ii)………. सः च परेभ्यः (iii)………….. कर्म कदापि न (iv)……
उत्तर:
(i) य:
(ii) इच्छति
(iii) अहितं
(iv) कुर्यात्

मञ्जूषा-अहितं, कुर्यात्, यः, इच्छति
अथवा
मञ्जूषायाः साहाय्येन श्लोकस्य भावार्थे रिक्तस्थानानि पूरयित्वा पुन: लिखत।
(मञ्जूषा की सहायता से श्लोक के भावार्थ से रिक्त स्थानों को पूरा करके पुनः लिखिए।)

गुणी गुणं वेत्ति न वेत्ति निर्गुणो, बली बलं वेत्ति न वेत्ति निर्बलः।
पिको वसन्तस्य गुणं न वायसः, करी च सिंहस्य बलं न मूषकः॥

भावार्थ:-गुणवान् जनः एव (i)…… महत्त्वं जानाति। गुणहीनं गुणानां महत्त्वं न अवगच्छति। तथैव (ii)……. जनः बलस्य महत्वं जानाति, परन्तु दुर्बलः महत्त्व न अवगच्छति। वसन्त-ऋतोः महत्त्वं कोकिलः एव सम्यक् (iii)………… शक्नोति न तु काकः। एवमेव गजः सिंहस्य (iv) ……. महत्त्वं जानाति । मूषक: तस्य शौर्यम अवगन्तुं न शक्नोति।
उत्तर:
(i) गुणानाम्
(iii) अवगन्तुम्
(ii) बलवान्
(iv) पराक्रमस्य

मञ्जूषा-अवगन्तुम्, बलवान्, पराक्रमस्य, गुणानाम्

17. अधोलिखित-कथांशं समुचित-क्रमेण लिखत (1/2 × 8 = 4)
(निम्नलिखित कथांश को समुचित क्रम में लिखिए।)

(i) ग्रामवासिनःवराकमतिथिमेव च चौरं मत्वाऽभर्ल्सयम्।
(ii) सर्व वृत्तांतम् अवगत्य न्यायाधीशः तं निर्दोषम् अमन्यत् आरक्षिणं च दोषी।
(iii) शवः प्रावारकमपसार्य सर्व वृतान्तं निवेदितवान्।
(iv) पावस्थिते ग्रामे करुणापरो गृही तस्मै आश्रयं प्रायच्छत्।
(v) विचित्र युक्त्या न्यायाधीशः निर्णयं कृत्वा तं जनं ससम्मानं मुक्तवान्।
(vi) कश्चन चौरः रात्रौ तस्मिन् गृहे प्रविष्य मञ्जूषामेकां आदाय पलायितः।
(vi) निर्धनः जनः सायं समयेऽप्यसौ गन्तव्याद् दूरे आसीत्।
(vi) निशान्धकारे प्रसृते विजने प्रदेशे पदयात्रा न शुभावहा।
उत्तर:
(vii) निर्धनः जनः सायं समयेsप्यसौ गन्तव्याद् दूरे आसीत्।
(viii) निशान्धकारे प्रसूते विजने प्रदेशे पदयात्रा न शुभावहा।
(iv) पार्श्वस्थिते ग्रामे करुणापरो गृही तस्मै आश्रयं प्रायच्छत् ।
(vi) कश्चन चौर: रात्रौ तस्मिन् गृहे प्रविष्य मञ्जूषामेकां आदाय पलायितः।
(i) ग्राम वासिनः वराकमतिथिमेव च चौरं मत्वाऽभर्स्सयन् ।
(iii) शव: प्रावारकमपसार्य सर्वं वृतान्तं निवेदितवान्।
(ii) सर्वं वृत्तांतं अवगत्य न्यायाधीशः तं निर्दोषम् अमन्यत् आरक्षिणं च दोषी।
(v) विचित्र युक्त्या न्यायाधीशः निर्णयं कृत्वा तं जनं ससम्मानं मुक्तवान्।

18. अधोलिखितवाक्येषु रेखाङ्कितपदानां प्रसङ्गानुकूलम् उचितार्थं चित्वा लिखत-(केवलं प्रश्नत्रयम)
(निम्नलिखित वाक्यों में रेखांकित शब्दों के प्रसंगानुकूल उचित अर्थ चुनकर लिखिए।) (केवल तीन प्रश्न) 1(1 × 3 = 3)

(i) लवकुशयो: एकमेव उतरम् आसीत् ।
(क) उपवेष्टुम्
(ख) प्रतिवचनम्
(ग) अध्यासयितुम्
(घ) वचनम्
उत्तर:
(ख) प्रतिवचनम्

(ii) एकान्ते कान्तारे क्षणमपि सञ्चरणं स्यात् ।
(क) वने
(ख) जने
(ग) गहने
(घ) राजमार्गे
उत्तर:
(क) वने

(iii) उदिते सविता रक्तः।
(क) कषिता
(ख) चन्द्रमा
(ग) सूर्यः
(घ) पृथ्वी
उत्तर:
(ग) सूर्यः

(iv) ‘दिवसः‘ पर्यायवाची चित्वा लिखत्।
(क) भानुः
(ख) दिनः
(ग) निशाकरः
(घ) निशा
उत्तर:
(ख) दिनः

CBSE Sample Papers for Class 12 Hindi with Solutions

CBSE Sample Papers for Class 12 Hindi with Solutions 2022-2023

Solved CBSE Sample Paper Class 12 Hindi 2022-2023 with Solutions: Solving Pre Board CBSE Sample Papers for Class 12 Hindi with Solutions Answers 2022-2023 Pdf Download helps to understand the pattern of questions asked in the board exam. Know about the important concepts to be prepared for CBSE Class 12 Hindi board exam and Score More marks. Here we have given CBSE Class 12 Hindi Sample Papers 2023.

Board – Central Board of Secondary Education (CBSE)
Subject – CBSE Class 12 Hindi
Year of Examination – 2022-2023

CBSE Sample Paper Class 12 Hindi 2022-2023 with Solutions

CBSE Class 12 Hindi Sample Papers 2023 with Solutions

Hindi Sample Paper Class 12 Question Paper Design 2022-23

खंड प्रश्नों के प्रकार प्रश्नों की संख्या कुल अंक

खंड-‘अ’
वस्तुपरक/बहुविकल्पीय

अपठित गद्यांश पर आधारित MCQs 1 × 10 = 10 10
अपठित पद्यांश पर आधारित MCQs 1 × 5 = 5 5
अभिव्यक्ति और माध्यम पर आधारित MCQs 1 × 5 = 5 25
आरोह भाग-2 पर आधारित MCQs (गद्यांश/काव्यांश) 1 × 10 = 10
वितान भाग-2 पर आधारित MCQs 1 × 10 = 10

खंड-‘ब’
वर्णनात्मक प्रश्न

रचनात्मक लेखन 6 × 1 = 6 20
कहानी लेखन 3 × 2 = 6
पत्रकारिता और जनसंचार लेखन 4 × 2 = 8
आरोह भाग-2
काव्य खंड पर आधारित प्रश्न 3 × 2 = 6 20
काव्य खड पर आधारित प्रश्न 2 × 2 = 4
गद्य खंड पर आधारित प्रश्न 2 × 3 = 6
गद्य खंड पर आधारित प्रश्न 2 × 2 = 4
कुल अंक – 80

We hope these CBSE Sample Papers for Class 12 Hindi with Solutions 2022-2023 will help in self-evaluation. Stay tuned for further updates on the CBSE Sample Paper of Hindi Class 12 2023 for their exam preparation.